Download as docx, pdf, or txt
Download as docx, pdf, or txt
You are on page 1of 134

Study Questions BASIC AND APPLIDE

CHAPETR 1
1. Which of the following is not a basic component of a QA program?
a. calibration
b. preventive maintenance
c. viral marker testing
d. record keeping
2. What criteria apply to the correction of manual records?
a. the original entry is neither obliterated nor deleted
b. the person making the correction dates and initials the change
c. the item to be corrected is crossed off with a single line
d. all of the above
3. What statement does not apply to the definition of an SOP?
a. step-by-step instructions
b. used to monitor accuracy and precision
c. written in compliance with cGMPs
d. written in compliance with manufacturer’s recommendations
4. What is the purpose of competency assessment?
a. identify employees in need of retraining
b. evaluate an individual’s level of knowledge during a job interview
c. identify employees who need to be fired
d. all of the above
5. What statement does not apply to the performance of internal audits?
a. help identify problems early
b. ensure continuous quality improvement efforts
c. are used solely for the purpose of identifying “troublemakers”
d. are one of the many responsibilities of the QA unit
6. Employees are the core of an organization. What item reinforces this statement?
a. employees must be trained
b. employees must report errors without fear of reprisal
c. employees are a key part of problem solving
d. all of the above
7. Where can cGMPs applicable to the blood industry be located?
a. AABB Standards for Blood Banks and Transfusion Services
b. SOP manual
c. FDA Code of Federal Regulations
d. QA manual

8. What term describes the process to identify what factors contributed to the occurrence of a
nonconformance?
a. root-cause analysis
b. accident reports
c. internal audits
d. FDA inspections
For questions 9 through 11, match the following descriptions with
the appropriate organization:
12. Who is ultimately responsible for a safety program in the laboratory?
a. CDC
b. the employer or laboratory director
c. OSHA
d. the employee
13. What organization publishes the Technical Manual, which is often used for principal guidelines in
the blood bank?
a. CAP
b. ISBT
c. AABB
d. OSHA
14. Which of the following items is mandated by law for all blood banks?
a. blood bank refrigerator
b. written laboratory safety program
c. BSC
d. foot-operated hand wash
15. What designation is given to goggles, face shields, and splash barriers?
a. personal protective equipment
b. not necessary unless working with HIV-positive or HBV-positive specimens
c. mandated at all times when working with blood specimens and blood products
d. provided by the employee if needed
16. What is the name of the policy of treating all body substances as potentially harmful, regardless
of the patient diagnosis?
a. OSHA Exposure Program
b. isolation guidelines
c. AABB safety policy
d. universal precautions
17. In addition to wearing gloves, what is the most effective defense for infection control and safety?
a. goggles
b. laboratory coats
c. posting warning signs
d. hand washing
18. Select the most economical disinfectant to use for decontamination.
a. 1:10 fresh solution of sodium hypochlorite (bleach)
b. 1:5 solution of household Lysol
c. 1:15 solution of sodium hydroxide
d. detergent and water mixture
19. When should an accident or injury be reported?
a. injury may result in a fatality
b. injury involves possible infection with HIV or HBV
c. accident involves nonemployees or jeopardizes a patient
d. at the time the accident or injury occurs
20. Select one of the best ways to protect employees and keep a safe laboratory environment.
a. health insurance
b. safety education
c. rest breaks
d. fluid-repellent laboratory coats
CHAPTR 2
1. What cells can produce antibodies?
a. natural killer cells
b. T cells
c. macrophages
d. plasma cells
2. What term describes the number of antigen-binding sites per molecule of antibody?
a. valency
b. bivalent
c. isotype
d. idiotype
3. Select the term that describes the unique part of the antigen that is recognized by a corresponding
antibody.
a. immunogen
b. epitope
c. avidity
d. clone
4. What classification of molecules does not make a good immunogenic substance?
a. protein
b. carbohydrate
c. lipid
d. glycoprotein
5. What is the chemical composition of an antibody?
a. protein
b. lipid
c. carbohydrate
d. glycoprotein
6. Where is the antigen located in a hemagglutination test?
a. on the red cell membrane
b. secreted by the red cell
c. in the red cell nucleus
d. in the plasma or serum
7. Which of the following situations can enhance hemagglutination reactions?
a. testing at a temperature higher than 37° C
b. increasing the incubation time
c. increasing the antigen concentration
d. making the pH greater than 7
8. What term describes molecules that bind to an antigen to increase phagocytosis?
a. opsonins
b. cytokines
c. haptens
d. isotypes
9. After performing a tube test, the supernatant of the test was pinkish and the red cell button was
small. How do you interpret the result of this test?
a. false-positive
95
b. false-negative
c. positive
d. negative
10. How would you grade an agglutination reaction if you observe many small agglutinates in a
background of free cells in tube testing?
a. 1+
b. 2+
c. 3+
d. 4+
11. To determine the presence of a red cell antibody in a patient’s sample, what is the source of
antigen?
a. commercial reagent red cells
b. commercial antisera
c. patient serum
d. patient red cells
12. How does complement activation demonstrate within the body?
a. cell lysis
b. enhanced cell clearance
c. neutrophil activation
d. generation of vasoactive amines
e. all of the above
13. After addition of anti-A reagent to a patient’s red cell suspension, agglutination was observed.
The result with anti-B reagent was negative. What is the interpretation of this patient’s ABO
typing?
a. patient is group B
b. patient is group A
c. cannot interpret this test
d. false-positive result

For questions 14 through 25, match the characteristic with the


correct
immunoglobulin class
CHAPTER3
1. What is the purpose of including a reagent control when interpreting group AB, D-positive red
cells after testing with a low-protein anti-D reagent?
a. to detect false-positive agglutination reactions
b. to detect false-negative agglutination reactions
c. to identify a mix-up with a patient’s sample
d. to confirm ABO typing results
2. What characteristic is associated with monospecific AHG reagents?
a. increase the dielectric constant in vitro
b. contain either anti-IgG or anti-C3d antibody specificities
c. are not useful in identifying the molecule causing a positive DAT
d. contain human IgG or complement molecules
3. You have added IgG-sensitized red cells to a negative indirect antiglobulin test. You observe
agglutination in the tube. What situation was not controlled for in testing by adding these control
cells?
a. the addition of patient serum
b. the addition of AHG reagent
c. adequate washing of cell suspension
d. adequate potency of AHG reagent
4. Part of the daily quality control in the blood bank laboratory is the testing of reagent antisera with
corresponding antigen-positive and antigen-negative red cells. What does this procedure ensure?
a. antibody class
b. antibody titer
c. antibody specificity
d. antibody sensitivity
5. Why are group O red cells used as a source for commercial screening cells?
a. anti-A is detected using group O cells
b. anti-D reacts with most group O cells
c. weak subgroups of A react with group O cells
d. ABO antibodies do not react with group O cells
6. Where can you locate information regarding reagent limitations?
a. SOPs
b. blood bank computer system
c. product inserts
d. product catalogs
7. What regulatory agency provides licensure for blood banking reagents?
a. AABB
b. FDA
c. American Red Cross
d. College of American Pathologists
8. What antibodies are present in polyspecific AHG reagent?
a. anti-IgG
b. anti-IgM and anti-IgG
c. anti-IgG and anti-C3d
d. anti-C3d
144
9. In which source are the regulations regarding the manufacturing of blood banking reagents
published?
a. Code of Federal Regulations
b. AABB Standards for Blood Banks and Transfusion Services
c. AABB Technical Manual
d. AABB Accreditation Requirements Manual
10. After the addition of anti-D reagent to a patient’s red cell suspension, agglutination was
observed. The result with anti-A reagent was negative. What is the interpretation of this patient’s
D typing?
a. patient is D-negative
b. patient is D-positive
c. cannot interpret the test
d. invalid result
11. Select the reagent to use for detection of unexpected red cell antibodies in a patient’s serum
sample.
a. A1 and B cells
b. panel cells
c. IgG-sensitized cells
d. screening cells
12. Select the method that uses the principle of sieving to separate larger agglutinates from smaller
agglutinates in Ag-Ab reactions.
a. gel technology
b. solid-phase adherence
c. microplate

d. none of the above


13. What source of antibody is selected to determine the specificity of a red cell antigen in a patient
sample?
a. commercial reagent red cells
b. commercial antisera
c. patient serum
d. patient plasma
14. What source of antigen is selected to determine the presence of a red cell antibody in a patient
sample?
a. commercial reagent red cells
b. commercial antisera
c. patient serum
d. patient’s red cells
15. What reagents are derived from plant extracts?
a. panel cells
b. commercial anti-B
c. lectins
d. antiglobulin reagents
CHAPTER4
1. Which of the following describes the expression of most blood group inheritance?
a. dominant
b. recessive
c. sex-linked
d. codominant
2. With which of the following red cell phenotypes would anti-Jka react most strongly?
a. Jk(a−b+)
b. Jk(a+b−)
c. Jk(a+b+)
d. Jk(a−b−)
3. In relationship testing, what is the criterion for a direct exclusion?
a. genetic marker is absent in the child but present in the mother and alleged father
b. genetic marker is absent in the child, present in the mother, and absent in the alleged father
c. genetic marker is present in the child, absent in the mother, and present in the alleged father
d. genetic marker is present in the child but absent in both the mother and the alleged father
4. Which of the following items is a useful genetic marker for relationship testing?
a. all races have the same gene frequencies
b. the genetic system is polymorphic
c. there are no amorphic genes
d. recombination is common
5. What term describes the inheritance of two of the same alleles from each parent?
a. homozygous
b. allele
c. heterozygous
d. syntenic
6. What term describes alternate forms of a gene at given genetic loci?
a. alleles
b. amplicons
c. nucleotides
d. amorphs
7. What is the name of the technique that uses a small amount of DNA and amplifies it for
identification?
a. RFLP
b. DNA probe
c. PCR
d. gene mapping
8. What is the term for a gene that does not express a detectable product?
a. an amorph
b. a cis gene
c. a trans gene
d. a regulator gene
9. What is the name of synthetic single-stranded DNA that determines the sequence of DNA for
amplification in the PCR reaction?
a. amplicons
181
b. nucleotides
c. DNA primer
d. Taq polymerases
10. What is the mode of inheritance when the genes are located close together on the same
chromosome?
a. inherited as a haplotype
b. crossover
c. show independent assortment
d. suppress each other
11. In molecular techniques for HLA typing, which high-resolution method analyzes the nucleotide
sequence to define the allele?
a. SSP
b. SSO
c. STR
d. SBT
12. Where is a gene inherited in a cis position to another gene located?
a. on an opposite chromosome
b. on a different chromosome number
c. on the same chromosome
d. antithetical
13. In PCR testing, the initial step involves adding the DNA in question to a mixture of Taq
polymerase, excess nucleotides, MgCl2, and primers. Where does the amplification process take
place?
a. flow cytometer
b. capillary array sequencer
c. thermal cycler
d. electrophoresis chamber
14. Patient RJ types as group O with anti-D and anti-K alloantibodies and requires one unit of RBCs.
What percentage of the white population would be compatible with her serum (group O = 0.45; D
− = 0.15; K− = 0.91)?
a. 3%
b. 6%
c. 20%
d. 50%
15. Molecular tests on samples from hematopoietic progenitor cell transplant recipients are used to
determine if engraftment is successful. What is the name of this method?
a. sequence-specific primers (SSPs)
b. sequence-based typing (SBT)
c. short tandem repeats (STRs)
d. sequence-specific oligonucleotide probes (SSOPs)
16. Select the phrase that defines the term antithetical.
a. similar gene
b. opposite allele
c. opposite antigen
d. heterozygous gene
17. What term is used when red cells with the genotype MM react stronger with anti-M than red
cells with genotype MN?
182
a. trans effect
b. dosage effect
c. cis effect
d. suppressor effect
18. Which term defines a group of antigens on the red cell membrane that share related serologic
properties and genetic patterns of inheritance?
a. meiosis
b. regulator system
c. blood group system
d. molecular classification
19. What genetic information is provided by hemagglutination testing for red cell antigens?
a. genotype
b. phenotype
c. zygosity
d. polymorphism
20. What genetic information is provided by DNA assays for red cell antigens?
a. loci location
b. phenotype
c. polymorphism
d. genotype
CHAPTER5
1. Given the following ABO typing results, what is a potential conclusion for these results?

a. expected results for a group O individual


b. expected results for a group AB individual
c. discrepant results; patient has A antigen on red cells with anti-A in serum
d. discrepant results; patient has B antigen on red cells with no anti-B in serum
2. What are the gene products of the A and B genes?
a. glycolipids
b. glycoproteins
c. oligosaccharides
d. transferase enzymes
For questions 3 through 5, use the following ABO typing results:

3. What is the ABO interpretation?


a. group O
b. group A
c. group B
d. group AB
4. What ABO phenotypes would be compatible if the patient required a transfusion of RBCs?
a. group AB, O, A, or B
b. group O or B
c. group AB or O
d. only group O
5. What ABO phenotypes would be compatible if the patient required a transfusion of fresh frozen
plasma?
a. group AB, O, A, or B
b. group O or B
c. group AB or O
d. only group O
6. What term describes using known sources of reagent antisera (known antibodies) to detect ABO
antigens on a patient’s red cells?
264
a. Rh typing
b. reverse grouping
c. direct antiglobulin test
d. forward grouping
7. Which result is discrepant if the red cell typing shown in the following chart is correct?

a. negative reaction with group B cells


b. positive reaction with anti-B
c. negative reaction with group A1 cells
d. no discrepancies in these results
8. What ABO antibody is expected in this patient serum based on the following information?

a. anti-B
b. anti-A
c. anti-A and anti-B
d. none
9. According to Landsteiner’s rule, if a patient has no ABO antibodies after serum testing, what
ABO antigens are present on the patient’s red cells?
a. A
b. B
c. both A and B
d. none
10. Select the ABO phenotypes, in order from most frequent to least frequent, that occur in whites:
a. A, B, O, AB
b. O, A, B, AB
c. B, A, AB, O
d. AB, O, B, A
11. Which of the following statements is true about ABO antibody production?
a. ABO antibodies are present in newborns
b. ABO titers remain at constant levels throughout life
c. ABO antibodies are stimulated by bacteria and other environmental factors
d. All of these statements are true
12. What immunoglobulin class is primarily associated with ABO antibodies?
a. IgA
b. IgG
265
c. IgE
d. IgM
13. What immunodominant sugar confers B blood group specificity?
a. D-galactose
b. L-fucose
c. N-acetylgalactosamine
d. L-glucose
14. An individual has the genotype of AO, hh. What antigens would be present on the red cells of
this individual?
a. A only
b. A and H
c. A and O
d. none of the above
15. What gene controls the presence of soluble H substance in saliva?
a. H
b. A
c. Se
d. B
16. Which lectin agglutinates A1 red cells?
a. Dolichos biflorus
b. Ulex europaeus
c. Dolichos europaeus
d. Ulex biflorus
17. What immunodominant sugar determines the specificity of H antigens?
a. D-galactose
b. L-fucose
c. N-acetylgalactosamine
d. L-glucose
18. Which of the following situations may produce ABO discrepancies in the serum testing?
a. newborn
b. patient with hypogammaglobulinemia
c. cold alloantibody
d. all of the above
19. What soluble antigen forms are detectable in saliva based on the following genotype: AB, HH,
SeSe?
a. none (nonsecretor)
b. only H
c. A, B, and H
d. A and B
20. Which ABO discrepancy is the best explanation for the results shown in the following chart?

266
a. an elderly patient
b. subgroup of A
c. deterioration of reagents
d. hypogammaglobulinemia
CHAPTER6
1. How is the Rh genotype CDE/cDE written in Wiener notation?
a. R0R1
b. RyR2
c. R2R1
d. RzR2
2. In Rosenfield notation, the phenotype of a donor may be written as Rh:1,−2,−3,4,5. What is the
correct phenotype in Fisher-Race (CDE) notation?
a. cDe
b. CcDe
c. CcDE
d. CDEe
3. Anti-f was identified in a patient. Because commercial antisera are not available, what is the best
course of action to locate compatible RBC units?
a. crossmatch E-negative units
b. contact the rare donor registry
c. release O, D-negative units
d. crossmatch c-negative units
4. A patient’s Rh phenotype was D+, c+, e+, C−, E−. What is the most likely race of this donor?
a. black
b. white
c. Asian
d. Native American
5. What test is needed to determine weak D antigen status?
a. the IAT
b. the DAT
c. anti-Du typing sera
d. anti-D antisera with a LISS potentiator
6. Which of the following red cell genotypes would react negatively with anti-G?
a. R0r
b. rr
c. R2r
d. r′r
7. What statement is true relative to the results of a weak D test performed on a patient with a
positive direct antiglobulin test?
a. accurate as long as the check cells were positive
b. unreliable because of immunoglobulins already on the cell
c. reliable if a high-albumin anti-D was used
d. false-negative because of antibody neutralization
8. Which of the following is associated with the Rhnull phenotype?
a. elevated D antigen expression
b. increased LW antigen expression
c. the Bombay phenotype
d. red cell membrane abnormalities
9. What is the name of the blood group system that was originally identified as the Rh blood group
297
system?
a. Kell
b. Lutheran
c. Lewis
d. LW
10. A donor tested D-negative using commercial anti-D reagent. The weak D test was positive. How
should the RBC unit be labeled?
a. D-positive
b. D-negative
c. D variant
d. varies with blood bank policy
11. Which offspring is not possible from a mother who is R2r and a father who is R1r?
a. DcE/DcE
b. DCe/DcE
c. DcE/ce
d. ce/ce
12. What is the common form of antibodies produced upon exposure to Rh blood group system
antigens?
a. naturally occurring IgM
b. immune IgG
c. immune IgM
d. naturally occurring IgG and IgM
13. Which of the following genotypes is heterozygous for the C antigen?
a. R1r
b. R2R2
c. R1R1
d. r′r′
14. What is the likelihood that two heterozygous D-positive parents will have a D-negative child?
a. less than 1%
b. not possible
c. 25%
d. 75%
15. Which of the following genotypes could make anti-Ce (Rh7)?
a. R2R2
b. R1R0
c. R1R2
d. r′r
16. Which of the following phenotypes would react with anti-f?
a. rr
b. R1R1
c. R2R2
d. R1R2
17. A donor is tested with Rh antisera; given the following results, what is the most probable Rh
genotype?
Rh Antisera Donor RBC Reaction
Anti-D +
Anti-C +
Anti-E 0
Anti-c +
Anti-e +
Rh control 0
+, Agglutination; 0, no agglutination.
a. R1R1
b. R1r
c. R0r
d. R2r
18. Anti-D was detected in the serum of a D-positive person. What is a possible explanation?
a. the antibody is really anti-G
b. compound antibody was formed
c. regulator gene failure
d. missing antigen epitope
19. An antibody to the E antigen was identified in a patient who received multiple transfusions.
What is the most likely phenotype of the patient’s red cells?
a. R1R1
b. R2R2
c. R1r
d. r′r′
20. The regulator gene RHAG:
a. is inherited on chromosome 1
b. is responsible for the Rhmod phenotype
c. must be inherited to express LW antigens
d. is responsible for the D-deletion phenotype
Answers to Study Questions can be found on page
CHAPTER 7
1. Which blood group system possesses the Jsb and Kpa antigens?
a. Duffy
b. Lutheran
c. Kell
d. Kidd
2. Which of the following antibodies is commonly associated with delayed transfusion reactions?
a. anti-Lua
b. anti-S
c. anti-Jkb
d. anti-M
3. Which phenotype is associated with a resistance to Plasmodium vivax?
a. Fy(a−b−)
b. Jk(a−b−)
c. Le(a−b−)
d. Lu(a−b−)
4. Enzyme-treated reagent red cells used in antibody identification can enhance antibody reactions.
Which of the following antibodies is not enhanced with the use of enzyme-treated red cells?
a. anti-M
b. anti-Lea
c. anti-Jkb
d. anti-I
5. Which of these antibodies are typically IgM? You may select more than one answer.
a. anti-K
b. anti-S
c. anti-U
d. anti-N
e. anti-Leb
f. anti-Jkb
g. anti-P1
6. Which of the following reagents destroys the Kell system antigens?
a. ficin
b. albumin
c. PEG
d. DTT
7. Which blood group system’s antigens are associated with glycophorin A and glycophorin B?
a. Duffy
b. Kidd
c. Lewis
d. MNS
8. Which of the following antibodies is characteristically clinically insignificant?
a. anti-Kpb
b. anti-S
c. anti-Leb
d. anti-Fya
365
9. Which of the following red cell phenotypes is associated with the McLeod phenotype?
a. Rhnull phenotype
b. K0 phenotype
c. U-negative phenotype
d. absence of Kx antigens
10. What statement is true regarding a phenotype of Lu(a−b−)?
a. rare in whites but not blacks
b. rare in blacks but not whites
c. rare in all populations
d. common in all populations
11. What is the common specificity of cold autoantibodies?
a. I
b. M
c. P1
d. S
12. What alloantibody is associated with individuals possessing the p phenotype?
a. anti-P2
b. anti-p
c. anti-P
d. anti-Tja
13. Select the alleles within the Lewis system.
a. Le, le
b. Lea, Leb
c. Le, Se, H
d. Le, Le
14. Which of the following antibodies requires the antiglobulin test for in vitro detection?
a. anti-M
b. anti-P1
c. anti-U
d. anti-I
15. What procedure helps distinguish between an anti-Fya and anti-Jka in an antibody mixture?
a. lowering the pH of the patient’s serum
b. using a thiol reagent
c. testing at colder temperatures
d. testing ficin-treated panel cells
16. What statement is true relative to anti-K?
a. agglutinates in IAT phases of the antibody screen
b. is usually of the IgM antibody class
c. does not agglutinate with K+k+ panel cells
d. loses reactivity in enzyme phases
17. Which of the following antigens is poorly expressed on cord blood cells?
a. K
b. M
c. Leb
d. D
18. Reagent antibody screening cells may not detect antibodies directed against low-incidence
366
antigens. Which antibody is most likely to go undetected?
a. Vel
b. S
c. Kpa
d. K
19. Select the disease commonly associated with the McLeod phenotype.
a. infectious mononucleosis
b. chronic granulomatous disease
c. Hodgkin disease
d. PCH
20. Which set of antibodies could you possibly find in a patient with no history of transfusion or
pregnancy?
a. anti-I, anti-S, and anti-P1
b. anti-M, anti-c, and anti-B
c. anti-A, anti-I, and anti-D
d. anti-B, anti-I, and anti-Lea
21. What is the most likely Lewis phenotype of a nonsecretor?
a. Le(a−b−)
b. Le(a+b+)
c. Le(a+b−)
d. Le(a−b+)
22. Anti-N is identified in a white patient who requires a blood transfusion. If 10 donor RBCs were
tested, how many of these units would most likely be negative for the N antigen?
a. 0
b. 3
c. 7
d. 10
23. Which of the following antibodies is neutralized by pooled human urine?
a. anti-Csa
b. anti-Sda
c. anti-Ch
d. anti-Vel
24. The red cells of a donor have a U-negative phenotype. What red cell antibody would not react
with these red cells?
a. anti-M
b. anti-S
c. anti-P1
d. anti-K
25. Which of the following blood group systems is associated with a depression of the antigens in
chronic granulomatous disease?
a. Duffy
b. Kidd
c. P
d. Kell
26. Which of the following cells expresses HLA class II antigens?
a. B cells
b. platelets
367
c. erythrocytes
d. T cells
27. The mixed lymphocyte culture (MLC) is an older technique in the HLA laboratory used to
determine:
a. HLA-A antigens
b. HLA-C antigens
c. HLA antibody identification
d. HLA-D antigens and compatibility
28. What term describes a poor platelet response to transfused platelets due to the presence of HLA
or platelet antibodies?
a. haplotype
b. refractory
c. vasoactive
d. thrombocytopenia
29. How do patients become sensitized to HLA antigens?
a. pregnancies
b. blood transfusions
c. previous transplants
d. all of the above
30. Which of the following HLA antigens is not characterized as class I?
a. C
b. A
c. DR
d. B
CHAPTR8
1. What is the purpose of the antibody screen?
a. detects most clinically significant antibodies
b. detects all low-frequency antibodies
c. helps to distinguish between an alloantibody and autoantibody
d. can be omitted if the patient has no history of antibodies
2. Select a characteristic of HTLA antibodies.
a. typically react at room temperature
b. can be enhanced with PEG
c. are usually clinically insignificant
d. are associated with HDFN
3. Which statement is a characteristic associated with anti-I?
a. It has weaker reactions with stored blood.
b. It can be neutralized with commercially prepared substance.
c. It reacts best at 37° C.
d. It does not react with cord blood cells.
4. A multiple antibody problem was resolved using enzymes. Panel cell reactions were eliminated
for one antibody specificity after testing with enzyme-treated red cells. Which of the following
antibodies was probably present?
a. anti-c
b. anti-I
c. anti-Jka
d. anti-Fya
5. What is the definition of an antibody demonstrating dosage?
a. homozygous red cells were stronger
b. heterozygous red cells were stronger
c. red cells reacted best with PEG
d. red cells reacted best at 4° C
6. The neutralization technique was performed on a sample containing an anti-Leb. The control and
the Lewis-neutralized sera were both negative when retested with panel cells. How would you
interpret the results of this test?
a. the anti-Leb was successfully neutralized and no underlying antibodies were found
b. the panel cells were not washed sufficiently
c. the sample was probably diluted
d. the antibody originally identified was probably not anti-Leb
7. Which p-value is achieved in antibody identification with the rule of three?
a. 0.09
b. 0.02
c. 0.05
d. 0.15
8. What would the DAT results demonstrate if the test was performed on a clotted sample stored at
4° C?
a. in vivo complement attachment
b. in vivo IgG attachment
c. in vitro complement attachment
414
d. in vitro IgM attachment
9. Which of the following antibodies may not be detected in the antibody screen?
a. anti-Jsb
b. anti-V
c. anti-k
d. anti-s
10. What is the name of the procedure that removes intact antibodies from the red cell membranes?
a. autoadsorption
b. neutralization
c. enzyme pretreatment
d. elution
11. What is the name of the procedure that removes antibody from serum or plasma using the
individual’s own red cells?
a. autoadsorption
b. differential adsorption
c. neutralization
d. elution
12. An antibody reacted in the screen at 37° C and did not react at the AHG phase. Which of the
following antibodies would you suspect?
a. anti-s
b. anti-e
c. anti-N
d. anti-Jka
13. What is the period when no antigen typing should be performed on a patient’s red cells after
transfusion?
a. up to 30 days
b. up to 2 months
c. up to 3 months
d. up to 6 months
14. DTT is useful in evaluating a sample when which antibody is suspected?
a. anti-Jsb
b. anti-Kpb
c. anti-k
d. all of the above
15. Why are additional procedures required when working up a warm autoantibody?
a. identify the warm autoantibody specificity in the serum
b. locate RBC units that are compatible with the autoantibody
c. identify potential underlying alloantibodies
d. identify the antibodies coating the red cells

CHAPTER9

1. What test detects serologic incompatibility between donor RBCs and recipient serum?
a. antibody screen
b. crossmatch
c. DAT
d. autologous control
2. What incompatibilities are detected in the antiglobulin phase of a crossmatch?
a. IgM alloantibodies in recipient’s serum
b. ABO incompatibilities
c. IgG alloantibodies in recipient’s serum
d. room-temperature incompatibilities
3. What tests are included in compatibility testing?
a. blood typing of recipient
b. antibody screening of recipient
c. crossmatch
d. all of the above
4. One group B, D-positive unit of RBCs is received in the transfusion service. What repeat testing is
required on this donor unit?
a. ABO typing only
b. ABO and D typing
c. ABO, D, and weak D typing
d. ABO and D typing; antibody screen
5. What ABO and D types are selected for RBC units issued to a patient in emergency release?
a. group O, D-positive
b. group O, D-negative
c. group A, D-positive
d. group AB, D-negative
6. What antibodies are detected in the immediate-spin crossmatch?
a. Rh antibodies
b. high-titer, low-avidity antibodies
c. ABO antibodies
d. Kell antibodies
For questions 7 and 8, use the following information. Current pretransfusion testing on John
Smith reveals a negative antibody screen with a previous history of anti-K. He is group A, Dpositive.
7. Which of the following crossmatch procedures is performed to identify compatible units?
a. immediate-spin crossmatch
b. electronic crossmatch
c. antiglobulin crossmatch
d. none of the above
8. Given the following inventory, which donor unit should be selected for crossmatching?
a. group A, D-positive, K+k+
b. group A, D-negative, K−k+
c. group O, D-positive, K+k−
d. group O, D-negative, K+k−
450
9. What information is required by AABB Standards for a labeled blood sample for the blood bank?
a. name
b. name, unique identification number, date of collection
c. name, unique identification number, date of collection, physician name
d. two independent identifiers
10. Which of the following products is crossmatched with the recipient if the unit contains greater
than 2 mL of RBCs?
a. granulocyte concentrates
b. plasma
c. platelets
d. cryoprecipitate
11. A patient who phenotypes as group AB, D-negative requires 1 unit of plasma. Which of the
following units of plasma would be best for transfusion?
a. group A, D-negative
b. group B, D-positive
c. group AB, D-positive
d. group O, D-negative
12. A donor’s RBC phenotype gave the following results when checked in the transfusion service.
The donor unit was labeled group A, D-negative. What is the next step?
a. transfuse as a group A, D-negative
b. transfuse as a group A, D-positive
c. discard the unit
d. notify the collection facility
13. An antiglobulin crossmatch is performed with a donor RBC unit. The antiglobulin crossmatch
result is a 2+ agglutination reaction. What is the most likely explanation for this result?
a. recipient’s RBCs are demonstrating polyagglutination
b. recipient’s RBCs have a low-frequency antigen
c. recipient possesses an IgG alloantibody
d. recipient possesses a cold autoantibody
14. A recipient’s antibody screen is negative; however, the recipient is incompatible with the
selected donor unit. Select a possible explanation for these results.
a. recipient RBCs possess a high-frequency antigen
b. recipient has a warm autoantibody
c. recipient possesses an antibody to a low-frequency antigen
d. recipient RBCs possess a cold autoantibody
True or False
____ 15. The computer crossmatch is easily implemented in the blood bank and does not require
validation.
____ 16. A crossmatch detects most errors in the identification of antigens on patient’s red cells.
____ 17. A crossmatch demonstrating a 2+ agglutination is interpreted as compatible.
____ 18. An immediate-spin crossmatch of a D-positive recipient with a D-negative donor unit is
451
usually incompatible.
____ 19. The computer crossmatch requires two ABO and D phenotypes on the recipient.
____ 20. A crossmatch prevents the immunization of the recipient to blood group antigens.
____ 21. A type and screen protocol provides a mechanism to increase the number of
uncrossmatched donor units in inventory.
____ 22. The only component that requires crossmatching is a unit of RBCs.
____ 23. Group O plasma is considered the universal donor of plasma products.
____ 24. A good practice for recipient’s sample is to label with full name, a second unique identifier,
date collected, and some means of identifying the phlebotomist.
____ 25. If a patient has been pregnant within the last 3 months before transfusion, the
pretransfusion sample must be no more than 3 days old at time of intended transfusion

CHAPTER10
1. Which of the following statements is true regarding interpretation of SPRCA results?
a. Grading reactions are based on a 1-to-4 grading scale where 4+ is a strong positive.
b. A tight button of cells on the bottom of the well would be interpreted as a positive reaction.
c. Hemagglutination assay results are interpreted the same as SPRCA results.
d. An evenly distributed layer of cells on the well surface is a negative reaction.
2. What are the indicator cells used in SPRCA assays?
a. IgM-coated red cells useful in determining the presence of ABO antibodies
b. cells used to agglutinate IgG antibodies to form a pellet in the well after centrifugation
c. IgG-coated red cells that cross-link with IgG antibodies attached to the well
d. check cells that are added to negative reactions to validate the washing step
3. In the gel test, what is a button of cells at the bottom of the well called?
a. 4+ positive reaction
b. 1+ positive reaction
c. negative reaction
d. invalid reaction
4. Which method is used in the automated Immucor and Bio-Rad systems for ABO and D
phenotype?
a. solid-phase red cell adherence assay (SPRCA)
b. enzyme-linked immunosorbent assay (ELISA)
c. reverse-passive hemagglutination (RPHA)
d. hemagglutination in a microwell
5. Where can false-negative reactions occur in both the gel and solid-phase testing?
a. failing to centrifuge
b. not mixing reagent red cells sufficiently
c. incubation time was too short
d. all of the above
6. Mixed-field reactions were observed in the ID-MTS Gel Test for ABO and D phenotype when the
test was automated. What is the most likely cause of this observation?
a. improper pipetting technique
b. the use of contaminated reagents
c. transfusion of group O cells to an A or B patient
d. centrifugation error
7. After centrifugation using the ID-MTS Anti-IgG gel cards, a layer of agglutinated cells was
observed at the top of two of the three screening cell microtubes. What should be done next?
a. repeat the test
b. proceed to an antibody identification panel
c. perform a DAT to determine whether the patient has an autoantibody
d. verify the negative reaction with check cells
8. What tests are performed with the MTS Monoclonal A/B/D Grouping Gel Card?
a. typing A, B, and D antigen
b. antiglobulin crossmatches
c. typing Rh system antigens
d. direct antiglobulin tests
9. What test uses the Capture-R Select microwell strips?
485
a. direct antiglobulin test
b. antiglobulin testing of selected panel cells
c. weak D test
d. all of the above
10. Select the method that uses the principle of sieving to separate larger agglutinates from smaller
agglutinates in Ag-Ab reactions.
a. gel technology
b. solid-phase
c. microplate
d. none of the above
True or False
____ 11. Bar-code technology is used in automated instruments to ensure positive sample identity.
____ 12. LIS interfaces link the automated test system with a laboratory information system for
reporting results.
____ 13. Automation should make it easier to do things the right way and more difficult to do the
wrong things.
____ 14. When selecting an automation partner, a laboratory does not need to determine its
requirements in terms of customer support, training, and service.
____ 15. Rapid turnaround times for selected samples and the ability to accommodate large-volume
batch testing contribute to overall operational efficiency.
____ 16. Vendors of automation for the transfusion service require prior approval from the Food
and Drug Administration (FDA).
____ 17. Vendor assessment, base technology, and instrument assessment are important
considerations in the selection of an automated system.

CHAPTER11

1. A patient experiences chills and fever, nausea, flushing, and lower back pain after infusion of 150
mL of blood. What action should be taken to rule out an acute hemolytic transfusion reaction?
a. perform a DAT and visually compare pretransfusion and posttransfusion serum samples
b. measure serum haptoglobin on prereaction and postreaction samples
c. repeat crossmatches on prereaction and postreaction samples
d. perform Gram stain and culture of the unit
2. Select the type of transfusion reaction that presents with dyspnea, severe headache, and
peripheral edema occurring soon after transfusion.
a. hemolytic
b. TRALI
c. TACO
d. anaphylactic
3. What is a common cause of a febrile nonhemolytic transfusion reaction?
a. recipient is allergic to the donor’s plasma proteins
b. donor unit is cold
c. donor unit has a positive DAT
d. recipient has antibodies to the donor’s HLA antigens
4. What plasma protein functions to bind hemoglobin after intravascular hemolysis?
a. albumin
b. haptoglobin
c. transferrin
d. C-reactive protein
5. Which of the following adverse complications of transfusion is prevented by the irradiation of
blood components?
a. TRALI
b. hyperkalemia
c. febrile
d. TA-GVHD
6. Which of the following characteristics is associated with a delayed serologic transfusion reaction?
a. hives and wheals
b. hemosiderosis
c. positive antibody screen in posttransfusion sample
d. ABO incompatibility between donor unit and recipient
7. What blood group system antibodies are more commonly associated with delayed hemolytic
transfusion reactions?
a. Rh
b. ABO
c. MNS
d. Lewis
8. A patient has experienced two febrile nonhemolytic reactions after RBC transfusion. What is the
preferred blood component if future transfusions are necessary?
a. leukocyte-reduced RBCs
b. irradiated RBCs
c. cytomegalovirus-negative RBCs
522
d. group O, D-negative RBCs
9. Which of the following patient histories might suggest future transfusions with saline-washed
RBCs?
a. history of multiple red cell alloantibodies
b. history of congestive heart failure
c. IgA-negative recipient with anti-IgA antibodies
d. history of transfusion-associated sepsis
10. What is the cause of transfusion-induced hemosiderosis?
a. excess citrate
b. HPA-1a antigen
c. iron overload
d. circulatory overload
11. What laboratory test is useful to detect clerical errors of sample identification in an acute
transfusion reaction investigation?
a. ABO typing
b. antibody screen
c. crossmatch
d. DAT
12. What microorganism grows well at 4° C and may result in a transfusion-transmitted sepsis?
a. Staphylococcus aureus
b. Yersinia enterocolitica
c. Staphylococcus epidermidis
d. Bacillus cereus
13. What is the expected therapeutic effect in the recipient’s hematocrit after the transfusion of 1 unit
of RBCs?
a. increase of 0.5%
b. increase of 1%
c. increase of 2%
d. increase of 3%
14. What is the usual cause of an anaphylactic reaction to transfusion?
a. anti-IgA in an IgA-deficient recipient
b. anti-IgG in an IgA-deficient recipient
c. IgA deficiency
d. IgG deficiency
15. Which of the following events is associated with a precipitous decrease in a recipient’s platelet
count after a transfusion?
a. circulatory overload
b. posttransfusion purpura
c. citrate toxicity
d. factor VIII deficiency
16. When evaluating a possible delayed hemolytic reaction, what is the best sample to use for
bilirubin determination?
a. 6 hours posttransfusion
b. 12 hours posttransfusion
c. 24 hours posttransfusion
d. 48 hours posttransfusion
17. In a delayed serologic or hemolytic transfusion reaction, what is the typical result of the DAT?
523
a. negative
b. weak positive, mixed field
c. positive with C3 only
d. negative if serum antibody screen is negative
18. Which type of transfusion reaction will be prevented by the use of plasma from only male
donors for transfusion?
a. febrile
b. TRALI
c. allergic
d. TACO
19. Premedication with diphenhydramine (Benadryl) is a common procedure when administrating
platelets to patients undergoing frequent transfusions. Which type of transfusion reaction does
this medication prevent?
a. allergic
b. TRALI
c. febrile nonhemolytic reactions
d. TACO
20. What is the usual cause of posttransfusion purpura after transfusion of platelets?
a. HLA antibodies in the donor unit
b. HLA antibodies made by the recipient
c. anti–HPA-1a made by the recipient
d. febrile reactions secondary to cytokines in the unit
21. Five days after a transfusion, a patient returned to his physician for postsurgical blood tests. It
was noted that the hemoglobin value decreased from 11 mg/dL to 9 mg/dL during that time. The
patient had not experienced any symptoms. To rule out a delayed hemolytic transfusion reaction,
what test should be performed?
a. DAT on current sample, elution if positive
b. antibody screen on the current sample
c. blood smear to check for spherocytes
d. all of the above
22. According to AABB Standards, what type of transfusion reaction does not have to be reported to
the blood bank?
a. febrile
b. TACO
c. TRALI
d. allergic

CHAPTER12
1. What item listed is not an objective for performing an exchange transfusion?
a. decrease the level of maternal antibody
b. reduce the level of indirect bilirubin
c. provide platelets to prevent disseminated intravascular coagulation
d. provide compatible RBCs to correct anemia
2. What is the greatest danger to a fetus affected by HDFN before delivery?
a. kernicterus
b. anemia
c. hyperbilirubinemia
d. hypertension
3. How many milliliters of whole blood from an FMH is covered with a 300-μg dose of RhIG?
a. 10 mL
b. 15 mL
c. 30 mL
d. 50 mL
4. What is the time frame for RhIG administration after delivery?
a. 6 hours
b. 48 hours
c. 72 hours
d. 96 hours
5. What is the name of an often-fatal condition characterized by general edema that results from
anemia?
a. kernicterus
b. disseminated intravascular coagulation (DIC)
c. erythroblastosis fetalis
d. hydrops fetalis
6. What is the mechanism for HDFN occurrence?
a. maternal antigens react with fetal antibodies
b. fetal antibodies react with maternal antibodies
c. maternal antibodies react with fetal antigens
d. fetal antigens react with maternal antigens
7. What is the greatest danger to the newborn affected by HDFN postpartum?
a. kernicterus
b. anemia
c. conjugated bilirubin
d. low L/S ratio
8. Which of the following women should receive postpartum RhIG?
Mother’s ABO/D phenotype Mother’s antibody screen Newborn’s ABO/D phenotype
a. A, D-negative- Negative- O, D-positive
b. O, D-negative -Negative- A, D-negative
c. A, D-positive- Negative- B, D-negative
d. B, D-negative -Immune anti-D- B, D-positive
9. Which of the following antibodies carries no risk of HDFN?
a. anti-Lea
b. anti-C
560
c. anti-K
d. anti-S
10. Which of the following is not a characteristic of ABO HDFN?
a. may occur in first pregnancy
b. usually treated with phototherapy
c. strongly positive DAT
d. most frequent in babies born to group O mothers
11. Which of the following requirements is important when selecting blood for exchange transfusion
to avoid high levels of potassium?
a. irradiated blood
b. CMV-negative blood
c. leukocyte-reduced blood
d. blood less than 7 days old
12. A mother is group A, D-negative with anti-D in her serum. Which of the following units should
be selected for an intrauterine transfusion?
a. group O, D-negative
b. group O, D-positive
c. group A, D-negative
d. group A, D-positive
13. Select the statement that is true regarding the rosette test.
a. performed on a cord blood sample
b. used to screen for FMH
c. a quantitative test used to calculate the volume of FMH
d. an acid elution used to estimate the volume of FMH
14. Which of the following tests is not necessary when testing a cord blood sample?
a. ABO
b. D
c. DAT
d. antibody screen
15. What is the principle of the Liley method for predicting the severity of HDFN?
a. resistance of fetal hemoglobin to acid elution
b. ratio of lecithin to sphingomyelin
c. change of optical density of amniotic fluid measured at 450 nm
d. direct bilirubin evaluation of a cord blood sample
16. A titer was performed on a prenatal sample from a D-negative woman with anti-D. The sample
was tested 4 weeks later in parallel with a current sample. The following results were obtained:

How would the titer results be interpreted?


a. an intrauterine transfusion is necessary
b. early induction of labor should be considered
c. color Doppler ultrasonography should be considered
d. RhIG should be administered
561
17. A group A, D-negative mother demonstrating anti-D antibodies delivered a group O, D-negative
baby with a positive DAT (2+), elevated bilirubin (18 mg/dL), and low hemoglobin (8 g/dL).
Which is the most probable explanation for these test results?
a. ABO hemolytic disease of the newborn
b. hemolytic disease of the newborn with a false-negative D typing due to blocking antibodies
c. large fetomaternal hemorrhage causing discrepancy in the blood type
d. prenatal RhIG administration
18. How would you interpret the appearance of spherocytes in a baby’s blood smear after delivery?
a. ABO HDFN
b. HDFN caused by anti-D
c. HDFN caused by other IgG antibodies
d. normal physiologic anemia detected in newborns
19. What is the purpose for the irradiation of blood selected for an exchange transfusion?
a. prevent formation of HLA antibodies
b. prevent sepsis from bacterial contamination
c. prevent graft-versus-host disease
d. prevent transmission of viruses
20. What bleed is detected by the rosette test used for screening for a fetomaternal hemorrhage?
a. 5 mL
b. 10 mL
c. 20 mL
d. 30 mL
21. A Kleihauer–Betke stain performed on a postpartum blood sample demonstrated 10 fetal cells in
a field of 2000. What is the estimated blood volume of the fetomaternal hemorrhage expressed as
whole blood?
a. 25 mL
b. 30 mL
c. 45 mL
d. 100 mL
22. A rosette test performed on a D-negative mother who delivered a D-positive baby demonstrated
two rosettes per three fields observed. What is the correct course of action?
a. submit the sample for a Kleihauer–Betke test
b. recommend two vials of RhIG
c. suggest that RhIG is not necessary because records indicate that the mother received prenatal
RhIG
d. recommend one vial of RhIG because it is below the cutoff for the fetal screen
23. What is the principle of the Kleihauer–Betke test?
a. fetal hemoglobin resists acid elution
b. adult hemoglobin resists acid elution
c. fetal red cells lose hemoglobin under alkaline conditions
d. adult red cells accept dye under alkaline conditions
24. Results of a Kleihauer–Betke test determine there was a fetomaternal hemorrhage of 35 mL of
whole blood during delivery. What is the correct dosage of RhIG?
a. one vial
b. two vials
c. three vials
d. four vials
562
25. A weakly reactive anti-D test was identified in a postpartum sample from a D-negative woman
who gave birth to a D-positive baby. What is the most likely cause?
a. immune anti-D produced from exposure during the first pregnancy
b. immune anti-D produced from exposure during the current pregnancy
c. antenatal RhIG given
d. error in antibody identification or D typing

CHAPTER13
For questions 1 through 12, determine the best course of action based on the information for
potential whole blood allogeneic donors. Indicate whether you would:
A = Accept
TD = Temporarily defer (indicate when donor is eligible)
PD = Permanently or indefinitely defer
1. A 28-year-old woman; 112 lb; hemoglobin, 12.5 g/dL; miscarried 2 weeks ago
2. A 56-year-old man; 168 lb; hematocrit, 44%; took aspirin 4 hours ago for arthritis pain
3. A 35-year-old woman; 115 lb; temperature, 37° C; pulse, 75
4. A 17-year-old female high-school student; taking isotretinoin (Accutane) for acne
5. A 75-year-old male donor center volunteer; first-time blood donor; contracted hepatitis 20 years
ago after surgery
6. A 22-year-old male; received tattoo while in the service 4 months ago, just before he returned
from Iraq
7. A 65-year-old female; has instructions from physician to donate for upcoming surgery; had
syphilis and was treated 40 years ago; hematocrit, 37%; temperature, 99° F
8. A 38-year-old male; received recombinant hepatitis B vaccine as a new employee 3 months ago
9. A 19-year-old male first-time donor; received human growth hormone 12 years ago
10. A 24-year-old female with a history of a positive test for hepatitis C from another blood center
11. A 52-year-old businessman who was a resident in England for 1 year in 1993
12. A 130-lb, 5-feet-1-inch female; hematocrit, 40%; would like to donate two RBC units by apheresis
13. Which of the following is a cause for temporary deferral of a whole blood donor?
a. intranasal influenza vaccine
b. antibiotics taken for acne
c. oral polio vaccine 4 weeks ago
d. rubella vaccine 2 weeks ago
14. A donor with a physician’s request to donate for planned surgery in 3 weeks has a hemoglobin
value of 10 g/dL. What is her eligibility status?
a. permitted to donate as an autologous donor
b. deferred because of low hemoglobin
c. permitted to donate with the approval of the blood bank’s medical director
d. permitted to donate a smaller unit of blood
15. What is the maximum number of donations for plateletpheresis donors in the period of a year?
a. 6
b. 12
c. 24
d. 48
True or False
____ 16. Viral marker tests are not required on autologous blood intended for use within the
collection facility.
____ 17. Autologous units may be given to other patients if they are not used for the patient who
donated the units.
____ 18. A unit donated therapeutically from a person with hereditary hemochromatosis cannot be
used for transfusion purposes.
____ 19. Donor centers are authorized to release positive test results to their state health department
605
if the donor signs a consent form.
____ 20. According to the FDA, prospective donors with a history of cancer are not permitted to
donate blood.

CHAPTER14
1. Which disease has the highest potential for transmission through a transfusion?
a. AIDS
b. syphilis
c. CMV
d. hepatitis
2. Syphilis tests on donors are usually performed by which method or methods?
a. RPR
b. Venereal Disease Research Laboratory
c. hemagglutination
d. both a and c
3. What characteristic is associated with HTLV-I/II?
a. an oncornavirus
b. found in patients with tropical spastic paraparesis
c. associated with adult T-cell leukemia
d. all of the above
4. What marker demonstrates a previous exposure to hepatitis B that remains in convalescence?
a. anti-HCV
b. anti-HBc
c. anti-HAV
d. HBsAg
5. Which of the following is the confirmatory test for a positive anti-HIV screen?
a. HIV RNA
b. RIBA
c. PCR
d. Southern blot
6. Which of the following conditions requires a thorough donor history because it is not a routinely
tested disease?
a. syphilis
b. Creutzfeldt-Jakob disease
c. hepatitis C
d. HTLV-I
7. Why is HAV transmission through a blood transfusion unusual?
a. transmitted enterically
b. an acute hepatitis
c. not infective after 2 weeks
d. all of the above
8. What is the donation status of a donor who is positive for HBsAg?
a. temporarily deferred
b. permanently deferred
c. deferred if the antibody to HBc is also present
d. deferred if ALT is elevated
9. Which of the following was a surrogate test for hepatitis and is no longer required?
a. ALT
b. CMV
640
c. anti-HBc
d. HBsAg
10. In a __________, a lower absorbance value indicates the detection of the viral marker.
a. RIBA
b. sandwich ELISA
c. competitive ELISA
d. Western blot

CHAPTER15

For questions 1 through 7, match the clinical condition to the component that would have the
best therapeutic value. Components may be used more than once.

For questions 8 through 15, match the correct expiration times on the right with the
appropriate blood component on the left. Expiration times can be used more thanonce.

16. What is the minimum amount of fibrinogen required in 1 unit of cryoprecipitated AHF?
a. 150 mg
b. 250 mg
c. 80 IU
d. 1000 mg
17. What fluid is administered with the transfusion of blood components?
a. Ringer’s lactate
b. 5% dextrose
c. 0.9% saline
d. All of the above
18. Eight units of platelets were pooled in an open system without the use of a sterile connecting
device. What is the new time of expiration of the pooled product?
690
a. 2 hours
b. 4 hours
c. 6 hours
d. 24 hours
19. What is the correct order of centrifugation in the preparation of platelets from a unit of whole
blood?
a. hard spin followed by hard spin
b. light spin followed by light spin
c. hard spin followed by light spin
d. light spin followed by hard spin
20. What are minimum temperature and maximum storage time for a unit of frozen RBCs?
a. −65° C for 5 years
b. −85° C for 10 years
c. −65° C for 10 years
d. −80° C for 10 years
21. How many platelets must be obtained in a plateletpheresis in order to meet acceptance criteria?
a. 5.5 × 1010
b. 3.3 × 1011
c. 5.0 × 1011
d. 3.0 × 1011
22. How are sterile connecting devices used?
a. connecting a leukocyte removal filter to RBCs
b. preparing small aliquot transfusions for infants
c. connecting platelets for pooling
d. all of the above
23. What are the temperature limits for shipping RBCs?
a. 1° C to 6° C
b. 1° C to 10° C
c. 2° C to 8° C
d. 20° C to 24° C
24. The pH and platelet count of four bags of platelets were tested at the end of the allowable
storage period. Which of the following results is an acceptable product?
a. 5.5 × 109 and pH of 6.5
b. 6.0 × 1010 and pH of 7.0
c. 4.2 × 1011 and pH of 5.9
d. 3.0 × 1010 and pH of 6.2
25. Although ABO compatibility is preferred, ABO incompatibility is acceptable for which of the
following components?
a. PF24
b. cryoprecipitated AHF
c. apheresis granulocytes
d. apheresis platelets

CHAPTER16

1. What disease has been treated effectively using plasmapheresis?


a. TTP
b. hemolytic disease of the newborn
c. sickle cell disease
d. renal disease
2. Why is it difficult to find compatible blood for patients with autoimmune disease?
a. potential of underlying alloantibodies
b. positive DAT
c. reactive eluate
d. hemolysis in the serum
3. What infant age period does not require a crossmatch procedure before transfusion?
a. the first 4 months
b. the first 6 months
c. the first year
d. an indefinite period if a parent’s blood is used
4. Select the example of a crystalloid solution used to treat hypovolemia.
a. PPF
b. albumin
c. Ringer’s lactate
d. HES solution
5. How are hemophilia A patients treated for bleeding?
a. cryoprecipitated AHF
b. FFP
c. RBCs
d. factor VIII
6. Select common complications of chemotherapy from the list.
a. bleeding
b. infection
c. anemia
d. all of the above
7. Where is erythropoietin produced in the adult for stimulation of red cell proliferation?
a. bone marrow
b. liver
c. kidneys
d. spleen
8. What factors are compensated for in the transfusion of RBCs to a neonate?
a. iatrogenic blood loss
b. hemoglobin F
c. insufficient erythropoiesis
d. all of the above
9. ABO-compatible organ transplants are not critical in which of the following transplants?
a. kidneys
b. liver
719
c. heart
d. bone marrow
10. What colony-stimulating factor can reduce infection while undergoing chemotherapy?
a. erythrocytes
b. megakaryocytes
c. granulocytes
d. lymphocytes

ANSWERS

Chapter 1

1. c

2. d

3. b

4. a

5. c

6. d

7. c

8. a

9. b

10. a

11. c
12. b

13. c

14. b

15. a

16. d

17. d

18. a

19. d

20. b

723

Chapter 2

1. d

2. a

3. b

4. c

5. d

6. a

7. b

8. a

9. c

10. a

11. a

12. e

13. b

14. c

15. c

16. a

17. b

18. b

19. c

20. c
21. d

22. c

23. b

24. c

25. c

724

Chapter 3

1. a

2. b

3. a

4. c

5. d

6. c

7. b

8. c

9. a

10. b

11. d

12. a

13. b

14. a

15. c

725

Chapter 4

1. d

2. b

3. d

4. b

5. a

6. a

7. c
8. a

9. c

10. a

11. d

12. c

13. c

14. b

15. c

16. c

17. b

18. c

19. b

20. d

726

Chapter 5

1. c

2. d

3. a

4. d

5. a

6. d

7. c

8. c

9. c

10. b

11. c

12. d

13. a

14. d

15. c

16. a
17. b

18. d

19. c

20. b

727

Chapter 6

1. d

2. a

3. d

4. a

5. a

6. b

7. b

8. d

9. d

10. a

11. a

12. b

13. a

14. c

15. a

16. a

17. b

18. d

19. c

20. b

728

Chapter 7

1. c

2. c

3. a
4. a

5. d, e, g

6. d

7. d

8. c

9. d

10. c

11. a

12. d

13. a

14. c

15. d

16. a

17. c

18. c

19. b

20. d

21. c

22. b

23. b

24. b

25. d

26. a

27. d

28. b

29. d

30. c

729

Chapter 8

1. a

2. c
3. d

4. d

5. a

6. c

7. c

8. c

9. b

10. d

11. a

12. c

13. c

14. d

15. c

730

Chapter 9

1. b

2. c

3. d

4. a

5. b

6. c

7. c

8. b

9. d

10. a

11. c

12. d

13. c

14. c

15. F

16. F
17. F

18. F

19. F

20. F

21. T

22. F

23. F

24. T

25. T

731

Chapter 10

1. a

2. c

3. c

4. d

5. d

6. c

7. b

8. a

9. d

10. a

11. T

12. T

13. T

14. F

15. T

16. T

17. T

732

Chapter 11

1. a
2. c

3. d

4. b

5. d

6. c

7. a

8. a

9. c

10. c

11. a

12. b

13. d

14. a

15. b

16. a

17. b

18. b

19. a

20. c

21. d

22. d

733

Chapter 12

1. c

2. b

3. c

4. c

5. d

6. c

7. a

8. a
9. a

10. c

11. d

12. a

13. b

14. d

15. c

16. c

17. b

18. a

19. a

20. b

21. a

22. a

23. a

24. b

25. c

734

Chapter 13

1. TD

2. A

3. A

4. TD

5. PD

6. TD

7. A

8. A

9. PD

10. PD

11. PD

12. PD
13. d

14. b

15. c

16. T

17. F

18. F

19. T

20. F

735

Chapter 14

1. d

2. d

3. d

4. b

5. a

6. b

7. d

8. b

9. a

10. c

736

Chapter 15

1. d

2. b

3. e

4. f

5. a

6. g

7. c

8. e

9. d
10. b

11. a

12. c

13. f

14. d

15. g

16. a

17. c

18. b

19. d

20. c

21. d

22. d

23. b

24. b

25. b

737

Chapter 16

1. a

2. a

3. a

4. c

5. d

6. d

7. c

8. d

9. d

10. c

738

2019. Modern blood banking & transfusion medicine


Review Questions1
1. What is the maximum volume of blood that can be
collected from a 110-lb donor, including samples for
processing?
a. 450 mL
b. 500 mL
c. 525 mL
d. 550 mL
2. How often can a blood donor donate whole blood?
a. Every 24 hours
b. Once a month
c. Every 8 weeks
d. Twice a year
3. When RBCs are stored, there is a “shift to the left.” This
means:
a. Hemoglobin-oxygen affinity increases, owing to an
increase in 2,3-DPG.
b. Hemoglobin-oxygen affinity increases, owing to a
decrease in 2,3-DPG.
c. Hemoglobin-oxygen affinity decreases, owing to a
decrease in 2,3-DPG.
d. Hemoglobin-oxygen affinity decreases, owing to an
increase in 2,3-DPG.
4. The majority of platelets transfused in the United States
today are:
a. Whole blood–derived platelets prepared by the
platelet-rich plasma method.
b. Whole blood–derived platelets prepared by the buffy
coat method.
c. Apheresis platelets.
d. Prestorage-pooled platelets.
5. Which of the following anticoagulant preservatives
provides a storage time of 35 days at 1°C to 6°C for units
of whole blood and prepared RBCs if an additive solution
is not added?
a. ACD-A
b. CP2D
c. CPD
d. CPDA-1
6. What are the current storage time and storage temperature
for platelet concentrates and apheresis platelet
components?
a. 5 days at 1°C to 6°C
b. 5 days at 24°C to 27°C
c. 5 days at 20°C to 24°C
d. 7 days at 22°C to 24°C
Chapter 1 Red Blood Cell and Platelet Preservation: Historical Perspectives and Current Trends 21
7. RBCs can be frozen for:
a. 12 months.
b. 1 year.
c. 5 years.
d. 10 years.
8. Whole blood and RBC units are stored at what
temperature?
a. 1°C to 6°C
b. 20°C to 24°C
c. 37°C
d. 24°C to 27°C
9. Additive solutions are approved for storage of red blood
cells for how many days?
a. 21
b. 42
c. 35
d. 7
10. One criterion used by the FDA for approval of new
preservation solutions and storage containers is an average
24-hour post-transfusion RBC survival of more than:
a. 50%.
b. 60%.
c. 65%.
d. 75%.
11. What is the lowest allowable pH for a platelet component
at outdate?
a. 6
b. 5.9
c. 6.8
d. 6.2
12. Which of the following occurs during storage of red
blood cells?
a. pH decreases
b. 2,3-DPG increases
c. ATP increases
d. plasma K+ decreases
13. Which of the following is approved for bacterial detection
specific to extending the expiration of apheresed
platelets to 7 days?
a. BacT/ALERT
b. eBDS
c. Gram stain
d. Pan Genera Detection (PGD) test
14. Which of the following is the most common cause of
bacterial contamination of platelet products?
a. Entry of skin plugs into the collection bag
b. Environmental contamination during processing
c. T in the donor
d. Incorrect storage temperature
15. The INTERCEPT pathogen reduction system uses which
of the following methods?
a. Riboflavin and UV light
b. Amotosalen and UV light
c. Solvent/detergent treatment
d. Irradiation

Review Questions2
1. Which statement best describes the process of mitosis?
a. Cell division by which only one-half of the daughter
cells produced are identical to the parent cell
b. Cell division of germ cells by which two successive
divisions of the nucleus produce cells that contain half
the number of chromosomes of somatic cells
c. Cell division that produces four daughter cells having
the same number of chromosomes as the parent
d. Cell division that produces two daughter cells with the
same number of chromosomes as the parent cell
2. When a recessive trait is expressed:
a. One gene carrying the trait was present.
b. Two genes carrying the trait were present.
c. No gene carrying the trait was present.
d. The gene is hemizygous.
3. In a pedigree analysis, which statement about the symbols
used is true?
a. Deceased family members have a line crossed through
the symbol.
b. A consanguineous mating is indicated by a single line
between a male and female.
c. The propositus is always found in the last generation
on the pedigree.
d. A stillbirth is indicated by a triangle.
Chapter 2 Basic Genetics 43
4. Which of the following nitrogenous bases make up DNA?
a. Adenine, leucine, guanine, thymine
b. Alanine, cytosine, guanine, thymine
c. Adenine, lysine, uracil, guanine
d. Adenine, cytosine, guanine, thymine
5. Mutations can occur following DNA replication that
escapes the proofreading and repair systems. Which statement
about DNA mutations is true?
a. All mutations result in a phenotypic change.
b. A frameshift mutation at the beginning of the coding
sequence is most likely to result in a phenotypic
change.
c. A transversion always encodes for a stop codon.
d. A missense point mutation never encodes for a stop
codon.
6. Which phenotype would be expected from the mating of
a Jk(a+b–) female and a Jk(a–b+) male?
a. Jk(a+b–)
b. Jk(a+b+)
c. Jk(a–b+)
d. All of the above
7. Which statement describes an intron?
a. The part of a gene that contains nonsense mutations
b. The coding region of a gene
c. The noncoding region of a gene
d. The resting stage between cell divisions
8. Which statement about isolation of nucleic acids is true?
a. All isolation methods involve the use of organic
solvents.
b. High protein concentration increases the DNA yield.
c. mRNA can be effectively isolated with the use of
poly-A–coated beads.
d. Silica particles bind DNA under high salt con -
centrations.
9. The purpose of transcription is to:
a. Produce a protein.
b. Read the mRNA by the ribosome.
c. Synthesize RNA using DNA as a template.
d. Replicate DNA.
10. When a male possesses a phenotypic trait that he passes
to all his daughters and none of his sons, the trait is said
to be:
a. X-linked dominant.
b. X-linked recessive.
c. Autosomal dominant.
d. Autosomal recessive.
11. When a female possesses a phenotypic trait that she
passes to all of her sons and none of her daughters, the
trait is said to be:
a. X-linked dominant.
b. X-linked recessive.
c. Autosomal dominant.
d. Autosomal recessive.
12. Which statement correctly describes DNA replication in
eukaryotes?
a. Semiconservative replication from RNA; requires
DNA polymerase
b. Bidirectional replication; requires RNA primer
c. Conservative replication from DNA; requires RNA
polymerase
d. Occurs at the replication fork; both strands replicated
in the same direction
13. How is tRNA different from other types of RNA?
a. It has a 3′ poly-T tail.
b. The immature form contains introns.
c. It has a 3′ methylated cap.
d. It recognizes amino acids and nucleic acids.
14. Which statement about translation of proteins is false?
a. It occurs on the ribosomes in the cytoplasm of
the cell.
b. Post-translation processing can include glycosylation.
c. mRNA delivers the amino acids to the growing
peptide chain during elongation.
d. The codon UAA would terminate translation.
15. The purpose of meiosis is to:
a. Generate two identical daughter cells after division.
b. Generate four identical daughter cells after division
with the same number of chromosomes as the parent
cells.
c. Generate gametes with a diploid number of
chromosomes.
d. Generate daughter cells that contain half the number
of chromosomes of somatic cells that contain new
DNA sequences.
16. The pattern of inheritance most commonly expressed by
blood group genes is:
a. X-linked recessive.
b. Autosomal recessive.
c. Autosomal codominant.
d. X-linked codominant.

Review Questions3
1. Which of the following is not involved in the acquired
(adaptive) immune response?
a. Phagocytosis
b. Production of antibody or complement
c. Induction of immunologic memory
d. Accelerated immune response upon subsequent
exposure to antigen
2. Which cells are involved in the production of antibodies?
a. Dendritic cells
b. T lymphocytes
c. B lymphocytes
d. Macrophages
3. Which of the following cells is involved in antigen recognition
following phagocytosis?
a. B lymphocytes
b. T lymphocytes
c. Macrophages
d. Granulocytes
4. The role of the macrophage during an antibody response
is to:
a. Make antibody.
b. Lyse virus-infected target cells.
c. Activate cytotoxic T cells.
d. Process antigen and present it.
5. Which of the following immunoglobulins is produced in
the primary immune response?
a. IgA
b. IgE
c. IgG
d. IgM
6. Which of the following immunoglobulins is produced in
the secondary immune response?
a. IgA
b. IgE
c. IgG
d. IgM
7. Which of the following MHC classes encodes complement
components?
a. Class I
b. Class II
c. Class III
d. Class IV
8. Which of the following immunoglobulins is most efficient
at binding complement?
a. IgA
b. IgE
c. IgG
d. IgM
9. Which portion of the immunoglobulin molecules contains
complement binding sites?
a. Heavy chain variable region
b. Light chain variable region
c. Heavy chain constant region
d. Light chain constant region
10. Which complement pathway is activated by the formation
of antigen-antibody complexes?
a. Classical
b. Alternative
c. Lectin
d. Retro
11. Which of the following is known as the “recognition
unit” in the classical complement pathway?
a. C1q
b. C3a
c. C4
d. C5
12. Which of the following is known as the “membrane
attack complex” in the classical complement pathway?
a. C1
b. C3
c. C4, C2, C3
d. C5b, C6, C7, C8, C9
13. Which of the following immunoglobulin classes is
capable of crossing the placenta and causing hemolytic
disease of the newborn?
a. IgA
b. IgE
c. IgG
d. IgM
14. Which of the following refers to the effect of an excess
amount of antigen present in a test system?
a. Postzone
b. Prozone
c. Zone of equivalence
d. Endzone
15. Which of the following refers to the presence of an
excess amount of antibody present in a test system?
a. Postzone
b. Prozone
c. Zone of equivalence
d. Endzone
16. Which one of the following properties of antibodies is
NOT dependent on the structure of the heavy chain
constant region?
a. Ability to cross the placenta
b. Isotype (class)
c. Ability to fix complement
d. Affinity for antigen
17. Molecules that promote the update of bacteria for
phagocytosis are:
a. Opsonins.
b. Cytokines.
c. Haptens.
d. Isotypes.
18. Select the term that describes the unique confirmation
of the antigen that allows recognition by a correspond -
ing antibody.
a. Immunogen
b. Epitope
c. Avidity
d. Clone
19. Which of the following terms refers to the net negative
charge surrounding red blood cells?
a. Dielectric constant
b. Van der Waals forces
c. Hydrogen bonding
d. Zeta potential

Review Questions4
1. The central dogma of molecular biology states that:
a. DNA is the genetic material.
b. RNA is the genetic material.
c. DNA is translated to mRNA.
d. Proteins are transcribed from mRNA.
2. Recombinant-DNA technology is possible because:
a. Restriction endonucleases cut RNA.
b. Restriction endonucleases cut proteins.
c. The genetic code is universal.
d. Bacteria are difficult to culture.
3. Agarose gel electrophoresis is a technique used for:
a. DNA synthesis.
b. RNA synthesis.
c. Separation of DNA molecules by size.
d. Oligonucleotide synthesis.
4. Restriction fragment length polymorphism (RFLP) is
based on the use of the enzymes:
a. Reverse transcriptases.
b. Bacterial endonucleases.
c. DNA polymerases.
d. RNA polymerases.
5. The polymerase chain reaction (PCR):
a. Is carried out in vivo.
b. Is used for peptide synthesis.
c. Requires RNA polymerase.
d. Is used for the amplification of DNA.
6. Plasmids are:
a. Vectors used for molecular cloning.
b. Antibiotics.
c. Enzymes.
d. Part of chromosomes.
7. Some model organisms:
a. Simplify the study of human disease.
b. Are used to produce recombinant proteins.
c. Are prokaryotes and some are eukaryotes.
d. All of the above
8. DNA sequencing:
a. Is more difficult than peptide sequencing.
b. Requires the use of RNA polymerase.
c. Can never be automated.
d. Is an enzymatic in vitro reaction.
9. RFLP and SSP are techniques used for:
a. Protein isolation.
b. RNA isolation.
c. DNA typing.
d. Protein typing.
10. Recombinant DNA techniques:
a. Are not used in a clinical setting.
b. Are useful research tools.
c. Are not used in blood banking.
d. Are useful only for research.
11. Transcription-mediated amplification:
a. Requires thermostable DNA polymerase.
b. Is an isothermal procedure.
c. Is an obsolete method currently replaced by SSOP.
d. Utilizes probes labeled with fluorescent tags.
12. Preseroconversion window:
a. Is the time when donors can be infected but do not
yet test positive by serologic methods.
b. May be narrowed by using molecular methods.
c. Refers mainly to viral pathogens.
d. All of the above
13. Red blood cell molecular antigen typing is useful in all
listed situations except:
a. In screening RBC inventory for antigen-negative
units.
b. When reagent antibodies are weak or unavailable.
c. In quantitative gene expression analysis.
d. When resolving ABO discrepancies.
Review Questions5
1. A description of the antiglobulin test is:
a. IgG and C3d are required for RBC sensitization.
b. Human globulin is completely eluted from RBCs
during saline washings.
c. Human globulin is injected into an animal.
d. AHG reacts with human globulin molecules bound to
RBCs.
2. Polyspecific AHG reagent contains:
a. Anti-IgG and anti-IgA.
b. Anti-IgG and anti-IgM.
c. Anti-IgG and anti-C3d.
d. Anti-IgA and Anti-C3d.
3. Monoclonal anti-C3d is:
a. Derived from one clone of plasma cells.
b. Derived from multiple clones of plasma cells.
c. Derived from immunization of rabbits.
d. Reactive with C3b and C3d.

4. Which of the following is a clinically significant


antibody whose detection has been reported in some
instances to be dependent on anticomplement activity
in polyspecific AHG?
a. Anti-Jka
b. Anti-Lea
c. Anti-P1
d. Anti-H
5. After the addition of IgG-coated RBCs (check cells)
to a negative AHG reaction during an antibody
screen, a negative result is observed. Which of the
following is a correct interpretation based on these
findings?
a. The antibody screen is negative.
b. The antibody screen cannot be interpreted.
c. The saline washings were adequate.
d. AHG reagent was added.
6. RBCs must be washed in saline at least three times
before the addition of AHG reagent to:
a. Wash away any hemolyzed cells.
b. Remove traces of free serum globulins.
c. Neutralize any excess AHG reagent.
d. Increase the antibody binding to antigen.
7. An in vivo phenomenon associated with a positive
DAT is:
a. Passive anti-D detected in the maternal sample.
b. Positive antibody screen tested by LISS.
c. Identification of alloantibody specificity using a
panel of reagent RBCs.
d. Maternal antibody coating fetal RBCs.
8. False-positive DAT results are most often associated
with:
a. Use of refrigerated, clotted blood samples in which
complement components coat RBCs in vitro.
b. A recipient of a recent transfusion manifesting an
immune response to recently transfused RBCs.
c. Presence of antispecies antibodies from administration
of immune globulin (IVIG).
d. A positive autocontrol caused by polyagglutination.
9. Polyethylene glycol (PEG) enhances antigenantibody
reactions by:
a. Decreasing zeta potential.
b. Concentrating antibody by removing water.
c. Increasing antibody affinity for antigen.
d. Increasing antibody specificity for antigen.
10. Solid-phase antibody screening is based on:
a. Adherence.
b. Agglutination.
c. Hemolysis.
d. Precipitation.

11. A positive DAT may be found in which of the following


situations?
a. A weak D-positive patient
b. A patient with anti-M
c. HDFN
d. An incompatible crossmatch
12. What do Coombs’ check cells consist of?
a. Type A-positive cells coated with anti-IgG
b. Type A-negative cells coated with anti-IgG
c. Type O-positive cells coated with anti-D
d. Type B-negative cells coated with anti-D
13. Which of the following IAT methods requires the use of
check cells?
a. Manual tube method with albumin
b. Gel
c. Automated solid-phase analyzer
d. Enzyme-linked
14. Which uncontrollable factor can affect AHG testing?
a. Temperature
b. Antibody affinity
c. Gravitational force in the centrifuge
d. Incubation time
15. Which would be the most efficient method for a laboratory
staffed by medical laboratory technicians?
a. LISS
b. Polybrene
c. Solid-phase or gel
d. Enzyme-linked
16. A 27-year-old group O mother has just given birth to a
group A baby. Since the mother has IgG anti-A, anti-B
and anti-A, B in her plasma, which of the following
methods and tests would be most effective at detecting
the anti-A on the baby’s RBCs?
a. DAT using common tube technique
b. DAT using gel
c. IAT using common tube technique
d. IAT using gel

Review Questions6
1. An ABO type on a patient gives the following reactions:
Patient Cells With Patient Serum With
Anti-A Anti-B A1 cells B cells
4+ 4+ Neg Neg
What is the patient’s blood type?
a. O
b. A
c. B
d. AB
2. The major immunoglobulin class(es) of anti-B in a group
A individual is (are):
a. IgM.
b. IgG.
c. IgM and IgG.
d. IgM and IgA.
3. What are the possible ABO phenotypes of the offspring
from the mating of a group A to a group B individual?
a. O, A, B
b. A, B
c. A, B, AB
d. O, A, B, AB
4. The immunodominant sugar responsible for blood group
A specificity is:
a. L-fucose.
b. N-acetyl-D-galactosamine.
c. D-galactose.
d. Uridine diphosphate-N-acetyl-D-galactose.
5. What ABH substance(s) would be found in the saliva of
a group B secretor?
a. H
b. H and A
c. H and B
d. H, A, and B
6. An ABO type on a patient gives the following reactions:
Patient Cells With Patient Serum With
Anti-A Anti-B Anti-A1 A1 cells B cells
4+ 4+ Neg 2+ Neg
The reactions above may be seen in a patient who is:
a. A1 with acquired B.
b. A2B with anti-A1.
c. AB with increased concentrations of protein in the
serum.
d. AB with an autoantibody.
Chapter 6 The ABO Blood Group System 147
7. Which of the following ABO blood groups contains the
least amount of H substance?
a. A1B
b. A2
c. B
d. O
8. You are working on a specimen in the laboratory that
you believe to be a Bombay phenotype. Which of the following
reactions would you expect to see?
a. Patient’s cells + Ulex europaeus = no agglutination
b. Patient’s cells + Ulex europaeus = agglutination
c. Patient’s serum + group O donor RBCs = no
agglutination
d. Patient’s serum + A1 and B cells = no agglutination
9. An example of a technical error that can result in an
ABO discrepancy is:
a. Acquired B phenomenon.
b. Missing isoagglutinins.
c. Cell suspension that is too heavy.
d. Acriflavine antibodies.
10. An ABO type on a patient gives the following reactions:
Patient Cells
With Patient Serum With
Anti-A Anti-B A1 cells B cells O cells Autocontrol
4+ Neg 2+ 4+ 2+ Neg
These results are most likely due to:
a. ABO alloantibody.
b. Non-ABO alloantibody.
c. Rouleaux.
d. Cold autoantibody.

Review Questions7
1. The Rh system genes are:
a. RHD and RHCE.
b. RHD and LW.
c. RHD and RHAG.
d. RHCE and RHAG.
2. What Rh antigen is found in 85% of the Caucasian population
and is always significant for transfusion purposes?
a. d
b. c
c. D
d. E
3. How are weaker-than-expected reactions with anti-D
typing reagents categorized?
a. Rhmod
b. Weak D
c. DAT positive
d. Dw
4. Cells carrying a weak D antigen require the use of what
test to demonstrate its presence?
a. Indirect antiglobulin test
b. Direct antiglobulin test
c. Microplate test
d. Warm autoadsorption test
5. How are Rh antigens inherited?
a. Autosomal recessive alleles
b. Sex-linked genes
c. Codominant alleles
d. X-linked
6. Biochemically speaking, what type of molecules are Rh
antigens?
a. Glycophorins
b. Simple sugars
c. Proteins
d. Lipids
7. Rh antibodies react best at what temperature (°C)?
a. 15
b. 18
c. 22
d. 37
8. Rh antibodies are primarily of which immunoglobulin
class?
a. IgA
b. IgD
c. IgG
d. IgM
9. Rh antibodies have been associated with which clinical
condition?
a. Hemolytic disease of the fetus and newborn
b. Thrombocytopenia
c. Hemophilia A
d. Stomatocytosis
10. What do Rhnull cells lack?
a. Lewis antigens
b. Normal oxygen-carrying capacity
c. Rh antigens
d. Hemoglobin
Chapter 7 The Rh Blood Group System 171
11. Convert the following genotypes from Wiener nomenclature
to Fisher-Race and Rosenfield nomenclatures,
and list the antigens present in each haplotype.
a. R1r
b. R2R0
c. RzR1
d. r_r
12. Which Rh phenotype has the strongest expression of D?
a. DCe/ce
b. DCe/DCe
c. DcE/DcE
d. D–
13. An individual has the following serologic reactions:
D+C+E+c+e+f–. What is the most probable genotype?
a. R1R2
b. Rory
c. Rzr
d. R1r_
14. Which of the following is the most common haplotype
in the African American population?
a. DCe
b. DcE
c. Dce
d. ce
15. If a patient who is R1R1 is transfused with RBCs that are
Ror, which antibody is he most likely to produce?
a. Anti-D
b. Anti-c
c. Anti-e
d. Anti-G

Review Questions8
1. The following phenotypes are written incorrectly except
for:
a. Jka+
b. Jka+
c. Jka(+)
d. Jk(a+)
2. Which of the following characteristics best describes
Lewis antibodies?
a. IgM, naturally occurring, cause HDFN
b. IgM, naturally occurring, do not cause HDFN
c. IgG, in vitro hemolysis, cause hemolytic transfusion
reactions
d. IgG, in vitro hemolysis, do not cause hemolytic transfusion
reactions
3. The Le gene codes for a specific glycosyltransferase that
transfers a fucose to the N-acetylglucosamine on:
a. Type 1 precursor chain
b. Type 2 precursor chain
c. Types 1 and 2 precursor chains
d. Either type 1 or type 2 in any one individual but not
both
4. What substances would be found in the saliva of a group
B secretor who also has Lele genes?
a. H, Lea
b. H, B, Lea
c. H, B, Lea, Leb
d. H, B, Leb
5. Transformation to Leb phenotype after birth may be as
follows:
a. Le(a–b–) to Le(a+b–) to Le(a+b+) to Le(a–b+)
b. Le(a+b–) to Le(a–b–) to Le(a–b+) to Le(a+b+)
c. Le(a–b+) to Le(a+b–) to Le(a+b+) to Le(a–b–)
d. Le(a+b+) to Le(a+b–) to Le(a–b–) to Le(a–b+)
6. In what way do the Lewis antigens change during
pregnancy?
a. Lea antigen increases only
b. Leb antigen increases only
c. Lea and Leb both increase
d. Lea and Leb both decrease
7. A type 1 chain has:
a. The terminal galactose in a 1-3 linkage to subterminal
N-acetylglucosamine
b. The terminal galactose in a 1-4 linkage to subterminal
N-acetylglucosamine
c. The terminal galactose in a 1-3 linkage to subterminal
N-acetylgalactosamine
d. The terminal galactose in a 1-4 linkage to subterminal
N-acetylgalactosamine
8. Which of the following best describes Lewis antigens?
a. The antigens are integral membrane glycolipids
b. Lea and Leb are antithetical antigens
c. The Le(a+b–) phenotype is found in secretors
d. None of the above
9. Which of the following genotypes would explain RBCs
typed as group A Le(a+b–)?
a. A/O Lele HH Sese
b. A/A Lele HH sese
c. A/O LeLe hh SeSe
d. A/A LeLe hh sese
10. Anti-LebH will not react or will react more weakly with
which of the following RBCs?
a. Group O Le(b+)
b. Group A2 Le(b+)
c. Group A1 Le(b+)
d. None of the above
11. Which of the following best describes MN antigens and
antibodies?
a. Well developed at birth, susceptible to enzymes, generally
saline reactive
b. Not well developed at birth, susceptible to enzymes,
generally saline reactive
c. Well developed at birth, not susceptible to enzymes,
generally saline reactive
d. Well developed at birth, susceptible to enzymes, generally
antiglobulin reactive
12. Which autoantibody specificity is found in patients with
paroxysmal cold hemoglobinuria?
a. Anti-I
b. Anti-i
c. Anti-P
d. Anti-P1
13. Which of the following is the most common antibody
seen in the blood bank after ABO and Rh antibodies?
a. Anti-Fya
b. Anti-k
c. Anti-Jsa
d. Anti-K
14. Which blood group system is associated with resistance
to P. vivax malaria?
a. P
b. Kell
c. Duffy
d. Kidd
15. The null Ko RBC can be artificially prepared by which
of the following treatments?
a. Ficin and DTT
b. Ficin and glycine-acid EDTA
c. DTT and glycine-acid EDTA
d. Glycine-acid EDTA and sialidase
16. Which antibody does not fit with the others with respect
to optimum phase of reactivity?
a. Anti-S
b. Anti-P1
c. Anti-Fya
d. Anti-Jkb
17. Which of the following Duffy phenotypes is prevalent
in blacks but virtually nonexistent in whites?
a. Fy(a+b+)
b. Fy(a–b+)
c. Fy(a–b–)
d. Fy(a+b–)
18. Antibody detection cells will not routinely detect which
antibody specificity?
a. Anti-M
b. Anti-Kpa
c. Anti-Fya
d. Anti-Lub
19. Antibodies to antigens in which of the following blood
groups are known for showing dosage?
a. I
b. P
c. Kidd
d. Lutheran
20. Which antibody is most commonly associated with
delayed hemolytic transfusion reactions?
a. Anti-s
b. Anti-k
c. Anti-Lua
d. Anti-Jka
21. Anti-U will not react with which of the following RBCs?
a. M+N+S+s–
b. M+N–S–s–
c. M–N+S–s+
d. M+N–S+s+
22. A patient with an M. pneumoniae infection will most
likely develop a cold autoantibody with specificity to
which antigen?
a. I
b. i
c. P
d. P1
23. Which antigen is destroyed by enzymes?
a. P1
b. Jsa
c. Fya
d. Jka

Review Questions9
1. The antibody to this high-prevalence antigen demonstrates
mixed-field agglutination that appears shiny and
refractile under the microscope.
a. Vel
b. JMH
c. Jra
d. Sda
2. What red blood cell treatment can be used to differentiate
between anti-D and anti-LW?
a. Ficin
b. Trypsin
c. DTT
d. Papain
3. Which of the following has been associated with causing
severe immediate HTRs?
a. Anti-JMH
b. Anti-Lub
c. Anti-Vel
d. Anti-Sda
4. Which of the following antibodies would more likely be
found in a black patient?
a. Anti-Cra
b. Anti-Ata
c. Anti-Hy
d. All of the above
5. Which of the following antigens is not in a blood group
system?
a. Doa
b. LKE
c. JMH
d. Kx
6. A weakly reactive antibody with a titer of 128 is neutralized
by plasma. Which of the following could be the
specificity?
a. Anti-JMH
b. Anti-Ch
c. Anti-Kna
d. Anti-Kpa
7. An antibody reacted with untreated RBCs and DTTtreated
RBCs but not with ficin-treated RBCs. Which
of the following antibodies could explain this pattern of
reactivity?
a. Anti-JMH
b. Anti-Yta
c. Anti-Cra
d. Anti-Ch
8. The following antibodies are generally considered
clinically insignificant because they have not been
associated with causing increased destruction of RBCs,
HDFN, or HTRs.
a. Anti-Doa and anti-Coa
b. Anti-Ge3 and anti-Wra
c. Anti-Ch and anti-Kna
d. Anti-Dib and anti-Yt
9. Which antigen is the receptor for Haemophilus influenza?
a. AnWj
b. PEL
c. FORS
d. Kna
10. Which antigen is not absent or is weakened on RBCs of
individuals with PNH?
a. Yta
b. Cra
c. CD59
d. Coa
11. Which of the following blood groups is carried on a
structure that helps to maintain the RBC membrane
integrity through interaction with protein band 4.1?
a. Di
b. Kn
c. Ge
d. Vel
12. What is the name of the Knops system serologic null
phenotype?
a. Gregory
b. Leach
c. Helgeson
d. McLeod
13. Which antigen when absent produces a null in the
Dombrock system?
a. Hy
b. Joa
c. Dob
d. Gya
14. Which antigens are strongly expressed on placental
tissue, allowing for the adsorption of antibodies?
a. Cromer
b. Knops
c. Diego
d. Vel
15. Which antigen was returned to the 901 series because
there was no determined linkage to the SMIM1 gene?
a. JMH
b. Ata
c. ABTI
d. MAM
16. The FORS blood group system was first thought to
be part of what system due to the addition of
N-acetylgalactosamine (GalNAc) to the P antigen?
a. ABO
b. Lewis
c. P1PK
d. Globoside
17. What glycophorin expresses the MN CHO collection
antigens that are associated with altered levels of sialic
acid (NeuNAc) or GlcNAc?
a. GPA
b. GPB
c. GPC
d. GPD
18. What techniques can be used to remove the reactivity of
Bg antigens?
a. EDTA/glycine-HCL
b. Platelet adsorption
c. Chloroquine treatment
d. All of the above
19. ABTI was thought to be classified with which antigen
prior to it gaining system status?
a. Jra
b. FORS1
c. Vel
d. Lan
20. The Jr(a–) phenotype is found more commonly in:
a. Japanese.
b. African Americans.
c. South American Indians.
d. Caucasians.

Review Questions10
1. Based on the following phenotypes, which pair of cells
would make the best screening cells?
a. Cell 1: Group A, D+C+c–E–e+, K+, Fy(a+b–), Jk(a+b–),
M+N–S+s–
Cell 2: Group O, D+C–c+E+e–, K–, Fy(a–b+), Jk(a–b+),
M–N+S–s+
b. Cell 1: Group O, D–C–c+E–e+, K–, Fy(a–b+), Jk(a+b+),
M+N–S+s+
Cell 2: Group O, D+C+c–E–e+, K–, Fy(a+b–), Jk(a+b–),
M–N+S–s+
c. Cell 1: Group O, D+C+c+E+e+, K+, Fy(a+b+), Jk(a+b+),
M+N–S+s+
Cell 2: Group O, D–C–c+E–e+, K–, Fy(a+b–), Jk(a+b+),
M+N+S–s+
d. Cell 1: Group O, D+C+c–E–e+, K+, Fy(a–b+), Jk(a–b+),
M–N+S–s+
Cell 2: Group O, D- C–c+E+e–, K–, Fy(a+b–), Jk(a+b–),
M+N–S+s–
2. Antibodies are excluded using RBCs that are homozygous
for the corresponding antigen because:
a. Antibodies may show dosage
b. Multiple antibodies may be present
c. It results in a P value of 0.05 for proper identification
of the antibody
d. All of the above
3. A request for 8 units of RBCs was received for patient LF.
The patient has a negative antibody screen, but 1 of the
8 units was 3+ incompatible at the AHG phase. Which of
the following antibodies may be the cause?
a. Anti-K
b. Anti-Lea
c. Anti-Kpa
d. Anti-Fyb
4. The physician has requested 2 units of RBCs for patient
DB, who has two antibodies, anti-L and anti-Q. The frequency
of antigen L is 45%, and the frequency of antigen
Q is 70% in the donor population. Approximately how
many units will need to be antigen-typed for L and Q to
fill the request?
a. 8
b. 12
c. 2
d. 7
5. Anti-Sda has been identified in patient ALF. What substance
would neutralize this antibody and allow detection
of other alloantibodies?
a. Saliva
b. Hydatid cyst fluid
c. Urine
d. Human breast milk
6. Patient JM appears to have a warm autoantibody. She was
transfused 2 weeks ago. What would be the next step performed
to identify any alloantibodies that might be in her
serum?
a. Acid elution
b. Warm autoadsorption using autologous cells
c. Warm differential adsorption
d. RESt adsorption
7. What is the titer and score for this prenatal anti-D titer?
(Refer to Fig. 10–17.)
a. Titer = 64; score = 52
b. Titer = 1:32; score = 15
c. Titer = 64; score = 21
d. Titer = 32; score = 52

8. Select the antibody(ies) most likely responsible for the


reactions observed.
a. Anti-E and anti-K
b. Anti-Fya
c. Anti-e
d. Anti-Jkb
9. What additional cells need to be tested to be 95% confident
that the identification is correct?
a. Three e-negative cells that react negatively and one
additional e-positive cell that reacts positively
b. One additional E-positive, K-negative cell to react
positively and one additional K-positive, E-negative
cell to react positively
c. Two Jkb homozygous positive cells to react positively
and one Jkb heterozygous positive cell to react
negatively
d. No additional cells are needed
10. Using the panel (Fig. 10–18), select cells that would make
appropriate controls when typing for the C antigen.
a. Cell number 1 for the positive control and cell number
2 for the negative control
b. Cell number 1 for the positive control and cell number
6 for the negative control
c. Cell number 2 for the positive control and cell number
4 for the negative control
d. Cell number 4 for the positive control and cell number
5 for the negative control

11. Which of the following methods may be employed to remove


IgG antibodies that are coating a patient’s red
blood cells?
a. Adsorption
b. Elution
c. Neutralization
d. Titration
12. A technologist has decided to test an enzyme-treated
panel of RBCs against a patient’s serum. Which of the
following antibody pairs could be separated using this
technique?
a. Anti-Jka and anti-Jkb
b. Anti-S and anti-Fya
c. Anti-D and anti-C
d. Anti-Jka and anti-Fya
13. An antibody demonstrates weak reactivity at the AHG
phase when the tube method is used with no enhancement
reagent and monospecific anti-IgG AHG reagent.
When repeating the test, which of the following actions
may increase the strength of the positive reactions?
a. Adding an enhancement reagent, such as LISS or PEG
b. Decreasing the incubation time from 30 minutes to
10 minutes
c. Employing the prewarm technique
d. Decreasing the incubation temperature to 18°C
Review Questions11
1. Which is not included on a properly labeled specimen?
a. Two unique patient identifiers
b. Date and time of draw
c. Phlebotomist’s initials
d. Patient’s home address
2. How many days before a pretransfusion specimen expires?
a. 3 days
b. 7 days
c. 14 days
d. 1 month
3. How many days must a pretransfusion specimen and
donor unit segments be retained post-transfusion?
a. 3 days
b. 7 days
c. 14 days
d. 1 month
4. If a blood type cannot be resolved, what ABO group
should be selected for a red blood cell transfusion?
a. Group A
b. Group B
c. Group O
d. Group AB
266 PART II Blood Groups and Serologic Testing
5. Which antibody specificity is not required in antibody
detection tests?
a. K
b. Cw
c. Fya
d. S
6. A patient has a history of anti-Jka. The antibody screen is
currently negative. Which red blood cell unit should be
selected, and what type of crossmatch should be performed?
a. Jk(a-) red blood cells, computer crossmatch
b. Jk(a-) red blood cells, antiglobulin crossmatch
c. Jk(a-) red blood cells, immediate spin crossmatch
d. ABO-compatible because the antibody screen is
negative
7. Which is not true of rouleaux formation?
a. Mimics agglutination
b. Appears like a “stacking of coins”
c. Can be seen in the antiglobulin test
d. Can be dispersed by saline
8. A patient’s blood type is AB-negative, but there are no
AB-negative red blood cell units available. What donor
units could be selected?
a. A-negative
b. O-positive
c. B-positive
d. All of the above
9. A patient requires 15 units of thawed plasma for
an apheresis procedure. The patient’s blood type is
O-negative. What donor units could be selected?
a. O-negative
b. AB-positive
c. A-negative
d. All of the above
10. The American College of Surgeons recommends transfusion
of red blood cells, thawed plasma, and platelets
in what ratio for a massive transfusion?
a. 2 units of red blood cells for every unit of platelets
b. 1 unit of red blood cells to 1 unit of thawed plasma
to 1 unit of platelets
c. 1 unit of red blood cells to 3 units of thawed plasma
d. It’s an emergency. Give the surgeon whatever she
wants
11. A patient’s antibody screen was positive and an anti-c
was identified. Antiglobulin crossmatches were performed
with c-negative units and 1 of the 6 units was
incompatible. What should be performed to resolve the
incompatible crossmatch?
a. Give O-negative red blood cells
b. Retype the incompatible unit for the c antigen
c. Perform a DAT on the incompatible unit
d. Perform additional identification testing to include
low-specificity antigens
e. b, c, and d
12. A mother, 30 weeks’ pregnant, has anti-K with a titer of 32.
An intrauterine red blood cell transfusion is indicated. The
donor unit selected should be all of the following except:
a. O-negative
b. K-negative
c. Positive for sickling hemoglobin
d. Irradiated
13. A patient with sickle cell disease is B-positive with a positive
antibody screen. The antibody identified is anti-D,
and the autocontrol is negative. What is a possible
explanation?
a. The patient is weak D-positive
b. Autoantibody is present
c. Patient possesses the partial D phenotype
d. The patient has a positive DAT
Review Questions12
1. The endpoint of the CAT test is detected by:
a. Agglutination.
b. Hemolysis.
c. Precipitation.
d. Attachment of indicator cells.
2. The endpoint of the SPRCA test is detected by:
a. Agglutination.
b. Hemolysis.
c. Precipitation.
d. Attachment of indicator cells.
3. The endpoint of the solid-phase protein A assay is:
a. Agglutination.
b. Hemolysis.
c. Precipitation.
d. Attachment of cells to microwell.
4. Protein A captures antibodies by binding to the:
a. Fab portion of immunoglobulin.
b. Fc portion of immunoglobulin.
c. Surface of test cells.
d. Surface of indicator cells.
5. Mixed-field reactions can be observed in:
a. Gel.
b. SPRCA.
c. Protein A technology.
d. None of the automated technologies.
6. An advantage for both CAT and solid-phase technology is:
a. No cell washing steps.
b. Standardization.
c. Use of IgG-coated control cells.
d. Specialized equipment.
7. A disadvantage for both CAT and solid-phase technology
is:
a. Decreased sensitivity.
b. Inability to test hemolyzed, lipemic, or icteric samples.
c. Inability to detect C3d complement–coated cells.
d. Large sample requirement.
8. A safety feature in the SPRCA test is:
a. Air bubble barrier.
b. Viscous barrier.
c. Color change of the LISS.
d. Use of IgG-coated control cells.

Review Questions13
1. Which of the following information is not required for
whole blood donation?
a. Name
b. Address
c. Transfusion history
d. Sex
e. Date of Birth
2. Which of the following would be cause for deferral for a
male donor?
a. Temperature of 99.2°F
b. Hematocrit of 37%
c. Spent 2 weeks in the United Kingdom in 1998
d. Weighs 80 kg
e. Received a blood transfusion 2 years ago
3. Which of the following would be cause for a permanent
deferral?
a. Received a dura mater graft 9 months ago
b. Received hepatitis B immune globulin
c. Is currently on warfarin
d. Diagnosis of babesiosis
e. Traveled to Senegal 2 years ago
4. Immunization for rubella would result in a temporary
deferral for:
a. 4 weeks.
b. 8 weeks.
c. 6 months.
d. 1 year.
e. 3 years.
5. Which of the following donors is acceptable?
a. Donor who had a first-trimester abortion 4 weeks ago
b. Donor whose husband is a hemophiliac who regularly
received cryoprecipitate before 1989
c. Donor who was treated for gonorrhea 6 months ago
d. Donor who had a needle-stick injury 10 months ago
6. Which of the following tests is not required as part of
the donor-processing procedure for allogeneic donation?
a. ABO
b. Rh
c. STS
d. Anti-HTLV-I
e. Anti-CMV
7. How long must a 2-unit RBC donor wait before donating
red blood cells again?
a. 8 weeks
b. 16 weeks
c. 6 months
d. 12 months
8. What is the deferral period for Plavix?
a. 14 days after last dose
b. 1 month after last dose
c. 12 months after last dose
d. 48 hours after last dose
9. All of the following records must be kept for 10 years,
except:
a. Unique ID of each unit.
b. Donor consent.
c. Request for blood or blood component.
d. A signed statement from requesting physician for
emergency release.
10. What is the causative agent of Chagas disease?
a. Trypanosoma cruzi
b. Yersinia pestis
c. Treponema pallidum
d. Plasmodium falciparum

11. Which of the following donors would be rejected for


whole blood donation?
a. A male who had sex with another male in 1988
b. A female who had sex with a male in 1992
c. A male who had sex with another male last month
d. A female who had sex with a male 9 months ago
12. What does “infrequent” refer to when talking about a
plasmapheresis program?
a. Donating no more frequently than once every 4 weeks
b. Donating once a year
c. Donating once every 6 months
d. Donating no more frequently than once every 8 weeks
13. A patient is having an exploratory laparotomy performed
and donated blood for use in the patient’s upcoming
surgery. Three units were collected, with the last unit
collected 2 days before surgery. Given this information,
can the patient undergo surgery as planned?
a. Yes
b. No
14. Which of the following refers to a temporary deferral?
a. Donor received varicella zoster live attenuated vaccine
b. Donor had a confirmed positive test for HBsAg
c. Donor has a history of CJD
d. Donor was diagnosed with babesiosis
15. Which of the following carries a 12-month deferral?
a. Donor received Hepatitis B immune globulin
b. Donor received pituitary growth hormone from
another human
c. Donor received the MMR vaccine
d. Donor spent 10 years in Africa

Review Questions14
1. The fecal-oral route is common in transmitting which of
these hepatitis viruses?
a. HAV and HEV
b. HBV and HCV
c. HDV
d. HGV
2. Which of the following is the component of choice for a
low-birth-weight infant with a hemoglobin of 8 g/dL if
the mother is anti-CMV negative?
a. Whole blood from a donor with anti-CMV
b. RBCs from a donor who is anti-CMV negative
c. Leukoreduced platelets
d. Solvent detergent–treated plasma
3. Which of the following is an FDA-licensed screening test
for HCV?
a. NAT + anti-HBc
b. RIBA
c. Lymph node biopsy
d. HCV RNA
4. Currently, which of the following does the AABB consider
to be the most significant infectious threat from
transfusion?
a. Bacterial contamination
b. CMV
c. Hepatitis
d. HIV
5. Which of the following is the most frequently transmitted
virus from mother to fetus?
a. HIV
b. Hepatitis
c. CMV
d. EBV
6. Jaundice due to HAV is seen most often in the:
a. Adolescent
b. Adult
c. Child younger than 6 years old
d. Newborn
7. Currently, steps taken to reduce transfusion-transmitted
CMV include:
a. Plaque reduction neutralization test
b. NAT testing
c. Leukoreduction
d. Minipool screening
8. HBV remains infectious on environmental surfaces for 1:
a. Day
b. Week
c. Month
d. Year
9. HBV is transmitted most frequently:
a. By needle sharing among IV drug users
b. Through blood transfusions
c. By unknown methods
d. By sexual activity
10. Which of the following is the most common cause of
chronic hepatitis, cirrhosis, and hepatocellular carcinoma
in the United States?
a. HAV
b. HBV
c. HCV
d. HDV
11. The first retrovirus to be associated with human disease
was:
a. HCV
b. HIV
c. HTLV-I
d. WNV
12. All of the following statements are true concerning
WNV except:
a. 1 in 150 infections results in severe neurological
disease
b. Severe disease occurs most frequently in the over-
50 age group
c. Deaths occur more often in those over 65 years who
present with encephalitis
d. Fatalities occur in approximately 38% of infected
individuals
13. The primary host for WNV is:
a. Birds
b. Horses
c. Humans
d. Bats
14. Tests for WNV include all of the following except:
a. ELISA
b. NAT
c. Plaque reduction neutralization test
d. Immunofluorescent antibody assay
15. Individuals exposed to EBV maintain an asymptomatic
latent infection in:
a. B cells
b. T cells
c. All lymphocytes
d. Monocytes
16. Fifth disease is caused by:
a. CMV
b. EBV
c. Parvovirus B19
d. HTLV-II
17. Transient aplastic crisis can occur with:
a. Parvovirus B19
b. WNV
c. CMV
d. EBV
18. Reasons why syphilis is so rare in the U.S. blood supply
include all of the following except:
a. 4°C storage conditions
b. Donor questionnaire
c. Short spirochetemia
d. NAT testing
19. Nucleic acid amplification testing for HIV was instituted
in donor testing protocols to:
a. Identify donors with late-stage HIV who lack
antibodies
b. Confirm the presence of anti-HIV in asymptomatic
HIV-infected donors
c. Reduce the window period by detecting the virus
earlier than other available tests
d. Detect antibodies to specific HIV viral proteins,
including anti-p24, anti-gp41, and anti-gp120
20. Screening for HIV is performed using the following
technique:
a. Radio immunoassay
b. WB
c. Immunofluorescent antibody assay
d. NAT
21. The first form of pathogen inactivation was:
a. Chemical
b. Heat
c. Cold-ethanol fractionation
d. Anion-exchange chromatography
22. What is the most common parasitic complication of
transfusion?
a. Babesia microti
b. Trypanosoma cruzi
c. Plasmodium species
d. Toxoplasma gondii
23. Which organism has a characteristic C- or U-shape on
stained blood smears?
a. Trypanosoma cruzi
b. Plasmodium vivax
c. Plasmodium falciparum
d. Babesia microti
24. Which transfusion-associated parasite may have asymptomatic
carriers?
a. Babesia microti
b. Trypanosoma cruzi
c. Plasmodium species
d. All of the above
25. Which disease is naturally caused by the bite of a deer
tick?
a. Chagas disease
b. Babesiosis
c. Malaria
d. Leishmaniasis
Review Questions15
1. Which of the following lists the correct shelf life for the
component?
a. Deglycerolized RBCs—24 hours
b. RBCs (CPD)—35 days
c. Platelet concentrate—10 days
d. FFP—5 years
e. RBCs (CPDA-1)—21 days
Zika virus by investigational nucleic acid test, and RBC phenotypes.
(See Figure 15-3.)
The unique identifier of the unit, the ABO and Rh type,
expiration date, and component labels must be checked with
a second person who is using a computer. If the unit is
shipped to a transfusion facility that applies its own unique
identifier for the unit, the original identifier of the collecting
facility must not be removed. There must be a method in place
for tracing the unit from its origin to its final disposition.
2. Each unit of cryoprecipitate prepared from whole blood
should contain a minimum of how many units of AHF
activity?
a. 40 IU
b. 80 IU
c. 120 IU
d. 160 IU
e. 180 IU

3. Platelet concentrates prepared by apheresis should contain


how many platelets?
a. 5.5 × 1010
b. 6 × 1010
c. 3 × 1011
d. 5.5 × 1011
e. 6 × 1011
4. The required storage temperature for frozen RBCs using
the high-glycerol method is:
a. 4°C
b. ≤–20°C
c. ≤–18°C
d. ≤–120°C
e. ≤–65°C
5. How does irradiation affect the shelf life of red blood
cells?
a. Irradiation has no effect on the shelf life
b. The expiration date is 28 days from the date of irradiation
or the original outdate, whichever is later
c. The expiration date is 28 days from the date of irradiation
or the original outdate, whichever is sooner
d. The expiration date is 25 days from the date of irradiation
or the original outdate, whichever is later
e. The expiration date is 25 days from the date of irradiation
or the original outdate, whichever is sooner
6. Once thawed, FFP must be transfused within __________
hours unless relabeled as thawed plasma:
a. 4
b. 6
c. 8
d. 12
e. 24
7. Quality control for nonadditive RBCs requires a maximum
hematocrit level of:
a. 75%
b. 80%
c. 85%
d. 90%
e. 95%
8. AHF concentrates are used to treat:
a. Thrombocytopenia
b. Hemophilia A
c. Hemophilia B
d. von Willebrand’s disease
e. Factor XIII deficiency
9. Prothrombin complex concentrates are used to treat
which of the following?
a. Factor IX deficiency
b. Factor VIII deficiency
c. Factor XII deficiency
d. Factor XIII deficiency
e. Factor V deficiency

10. RBCs that have been leukoreduced must contain less


than ______ leukocytes and retain at least ______ of
original RBCs.
a. 8 × 106/85%
b. 8 × 106/90%
c. 5 × 106/85%
d. 5 × 106/80%
11. Random-donor platelets that have been leukoreduced
must contain less than ______ leukocytes.
a. 8.3 × 105
b. 8 × 106
c. 5 × 106
d. 3 × 1011
12. A single unit of FFP or PF24 should contain ______ mL
of plasma.
a. 100–150
b. 200–400
c. 150–250
d. 50–150
13. Cryoprecipitate that has been pooled in an open system
must be transfused within ______ hours.
a. 24
b. 6
c. 4
d. 8

Review Questions16
1. Leukocyte-reduced filters can do all of the following
except:
a. Reduce the risk of CMV infection
b. Prevent or reduce the risk of HLA alloimmunization
c. Prevent febrile, nonhemolytic transfusion reactions
d. Prevent TA-GVHD
2. Albumin should not be given for:
a. Burns
b. Shock
c. Nutrition
d. Plasmapheresis
3. Of the following, which blood type is selected when a
patient cannot wait for ABO-matched RBCs?
a. A
b. B
c. O
d. AB
4. Which patient does not need an irradiated component?
a. Bone marrow transplant recipient
b. Neonate weighing less than 1,200 g
c. Adult receiving an RBC transfusion
d. Adult receiving an RBC transfusion from a blood
relative
5. RBC transfusions should be given:
a. Within 4 hours
b. With lactated Ringer solution
c. With dextrose and water
d. With cryoprecipitate
6. Which type of transplantation requires all cellular blood
components to be irradiated?
a. Bone marrow
b. Heart
c. Liver
d. Kidney
7. Characteristics of deglycerolized RBCs include the
following except:
a. Inexpensive
b. 24-hour expiration date after thawing
c. Used for rare antigen-type donor blood
d. Used for IgA-deficient recipient with history of severe
reaction
8. Select the appropriate product for a bone marrow transplant
patient with anemia:
a. RBCs
b. Irradiated RBCs
c. Leukoreduced RBCs
d. Washed RBCs
9. Which blood product should be selected for vitamin K
deficiency?
a. Cryoprecipitate
b. Factor VIII
c. Factor IX
d. Plasma
10. Which fluid should be used to dilute RBCs?
a. 0.9% saline
b. 5% dextrose and water
c. Immune globulin
d. Lactated Ringer solution

Review Questions17
1. What component is most frequently involved with
transfusion-associated sepsis?
a. Plasma
b. Packed red blood cells
c. Platelets
d. Whole blood
2. Fatal transfusion reactions are mostly caused by:
a. Serologic errors
b. Improper storage of blood
c. Clerical errors
d. Improper handling of the product
3. Early manifestation of an acute hemolytic transfusion
reaction can be confused with:
a. Allergic reaction
b. Febrile nonhemolytic reaction
c. Anaphylactic shock
d. Sepsis
4. Pain at infusion site and hypotension are observed with
what type of reaction?
a. Delayed hemolytic transfusion reaction
b. Acute hemolytic transfusion reaction
c. Allergic reaction
d. Febrile nonhemolytic reaction
5. Irradiation of blood is performed to prevent:
a. Febrile nonhemolytic transfusion reaction
b. Delayed hemolytic transfusion reaction
c. Transfusion-associated graft-versus-host disease
d. Transfusion-associated circulatory overload
6. The only presenting sign most often accompanying a
delayed hemolytic transfusion reaction is:
a. Renal failure
b. Unexplained decrease in hemoglobin
c. Active bleeding
d. Hives
7. Which transfusion reaction presents with fever, maculopapular
rash, watery diarrhea, abnormal liver function,
and pancytopenia?
a. Transfusion-associated sepsis
b. Transfusion-related acute lung injury
c. Transfusion-associated graft-versus-host disease
d. Transfusion-associated allergic reaction
8. A suspected transfusion-related death must be reported to:
a. AABB
b. Federal and Drug Administration (FDA)
c. College of American Pathologists (CAP)
d. The Joint Commission (TJC)
9. Nonimmune hemolysis can be caused during transfusion
by:
a. Use of small bore size needle
b. Use of an infusion pump
c. Improper use of a blood warmer
d. All of the above
10. Transfusion reactions are classified according to:
a. Signs or symptoms presenting during or after
24 hours
b. Immune or nonimmune
c. Infectious or noninfectious
d. All of the above
11. With febrile nonhemolytic transfusion reactions:
a. They are self-limited
b. Fever resolves within 2–3 hours
c. Treatment is required
d. a and b are correct
e. All of the above
12. Absolute IgA deficiency is a classic example of a severe
allergic reaction. A result indicating an absolute IgA
deficiency is:
a. <0.05 mg/dL
b. <0.50 mg/dL
c. <0.50 gm/dL
d. <5 mg/dL
13. How are mild allergic transfusion reactions with isolated
symptoms or hives and urticaria treated?
a. Transfusion is stopped and transfusion reaction
workup is initiated
b. Transfusion is stopped and antihistamines administrated;
when symptoms improve, transfusion is
restarted
c. Stop transfusion and prepare washed red blood cells
d. Continue transfusion with a slower infusion rate
14. TRALI presents with the following symptoms:
a. Respiratory distress
b. Severe hypoxemia and hypotension
c. Fever
d. All of the above
15. Which of the following is characteristic of iron overload?
a. Delayed, nonimmune complication occurs
b. Chelating agents are used
c. Multiorgan damage may occur
d. All of the above

Review Questions18
1. The most common anticoagulant used for apheresis
procedures is:
a. Heparin
b. Sodium fluoride
c. Warfarin
d. Citrate
2. Therapeutic cytapheresis has a primary role in treatment
of patients with:
a. Sickle cell disease and acute chest syndrome
b. Systemic lupus erythematosus to remove immune
complexes
c. Leukemia to help increase granulocyte production
d. Myasthenia gravis to increase antibody production
3. The minimum interval allowed between plateletpheresis
component collection procedures is:
a. 1 day
b. 2 days
c. 7 days
d. 8 weeks
4. In plasma exchange, the therapeutic effectiveness is:
a. Greatest with the first plasma volume removed
b. Affected by the type of replacement fluid used
c. Enhanced if the unwanted antibody is IgG rather than
IgM
d. Independent of the use of concomitant immunosuppressive
therapy
5. The replacement fluid indicated during plasma exchange
for TTP is:
a. Normal (0.9%) saline
b. Hydroxyethyl starch (HES)
c. FFP
d. Albumin (human) 5%
6. The most common adverse effect of plateletpheresis collection
is:
a. Allergic reaction
b. Hepatitis
c. Hemolysis
d. Citrate effect
7. Apheresis technology can be used to collect each of the
following components except:
a. Leukocytes
b. Macrophages
c. Hematopoietic progenitor cells
d. Platelets
8. The anticoagulant added to blood as it is removed from a
donor or patient during an apheresis procedure acts by:
a. Binding calcium ions
b. Increasing intracellular potassium
c. Binding to antithrombin III
d. Inactivating factor V

9. Peripheral blood stem cells are:


a. Responsible for phagocytosis of bacteria
b. Removed during erythrocytapheresis
c. Pluripotential hematopoietic precursors that circulate
in the peripheral blood
d. Lymphocytes involved with the immune response
10. Which of the following can be given to an apheresis donor
to increase the number of circulating granulocytes?
a. DDAVP
b. Hydroxyethyl starch (HES)
c. Immune globulin
d. G-CSF

Review Questions19
1. When an HPC donor is unrelated to the recipient of an
HPC transplantation, the transplant is categorized as:
a. Allogeneic
b. Autologous
c. Syngeneic
d. Hematopoietic
2. Stem cells from HPC donors may be mobilized with:
a. Plerixafor
b. Filgrastim (GCSF)
c. Chemotherapy
d. All of the above
3. Which is an advantage of an HPC transplant using umbilical
cord blood as the HPC source?
a. Recipient weight of no concern
b. Donor screening and testing abbreviated
c. Higher risk of GVHD
d. No significant risk to the donor or mother
4. The recommended minimum number of CD34+ cells required
in an HPC-apheresis collection to ensure timely
engraftment is:
a. 2 × 102 CD34+ cells/kg
b. 2 × 104 CD34+ cells/kg
c. 2 × 106 CD34+ cells/kg
d. 2 × 108 CD34+ cells/kg
5. The cellular marker used to quantify the collection of
HPCs using flow cytometry is:
a. CD4
b. CD33
c. CD34
d. CD59
6. An A patient received an HPC transplant from a B donor.
What type of ABO mismatch does this represent?
a. Major
b. Minor
c. Bidirectional
d. Any of the above
7. Which of the following terms describe an HPC transplant
where donor and recipient are the same person?
a. Allogeneic
b. Autologous
c. Syngeneic
d. Hematopoietic
8. Three weeks after sustaining a car accident that required
emergency transfusion of blood products for resuscitation,
an allogeneic HPC transplant recipient developed
a fever, erythematous skin rash, diarrhea, and cytopenias,
which ultimately were fatal. What intervention
may have prevented this outcome?
a. The use of leukoreduced blood products
b. The use of irradiated of blood products
c. The use of CMV-negative blood products
d. The use of washed blood products
9. What common cryoprotectant is added to HPC products
for freezing?
a. Dimethyl sulfoxide
b. Polyethylene glycol
c. Glycerol
d. Normal saline
10. An O patient received an HPC transplant from a B donor.
What type of ABO mismatch does this represent?
a. Major
b. Minor
c. Bidirectional
d. Any of the above

Review Questions20
1. The etiology of HDFN is characterized by:
a. IgM antibody
b. Nearly always anti-D
c. Different RBC antigens between mother and father
d. Antibody titer less than 32
2. An important difference between the fetus and the
newborn physiology is:
a. Bilirubin metabolism
b. Maternal antibody level
c. Presence of anemia
d. Size of RBCs
3. Kernicterus is caused by the effects of:
a. Anemia
b. Unconjugated bilirubin
c. Antibody specificity
d. Antibody titer
4. The advantage of middle cerebral artery peak systolic
velocity Doppler (MCA-PSV) is that it is:
a. Able to measure fetal hemoglobin and hematocrit
levels
b. Able to support antigen typing of fetal blood using
DNA
c. Helpful for direct transfusion of fetal circulation
d. Noninvasive and decreases risk of adverse events
5. Blood for intrauterine transfusion (IUT) should be:
a. Irradiated, leukocyte reduced, more than 7 days old,
HbS negative
b. Irradiated, leukocyte reduced, less than 7 days old,
HbS positive
c. Irradiated, leukocyte reduced, less than 7 days old,
HbS negative
d. Irradiated, leukocyte reduced, more than 7 days old,
HbS positive
6. RhIG is indicated for:
a. Mothers who have anti-D due to allosensitization
b. Infants who are RhD-negative
c. Infants who have anti-D
d. Mothers who are RhD-negative
7. RhIG is given to RhD-negative mothers without regard
for fetal RhD type in all of the following conditions
except:
a. Ectopic pregnancy rupture
b. Full-term delivery
c. Amniocentesis
d. Induced abortion
8. A Kleihauer-Betke test or flow cytometry indicates 10 fetal
cells per 1,000 adult cells. For a woman with 5,000-mL
blood volume, the proper dose of RhIG is:
a. One regular-dose (300 μg) vial
b. Two regular-dose vials
c. Three regular-dose vials
d. Four regular-dose vials
9. ABO HDFN is usually mild because:
a. ABO antigens are poorly developed in the fetus
b. ABO antibodies prevent the disease itself
c. ABO antibodies readily cross the placenta
d. ABO incompatibility is rare
10. A woman without prenatal care delivers a healthy term
infant. A cord blood sample shows the infant is A-positive
with a positive DAT. The workup of the unexpected finding
should include:
a. Anti-C3 antiglobulin test
b. ABO testing of the mother
c. Direct antiglobulin testing of the mother’s specimen
d. ABO and Rh typing of the father

Review Questions21
1. Immune hemolytic anemias may be classified in which of
the following categories?
a. Alloimmune
b. Autoimmune
c. Drug-induced
d. All of the above
2. When preparing cells for a cold autoadsorption procedure,
it is helpful to pretreat the cells with which of the
following?
a. Dithiothreitol
b. Ficin
c. Phosphate-buffered saline at pH 9
d. Bovine albumin
3. The blood group involved in the autoantibody specificity
in PCH is:
a. P.
b. ABO.
c. Rh.
d. Lewis.
4. Which of the following blood groups reacts best with an
anti-H or anti-IH?
a. O
b. B
c. A2
d. A1
5. With cold-reactive autoantibodies, the protein coating
the patient’s cells and detected in the DAT is:
a. C3.
b. IgG.
c. C4.
d. IgM.
6. Problems in routine testing caused by cold-reactive
autoantibodies can usually be resolved by all of the
following except:
a. Prewarming.
b. Washing with warm saline.
c. Using anti-IgG antiglobulin serum.
d. Testing clotted blood specimens.
7. Pathological cold autoagglutinins differ from common
cold autoagglutinins in:
a. Immunoglobulin class.
b. Thermal amplitude.
c. Antibody specificity.
d. DAT results on EDTA specimen.
8. Cold AIHA is sometimes associated with infection by:
a. Staphylococcus aureus.
b. Mycoplasma pneumoniae.
c. Escherichia coli.
d. Group A Streptococcus.
9. Many warm-reactive autoantibodies have a broad specificity
within which of the following blood groups?
a. Kell.
b. Duffy.
c. Rh.
d. Kidd.
10. Valid Rh typing can usually be obtained on a patient
with WAIHA using all of the following reagents or techniques
except:
a. Slide and modified tube anti-D.
b. Chloroquine-treated RBCs.
c. Rosette test.
d. Monoclonal anti-D.
11. In pretransfusion testing for a patient with WAIHA, the
primary concern is:
a. Treating the patient’s cells with chloroquine for reliable
antigen typing.
b. Adsorbing out all antibodies in the patient’s serum to
be able to provide compatible RBCs.
c. Determining the exact specificity of the autoantibody
so that compatible RBCs can be found.
d. Discovering any existing significant alloantibodies in
the patient’s circulation.
12. Penicillin given in massive doses has been associated
with RBC hemolysis. Which of the classic mechanisms
is typically involved in the hemolytic process?
a. Immune complex.
b. Drug adsorption.
c. Membrane modification.
d. Autoantibody formation.
13. Which of the following drugs has been associated
with complement activation and rapid intravascular
hemolysis?
a. Penicillins.
b. Quinidine.
c. Alpha-methyldopa.
d. Cephalosporins.
14. A patient is admitted with a hemoglobin of 5.6 g/dL. Initial
pretransfusion workup appears to indicate the presence of
a warm autoantibody in the serum and coating his RBCs.
His transfusion history indicates that he received 6 units
of RBCs 2 years ago after an automobile accident. Which
of the following would be most helpful in performing
antibody detection and compatibility testing procedures?
a. Adsorb the autoantibody using the patient’s enzymetreated
cells.
b. Perform an elution and use the eluate for compatibility
testing.
c. Crossmatch random units until compatible units are
found.
d. Collect blood from relatives who are more likely to
be compatible.
15. A patient who is taking Aldomet has a positive DAT. An
eluate prepared from his RBCs would be expected to:
a. React only with Aldomet-coated cells.
b. Be neutralized by a suspension of Aldomet.
c. React with all normal cells.
d. React only with Rhnull cells.
16. One method that can be used to separate a patient’s
RBCs from recently transfused donor RBCs is:
a. Chloroquine diphosphate treatment of the RBCs.
b. Reticulocyte harvesting.
c. EGA treatment.
d. Donath-Landsteiner testing.
17. Monoclonal antisera is valuable in phenotyping RBCs
with positive DATs because:
a. Both polyspecific and monospecific antihuman serum
can be used in antiglobulin testing.
b. Anti-C3 serum can be used in antiglobulin testing.
c. It usually does not require antiglobulin testing.
d. It does not require enzyme treatment of the cells prior
to antiglobulin testing.
18. Autoadsorption procedures to remove either warm or
cold autoantibodies should not be used with a recently
transfused patient. Recently means:
a. 3 days.
b. 3 weeks.
c. 6 weeks.
d. 3 months.

Review Questions22
1. Implant records must be kept for what duration?
a. Ten years after the tissue has been harvested
b. Indefinitely
c. For a reasonable time to ensure that the recipient is not
still alive when records are destroyed
d. Ten years following the disposition or expiration of the
tissue
2. FDA CFR 1270 and 1271 include all of the following
tissues except:
a. Cancellous bone chips
b. Blood vessels associated with vascular organs for
transplant
c. Cornea
d. Heart valve
3. Hospital tissue banks must register with the FDA if:
a. Tissue for transplant is stored
b. Autologous tissue is stored and issued
c. Tissue is transferred to another facility
d. The tissue bank is located outside the blood bank
4. The Joint Commission requires all of the following except:
a. Hospital tissue banks must ensure that suppliers are
complying with applicable state laws
b. Tissue-manufacturing establishments must register
with the FDA
c. Hospitals must assign responsibility for overseeing the
tissue program throughout the organization
d. Hospital tissue banks must verify supplier’s registration
with the FDA yearly
5. The American Association of Tissue Banks (AATB) is:
a. A mandatory accrediting agency for all tissue banks
b. A voluntary accrediting agency for tissue-manufacturing
establishments
c. An historic name for the U.S. Navy Tissue Bank
d. A subdivision within the AABB
6. Transmission of malignancy in tissue:
a. Is most likely to occur with the use of bone
b. Is relatively common (1/10,000 cases)
c. Is more likely to occur in whole organ transplant
d. Has never been reported in cornea transplant
7. The medical director for the tissue bank can be:
a. Any individual appointed by the hospital medical
director
b. The lead supervisor in the blood bank
c. The head nurse/transplant coordinator from surgical
nursing
d. A qualified physician involved in tissue transplant or
blood banking
8. Notification of a recipient of tissue that has been recalled
because of possible contamination:
a. Should be conducted by the tissue bank director only
b. Should be conducted by the patient’s transplanting
surgeon
c. The patient does not need to be told unless an infection
develops
d. The informed consent covers this contingency and no
further notification is necessary

9. Tissue receipt records must include all of the following


except:
a. Unique tissue identification number
b. Name and address of tissue supplier
c. Expiration date
d. Tissue supplier’s FDA registration number
10. Records that must be reviewed to determine donor
eligibility by the tissue manufacturer include:
a. Donor family history
b. Records from any source pertaining to risk factors for
communicable diseases
c. Interview of next-of-kin
d. Consent to harvest tissue

Review Questions23
1. The HLA genes are located on which chromosome?
a. 2
b. 4
c. 6
d. 8
2. The majority of HLA antibodies belong to what immunoglobulin
class?
a. IgD
b. IgE
c. IgG
d. IgM
3. What is the test of choice for HLA antigen testing?
a. Agglutination
b. Molecular
c. Cytotoxicity
d. ELISA
4. Of the following diseases, which one has the highest relative
risk in association with an HLA antigen?
a. Ankylosing spondylitis
b. Juvenile diabetes
c. Narcolepsy
d. Rheumatoid arthritis
5. Why is HLA matching not feasible in cardiac transplantation?
a. No HLAs are present on cardiac cells
b. No donors ever have HLA antibodies
c. Total ischemic time is too long
d. Total ischemic time is too short
6. DR52 molecules are the product of which alleles?
a. DRA and DRB1
b. DRA and DRB3
c. DRA and DRB4
d. DRA and DRB5
7. What is the molecular technique that detects undefined
alleles?
a. Restriction fragment length polymorphism
b. Sequence-specific primer typing
c. Sequence-specific oligonucleotide typing
d. Direct nucleotide sequencing
8. What represents the association of the alleles on the two
C6 chromosomes as determined by family studies?
a. Haplotype
b. Genotype
c. Phenotype
d. Xenotype

9. Which type of HSCT can be performed within a relatively


short period of time?
a. Matched unrelated
b. Cord blood
c. Autologous
d. HLA haploidentical
10. The SAB describes the amount of bound antibody on
each bead as:
a. MCS
b. MFI
c. CDC
d. AHG

Review Questions24
1. Among the combinations of attributes described below,
select the one that would not be suitable for a genetic
system used in parentage testing analysis.
a. The system has multiple alleles in Hardy-Weinberg
equilibrium
b. The system has a high mutation rate
c. Databases of allele frequencies are available for all
ethnic groups tested by the laboratory
d. All systems selected are genetically independent from
each other
2. In which of the following genetic systems is the allele
frequency distribution continuous (not discrete)?
a. DNA polymorphisms by RFLP
b. DNA polymorphisms by PCR
c. RBC antigens
d. RBC enzymes
3. A false direct exclusion in RBC antigen genetic systems
can be caused by:
a. A silent allele
b. A lack of precursor substance
c. An alternate untested allele
d. Weak reagents
4. Among the following organizations, which one offers an
accreditation program for parentage testing laboratories?
a. AABB
b. ASCP
c. FDA
d. HCFA

Review Questions25
1. A compliance program:
a. Evaluates how effectively the facility meets regulatory
requirements
b. Always identifies quality problems
c. Is part of quality control
d. Is an evaluation of efficiency
2. The quality system essentials are applied to:
a. The blood bank’s management staff
b. Blood bank quality control activities
c. Blood component manufacturing
d. The blood bank’s path of workflow
3. cGMP refers to:
a. Regulations pertaining to laboratory safety
b. Validation of testing
c. Nonconformance reporting
d. Manufacturing blood components
4. Internal and external failure costs are:
a. Readily identifiable in facility reports
b. Controlled through prevention and appraisal
c. Built into the facility’s operating budget
d. Part of prevention and appraisal
5. Which one statement below is correct?
a. A process describes how to perform a task
b. A procedure simply states what the facility will do
c. A procedure informs the reader how to perform a task
d. A policy can be flowcharted
6. A blank form is a:
a. Record
b. Procedure
c. Flowchart
d. Document
7. An example of a remedial action is:
a. Applying the problem-solving process
b. Starting a process improvement team
c. Resolving the immediate problem
d. Performing an internal audit
8. The DMAIC methodology is used for:
a. Problem resolution
b. Process control
c. Validation
d. Auditing
9. ___________________________ is a set of planned
actions that ensure that systems and elements that influence
the quality of service are working as expected.
a. Quality control
b. Quality assurance
c. Quality indicator
d. Quality management system
10. The Centers for Medicare and Medicaid Services (CMS)
developed an alternative quality control option, an
individualized quality control plan (IQCP). How is
the minimum frequency of running quality controls
determined?
a. Through risk assessment
b. By the quality control plan
c. After quality assessment
d. By the manufacturer

Review Questions26
1. Which type of review does not require direct discussion
between the ordering clinician and transfusion service
personnel?
a. Discontinuous prospective
b. Targeted prospective
c. Concurrent
d. Retrospective
e. Prospective
2. Which of the following is the most important first step in
developing a comprehensive PBM/BUM program?
a. Identification of a qualified transfusion safety officer
with excellent blood banking bench skills
b. Determination of patient populations within the hospital
with the highest blood utilization
c. Creation of a multidisciplinary transfusion committee
to determine the category of blood utilization review
d. Meet with key physician and nursing leadership to
facilitate the creation of a hospital-wide transfusion
guideline
e. Determine the estimated cost savings through the implementation
of an anemia clinic for elective surgical
patients
3. Optimal value as it relates to blood utilization is best
obtained by which of the following:
a. Development of standardized cost metric for blood
utilization
b. Reduction in variabilities in transfusion ordering practice
from evidence-based standards
c. Consistent reporting of outcome metrics such as
decreased sepsis or emergency room admissions
d. Decreasing peril and waste by encouraging bloodless
surgery and conversion to lower volume phlebotomy
tubes
e. Use of LEAN practices to improve value and prevent
waste by considering the ordering clinician as a customer
4. To receive benefit from a transfusion the patient must
have:
a. A hemoglobin level less than 8 g/dL
b. An invasive procedure planned
c. A pathological lesion or deficiency that can be remedied
by functioning stored components
d. A blood order signed by the attending physician
e. Understood the risks, benefits and alternatives, and
given informed consent
5. Which of the following statements best describes the most
important reason for creating a PBM/BUM program?
a. PBM/BUM decreases blood bank exposure to risk management
and litigation by promoting optimal documentation
of transfusion indication and expected
outcome within the electronic medical record
b. PBM/BUM reduces patient exposure to transfusion associated
acute lung injury (TRALI) through physician
education and by prospectively encouraging mitigation
strategies from the blood supplier
c. PBM/BUM improves patient safety by promoting
evidence-based transfusion, transfusion avoidance
strategies, and prevention of inappropriate transfusion
d. PBM/BUM are laboratory and hospital regulatory requirements
that are essential for ensuring a hospital
culture that promotes patient safety through the periodic
direct observation and assessment of transfusion
administration by nursing staff
e. PBM/BUM provides significant and substantial direct
and indirect cost savings to both patients and blood
banks by reducing the number of unnecessary or inappropriate
transfusions
6. Which of the following is an example of targeted prospective
review as it related to blood utilization?
a. Continuing education on the proactive use of iron to
correct anemia in presurgical patients
b. Viscoelastic testing to assess real-time platelet need for
cardiac surgery patients
c. Hematologist directed erythropoietin clinic for anemic
cancer patients prior to chemotherapy
d. Decision support pop-up restricted to routine orders
for two or more units of RBCs
e. Report of average RBC usage per patient for a targeted
procedure or physician group
7. Which of the following pairs best describes an intervention
strategy that is best paired with a utilization review
category?
a. Minimize unnecessary phlebotomy loss, discontinuous
prospective review
b. Periodic physician feedback, retrospective review
c. Annual continuing education for medical residents,
prospective review
d. Predictive modeling, retrospective review
e. Automatic cancellation of surgical blood orders, concurrent
review
8. The PBM program planning team should include:
a. Nursing representatives
b. Representatives from the facility’s practicing clinicians
c. The blood bank medical director
d. A laboratorian who is knowledgeable in blood bank
policies and procedures
e. All of the above
9. Metrics should be:
a. Chosen to indicate progress during the process of
continuous improvement
b. Tracked and disseminated only to members of the
blood utilization management team
c. The same for all institutions
d. Selected only by the transfusion service leadership
e. Only qualitative in nature, since medical decision
making is a complex process
10. Which statement is most accurate regarding continuous
improvement as it relates to PBM/BUM?
a. Blood utilization metrics should be periodically converted
to national standardized metrics
b. An example of an ideal PBM metric is the ratio of the
facility cost of one unit of RBCs divided by the regional
red blood cell cost
c. Continuous improvement should be performed by a
different set of players than were used in the planning
process
d. Outcomes during prospective review should be limited
to the transfusion committee
e. Feedback loops and appropriate metrics are required
to achieve long-term improvements

Review Questions27
1. Which of the following is responsible for overseeing the
safety of the nation’s blood supply?
a. Joint Commission on Accreditation of Healthcare
Organizations
b. Food and Drug Administration
c. College of American Pathologists (CAP)
d. Occupational Safety and Health Administration
2. Where are the regulations for blood and blood components
published?
a. The AABB Technical Manual
b. CAP inspection checklist
c. The Code of Federal Regulations
d. State Inspectional Guidance Documents
3. What was the important tragedy that led to the regulation
of biological products?
a. Three patients contracted hepatitis C following transfusion
b. A child died following transfusion of hemolyzed red
blood cells
c. A group O patient received group A blood
d. Thirteen children died after receiving diphtheria antitoxin
contaminated with tetanus
4. What is required to ship blood and blood components
across state lines (interstate)?
a. AABB accreditation
b. State license
c. CMS certification
d. Approved biologics license application
5. Which of the following government organizations inspect
blood and blood component manufacturers?
a. CBER
b. ORA
c. CMS
d. All of the above
6. Which of the following is true about CGMP?
a. CGMP is the minimum current practice for methods
and facilities used to manufacture a drug to ensure
that it is safe, pure, and potent
b. The FDA will approve a biologics license application
if the manufacturer does not have a quality control
plan
c. The quality control unit must perform all the quality
functions
d. Blood and blood components do not have to be in
compliance with the drug CGMP regulations
7. A donor calls the blood bank and informs them that
within a year prior to his donation, he had intimate contact
with a person diagnosed with HIV. Which of the following
actions is not required by the FDA?
a. Identify and quarantine all blood and blood components
produced from the blood supplied by the
donor
b. Report the biological product deviation to CBER if
the product has been distributed
c. Enter the donor in a record so that he can be identified
and his product not be distributed while he is
deferred
d. Notify the AABB
8. A patient dies following transfusion of ABO-incompatible
blood. To whom should this event be reported?
a. The Center for Biologics Evaluation and Research
b. Center for Medicare and Medicaid Services
c. The AABB central office
d. The Occupational Safety and Health Administration
9. Which federal agency has the responsibility to routinely
inspect an unregistered transfusion service that does not
collect blood?
a. Food and Drug Administration
b. Centers for Medicare and Medicaid Services
c. Occupational Safety and Health Administration
d. State health department
10. Which of the following is not one of the FDA layers of
safety?
a. Donor screening
b. Biologics License Application
c. Investigation of manufacturing problems
d. Testing for relevant transfusion-transmitted infections

Review Questions28
1. Components of an information system consist of all of
the following except:
a. Hardware
b. Software
c. Validation
d. People
2. To be in compliance with regulatory and accreditation
agency requirements for blood bank information systems,
blood banks must maintain SOPs for all of the following
except:
a. Vendor validation testing
b. Computer downtime
c. System maintenance
d. Personnel training
3. A validation test case that assesses the system’s ability to
recognize an erroneous input is called:
a. Normal
b. Boundary
c. Stress
d. Invalid
4. An example of interface software functionality is:
a. The entry of blood components into the blood bank
database
b. The transmission of patient information from the HIS
into the blood bank system
c. The printing of a workload report
d. Preventing access to the system by an unauthorized
user
5. Backup copies of the information system:
a. Can be used to restore the information system data and
software if the production system is damaged
b. Are used to maintain hardware components
c. Are performed once a month
d. Are created any time changes are made to the system
6. User passwords should be:
a. Shared with others
b. Kept confidential
c. Posted at each terminal
d. Never changed

7. Preventing the issue of an incompatible blood component


is an example of:
a. Inventory management
b. Utilization review
c. System security
d. Control function
8. Information is stored in a collection of many different
files called the:
a. Database
b. Configuration
c. Hardware
d. Disk drive
9. Application software communicates with this type of
software to retrieve data from the system disks:
a. Interface
b. Operating system
c. Security
d. Program
10. Validation testing for software should consider all of the
following items except:
a. Data entry methods
b. Control functions
c. Performance of testing in production database
d. Invalid data
11. Complete the truth table below for a negative antibody
screen using two screening cells (SCI, SCII) at the immediate
spin (IS), 37°C (37), and antihuman globulin

(AHG) phases

. 12. During validation testing, a computer user entered the

following results for an antibody screen test:

After the user verified the entries, the monitor displayed


the following message: “Invalid test results.” What
caused the error message to display?
a. An invalid entry was made in the check cells (CC)
column
b. The truth table was set up incorrectly
c. The interpretation does not correlate with the test
entries
d. The interface to the laboratory computer system is
down
13. The following test plan has been created to validate the
blood bank computer function used to update the status
of blood units that have been transfused. The test plan
contains each of the sections, lettered A through H, required
for a thorough test plan. Evaluate each section
and, using the list below, assign a name to each section
Section A
Description: This function is used to change the status of issued
units to a transfused status. All records pertaining to
the unit and patient will be updated:
Name of Section A. ________________________________
Section B
1. Preventing the assignment of transfused status to a quarantined
blood unit.
2. Preventing the assignment of transfused status to a blood
unit that has already been transfused.
3. Preventing the assignment of transfused status to a blood
component that has not been issued.
Name of Section B. ________________________________
Section C
1. The computer will beep and display **Unit Has Not Been
Issued** when a blood unit number that has not been issued
is entered.
2. The computer will beep and display **Unit Is in Quarantined
Status** when a blood component that is in quarantine
status is entered.
3. The computer will beep and display **Unit Has Been
Transfused** when a blood component that has already
been transfused is entered

4. The Transfusion History screen display will indicate the


patient has been transfused and will display the date of
the last transfusion.
5. Printed reports will indicate relevant units were assigned
transfused status.
Name of Section C. ________________________________
Section D
1. Attempt to assign transfused status to the following units:
a. Quarantined blood component
b. Selected (but not issued) blood component
c. Transfused blood component
2. Selection of units from issued inventory list
3. Manual entry of issued blood components
Name of Section D. ________________________________
Section E
1. Operator will input blood component information and
select blood components.
2. Operator will input selected patient information.
3. Blood bank computer will update the patient and unit
record.
Name of Section E. ________________________________
Section F
The following screen displays and printed reports will be
verified for accuracy:
Screen Displays Printed Reports
Patient Information Patient History Report
Unit Information Transfusion Listing
Unit History
Transfusion History
Name of Section F. _________________________________
Section G
The acceptability of the results of each test case will be determined
by the blood bank manager and documented on
the validation documentation form. If the software does not
perform as expected, the problem must be recorded on a
computer problem report and the supervisor alerted. A remedial
action plan will be devised with the assistance of the
blood bank computer system vendor.
Name of Section G. _________________________________
Section H
Blood bank director signature: ____ Date: _____________
Comments: _______________________________________
Name of Section H. _______________________________

Review Questions29
1. Transfusion-transmitted diseases can result in lawsuits
claiming:
a. Battery
b. Invasion of privacy
c. Negligence
d. a, b, and c
2. Laws applicable to blood banking and transfusion medicine
can arise:
a. In state and federal courts
b. In the U.S. Congress, state legislatures, and state and
federal courts
c. In state legislatures and courts
d. In state legislatures and the U.S. Congress
3. The reasons patients have sued for transfusion injury
include:
a. Failure to perform surrogate testing
b. Failure to properly test blood components
c. Failure to properly screen donors
d. All of the above
4. Blood banking professionals may increase the threat of
litigation by:
a. Following published regulations and guidelines
b. Knowing the legal bases for liability
c. Disclosing all information about patients and donors
d. Practicing good medicine

5. Issues about transfusion-transmitted diseases:


a. Are evolving and will continue to result in litigation in
the foreseeable future
b. Frequently result in plaintiff verdicts
c. Have all been litigated
d. Are known and avoidable

ANSWERS IN THE PDF

Clinical Laboratory Blood Banking and Transfusion Medicine Practices 1e 2015

Review Questions1
1. Match the term with its corresponding definition. (Objective
#1)
_____ A. Plasma cells
_____ B. Innate immunity
_____ C. Anamnestic
_____ D. Antigen
_____ E. Clonal selection theory
1. The immune response that occurs upon repeated
exposure to the same antigen.
2. The binding of antigen to lymphocytes, resulting in
activated cells that may result in clones.
3. Any substance that elicits an immune response.
4. A producer of soluble antibodies.
5. The body’s natural defense system.
6. Developing lymphocytes that are potentially self-reactive
eliminated prior to release into the bloodstream.
2. For a substance to act as an antigen and produce an
immune response, the substance must have which characteristics?
(Objective #5)
A. Foreign
B. Structurally complex
C. Agglutinate with an antibody
D. Chemical complexity
Chapter 1 • The Immune Proce ss: The Ori gin and Interacti on bet wee n Anti gens and Anti bodie s 23
3. The purpose of B lymphocytes is to: (Objective #5)
A. Secrete antibody
B. Engulf foreign antigens
C. Secrete antigen
D. Alter microorganisms in preparation for phagocytosis
4. What role do immunoglobulins play in the process of neutralization?
(Objective #5)
A. They are responsible for activating complement.
B. They prevent entry of foreigners into cells.
C. They bind to foreigners in preparation for digestion.
D. They trigger the production of plasma cells.
5. In terms of size, the largest class of immunoglobulins is:
(Objective #8)
A. IgM
B. IgG
C. IgE
D. IgD
E. IgA
6. This immunoglobulin class accompanies IgM by binding to
the surface of naive lymphocytes. (Objective #8)
A. IgA
B. IgD
C. IgE
D. IgG
7. Complement has been activated in a test. This activation
may be detected by: (Objective #20)
A. Agglutination at 98.7° F (37° C)
B. Activation of the membrane attack complex
C. Agglutination at room temperature
D. Hemolysis in the tube
8. Characteristics of IgM antibodies include: (Objective #8)
A. Able to cross the placental barrier
B. Good complement activator
C. Reacts best at cold temperatures (39.2–71.6° F [4–22° C])
D. Antigen is usually carbohydrate
9. Which of the following factors affects the first step of
agglutination (sensitization)? (Objective #12)
A. pH
B. Incubation time
C. Incubation temperature
D. Enhanced by addition of antihuman IgG
10. Which of the following represent noncovalent bonds
involved in step one of agglutination? (Objective #10)
A. Hydrogen bonds
B. Van der Waals forces
C. Hydrophilic bonds
D. Electrostatic bonds
11. Which of the following immune complex characteristics
applies to the affinity between antigen and antibody?
(Objective #6)
A. Monovalent binding
B. Multivalent binding
C. Force of attraction between single binding sites on
antibody with antigen
D. Sum of all attractive forces
12. Which of the following best describes the purpose of the
complement membrane attack complex? (Objective #20)
A. Required for lysis of red blood cells
B. C5 convertase
C. Major contributor to red blood cell damage is due to
C9
D. C6 and C7 bind to C5b, creating a rod through the
red blood cell membrane
13. Which statement best differentiates between passive and
adaptive immunity? (Objective #4)
A. The process of passive immunity occurs within the
body whereas adaptive immunity is transferred to the
body from an outside source.
B. Individuals are born with the ability to display passive
immunity but not adaptive immunity.
C. Individuals obtain passive immunity from outside
the body whereas in adaptive immunity the body
remembers first exposures to antigens without outside
assistance.
14. Which of the following factors affects the strength of antigen
and antibody reactions? (Objective #6)
A. Concentration of red blood cell suspension (antigen)
B. Incubation temperature
C. Number of antigen sites on the red blood cell
membrane
D. Immunoglobulin class
24 CHAPTER 1 • The Immune Proce ss: The Ori gin and Interacti on bet wee n Anti gens and Anti bodie s
17. Which of the following phrases differentiates sensitization
from lattice formation? (Objective #14)
A. Antibody bound to red blood cell antigen
B. Antibody bridges between red blood cells
C. Allows visualization of antigen–antibody reactions
D. Ionic strength of the test system
15. Characteristics of the law of mass action that affect antigen
and antibody reactions include: (Objective #9)
A. Concentration of antigen
B. Concentration of antibody
C. Valence of antibody
D. Antigen and antibody complementarity
16. Which of the following characteristics applies only to the
classical complement pathway? (Objective #18)
A. Shaped like six tulips in a bunch
B. Binds with variable region of an IgM molecule
C. Binds with two IgG molecules
D. C1qC1r2C1s2

Review Questions2
1. Match the characteristics with the appropriate cellular
division process: (Objective #1)
_____ S omatic cell division a. Mitosis
_____ Daughter cells are haploid b. Meiosis
_____ S exual reproduction
_____ Daughter cells have two
sets of chromosomes
2. A DNA nucleotide consists of: (Objective #4)
A. Deoxyribose sugar, phosphate group, four bases
B. Deoxyribose sugar, phosphate group, one of four
bases
C. Ribose sugar, potassium group, one of four bases
D. Ribose sugar, phosphate group, one of four bases
3. Most common blood group antigens are formed as a
result of: (Objective # 3)
A. Genetic mutations
B. Recombinations
C. Crossovers
D. S ingle-nucleotide substitutions
4. Select the characteristics that differentiate RNA from
DNA: (Objective #6)
A. RNA is single-stranded.
B. RNA contains a thymine base.
C. RNA contains a ribose sugar.
D. RNA is transcribed to mRNA.
5. Which of the following blood group phenotypes result
in a homozygous expression of the Jkb antigen?
(Objective #9)
A. Jk(a + b-)
B. Jk(a - b+)
C. Jk(a + b+)
D. Jk(a - b-)
6. The synthesis of a carbohydrate blood group antigen
occurs as a result of: (Objective #8)
A. A mutated protein that attaches a carbohydrate group
to the red blood cell membrane.
B. A direct expression of the gene on the red blood cell
surface.
C. An enzymatic reaction that causes a carbohydrate
molecule to attach to the red blood cell membrane.
D. Attachment of a red blood cell membrane associated
protein.
7. The most common inheritance pattern for blood group
antigens is: (Objective #7)
A. Autosomal dominant
B. Autosomal recessive
C. Autosomal co-dominant
D. S ex-linked
8. A maternal ABO blood group genotype is determined to
be AA and the paternal genotype is BO. What percentage
of their offspring is expected to be blood type A?
(
Objective #11)
A. 25%
B. 50%
C. 75%
D. 100%
9. An antibody to a high-incidence antigen is identified in a
patient. Which of the following donors could best serve as
a source of compatible blood? (Objective #13)
A. S ibling
B. Community donor
C. F riend
D. None of the above
10. The advantages of using polymerase chain reaction
(PCR) technology for DNA amplification include:
(Objective #14)
A. S mall sample size requirement
B. High sensitivity
C. High specificity
D. Enzymatic recognition of DNA sequence

Review Questions3
1. Select the characteristics that are associated with
antibodies
of the IgM class: (Objective #3)
A. Produced in the primary immune response
B. 10 antigen-binding sites
C. React best at room temperature
D. Can cross the placenta
2. Which of the following methods detects in vivo sensitization
of red blood cells with antibody? (Objective #5)
A. Indirect antiglobulin test
B. Direct antiglobulin test
C. Polyspecific antihuman globulin
D. Enzyme treatment
3. The DAT and IAT methods are similar in that they both:
(Objective #7)
A. Require the addition of check cells to confirm test validity
B. Detect in vitro sensitization of red blood cells with
antibody
C. Require enhancement media for maximal binding
D. Require the use of AHG reagent
4. In the antibody screening procedure, IgG antibody is
detected at the ___________ phase of testing. (Objective #5)
A. Immediate spin
B. AHG
C. Coombs control
D. Check cell
5. Upon the addition of check cells in the direct antiglobulin
test, no agglutination is seen in the test tube. The test is
interpreted as: (Objective #6)
A. Positive for IgG antibody
B. N egative for IgG antibody
C. Valid
D. Invalid
6. Nonspecific aggregation of red blood cells occurs at
98.6°F (37°C) with which of the following enhancement
reagents? (Objective #12)
A. LISS
B. Albumin
C. Ficin
D. PEG
7. Select the factors that can affect the AHG phase of the
antibody screen test: (Objective #9)
A. Ionic strength
B. Incubation time
C. Use of check cells
D. Temperature
8. The solid phase methodology for antigen–antibody
interactions
is based on which of the following principles?
(Objective #15)
A. Antibody binds to red blood cell antigens that
are coated to the bottom of microtiter plate test
wells.
B. Antibody and antigen form visible agglutination
complexes
in a test tube upon centrifugation.
C. Antigen–antibody complexes become trapped
in a microtubule filled with dextran acrylamide gel.
D. Antigen and antibody interactions in a sample are
detected and quantified.
9. False-positive reactions in the indirect antiglobulin test
include: (Objective #11)
A. Failure to add test plasma
B. Contaminated saline
C. Excessive centrifugation
D . N eutralized AHG reagent
10. When the body first encounters a foreign antigen, the
period of time when there is no detectable antibody is
referred to as: (Objective #3)
A. Lag phase
B. Log phase
C. Plateau phase
D. Decline phase
11. Choose the appropriate immunoglobulin:
(Objective #4)
1. _______ Crosses the placenta
2. _______ Pentamer structure
3. _______ Has two binding sites
4. _______ Has four subclasses
5. _______ Direct agglutinate
a. IgM
b. IgM
Chapter 3 • Blood Bank App licat ions of Antigen–Antibody React ions 51
12. In the gel test, a negative reaction is demonstrated by:
(Objective #15)
i. A single layer of cells at the top of the gel column
ii. Cells and agglutinates distributed through the column
iii. N o cells in the gel layer, hazy red background to the gel
iv. Pellet of cells at the bottom of the gel column
13. A DAT is performed on a newborn cord blood specimen.
A 2+ agglutination is observed upon the addition of AHG
reagent. What clinical situation is most likely occurring?
(Objective #6)
A. Hemolytic transfusion reaction
B. Hemolytic disease of the newborn
C. Autoimmune hemolytic anemia
D. Drug-induced anemia
14. Clinically significant antibodies are detected at which
phase(s) of testing? (Objective #14)
A. Immediate spin (IS) phase
B. Incubation phase
C. AHG phase
D. Coombs control phase
15. Which anticoagulant is preferred for DAT testing?
(Objective #5)
A. Ammonium oxalate
B. EDTA
C. Heparin
D . N o anticoagulant

Review Questions4
1. The immunodominant sugar for blood type B is _________.
(Objective #2)
A. N-acetyl-D-galactosamine
B. L-fucose
C. D-galactose
D. All of the above
2. The H gene codes for _________, which adds L-fucose to
a precursor carbohydrate chain to form the H antigen.
(Objective #2)
A. _@1,2@L@fucosyltransferase
B. _@1,3@N@acetyl@galactosaminyltransferase
C. _@1,3@D@galactosyl transferase
D. _@1,3@N@acetyl@glucosamine
3. The frequency of blood type AB in the general population
is: (Objective #5)
A. 10%
B. 7%
C. 4%
D. 1%
4. Weak subgroups of A can be identified through all of the
following except: (Objective #10)
A. Strength of reaction with anti-H lectin
B. Presence or absence of anti@A1
C. Strength of reaction with human source anti-A, anti-B,
and anti-A,B
D. Absence of Lewis substance in the saliva of secretors
5. Characteristics of antibodies of the ABO blood group
system
include all of the following except: (Objective #13)
A. Naturally occurring
B. Present at birth
C. Mainly IgM class
D. Activate complement
6. The genotype of a para-Bombay individual is:
(Objective #9)
A. hh, Sese
B. HH, Sese
C. hh, sese
D. Hh, Sese
7. The following reactions were observed in an ABO typing
test: (Objective #16)
Forward Group R everse Group
Anti-A Anti-B A1 cells B cells
0 4+ 0 0
Before reporting blood typing results, the laboratorian
should:
A. Perform the saline replacement test
B. Type the patient red blood cells with Dolichos biflorus
C. Wash patient red blood cells four times and repeat
D. Incubate reverse typing for 15 minutes at 39.2° F (4° C)
8. Causes of unexpected positive reactions in the forward
grouping include: (Objective #15)
A. Contamination with Wharton’s jelly
B. Presence of cold autoantibody
C. Subgroup of A
D. Age of the patient
9. Which of the following statements is true regarding
ABO-
incompatible hematopoietic progenitor cell (HPC)
transplants? (Objective #18)
A. Results in immediate engraftment of donor red blood
cells to the recipient
B. Causes various types of ABO discrepancies
C. Causes red blood cell hemolysis
D. Are usually fatal
10. The forward grouping test identifies which of the following
red blood cell antigens? (Objective #14)
A. A
B. B
C. A and B
D. A, B, and H
11. Which of the following statements is true regarding the
characteristics of the Bombay and para-Bombay phenotypes?
(Objective #7)
A. Occurs in 1% of the population and most frequently in
Indian and Japanese ethnicities.
B. Occurs in 1% of the population and least often in
Indian and Japanese ethnicities.
C. Occurs in 0.1% of the population and most frequently
in Indian and Chinese ethnicities.
D. Occurs in 0.1% of the population and most frequently
in Indian and Japanese ethnicities.
74 CHAPTER 4 • ABO and Hh Blood Group Systems
14. The B phenotype within the United States is:
(Objective #7)
A. More common in people of European and African
descent than those of Hispanic descent
B. More common in people of Hispanic and African
descent than those of Asian descent
C. More common in people of Asian and African descent
than those of European descent
D. More common in people of Europeon and Hispanic
descent than in those of African descent
15. If a cis-AB male and a group O female have a baby,
the baby’s phenotype could be which of the following?
(Objective #12)
A. A, B, O, or AB
B. A, B, or AB
C. A or B
D. AB or O
12. The genetic difference between the most common gene
responsible for group O and the A101 gene is:
(Objective #3)
A. Deletion of cytosine at position 261
B. Deletion of guanine at position 261
C. Addition of cytosine at position 261
D. Addition of guanine at position 261
13. Which of the following statements is true regarding polyagglutination?
(Objective #17)
A. Tx is a bacterially acquired and transient cause of
polyagglutination.
B. Tn is a bacterially acquired and transient cause of
polyaggluination.
C. Hemoglobin M-Hyde Park is an inherited and transient
cause of polyagglutination.
D. Hemoglobin S is an inherited and permanent cause of
polyagglutination.

Review Questions5
1. The RHD and RHCE genes are located on which chromosome?
(Objective #4)
A. 1
B. 5
C. 8
D. 10
2. The presence of the ______ antigen on the surface of the
red blood cell membrane designates a person as Rh positive.
(Objective #3)
A. Rh
B. D
C. d
D. CE
3. Which of the following genetic circumstances results in
the absence of the D antigen? (Objective #4)
A. Deletion of RHD
B. Mutated RHD alleles
C. Partial RHD alleles
D. Partial RHCE alleles
4. How do Rh proteins differ from ABO blood group antigens?
(Objective #3)
A. Rh proteins are not an integral part of the red blood
cell membrane.
B. Rh proteins are not present at birth.
C. Rh proteins are found in most body fluids.
D. Rh proteins do not contain a carbohydrate group.
5. The Rh null phenotype is associated with: (Objective #2)
A. Hemolytic anemia
B. The presence of stomatocytes on the peripheral blood
smear
C. Blood clotting abnormalities
D. Weak expression of Rh antigens
6. How is Rh-associated glycoprotein (RhAG) distinguished
from Rh protein? (Objective #12)
A. Their genes are found on different chromosomes.
B. Their blood group systems are different.
C. RhAG protein contains no Rh antigen.
D. Absence of RhAG protein does not result in RBC
membrane defects.
7. Cells that do not have the RhAG protein will also lack:
(Objective #5)
A. RHCE and RHD proteins
B. LW glycoproteins
C. CD47
D. Glycophorin B
8. Which blood group system is linked with the RH gene?
(Objective #4)
A. Duffy
B. Scianna
C. Kidd
D. Lewis
9. The terminology that describes the inheritance of the Rh
antigens through a single gene at a single locus was developed
by: (Objective #7)
A. Weiner
B. Fisher-Race
C. Rosenfield
D. Tippett
10. Matching. Convert Fisher-Race to Wiener nomenclature:
(Objective #8)
_______ dce/dce A. R1r
_______ DCe/DcE B. rr
_______ DCe/dce C. R1R2
_______ DcE/dce D. R2r
11. Which of the following phenotypes will react with anti-f?
(Objective #16)
A. DCe/DcE
B. dce/dce
C. DcE/dCE
D. Dce/Dce
12. What is the most probable genotype for a patient’s red
blood cells that demonstrates the following results with
reagent antisera? (Objective #19)
Antisera Result
D 3+
C 3+
E0
c0
e 3+
A. R2R2
B. R1R1
C. R1r
D. RzR1

13. A patient’s red blood cell phenotype is R1r . The Rosenfield


nomenclature should be interpreted as: (Objective #7)
A. Rh: -1, 2, -3, 4, 5
B. Rh: 1, -2, -3, 4, 5
C. Rh: 1, -2, 3, 4, 5
D. Rh: 1, 2, -3, 4, 5
14. Explanations for the weak D phenotype include which of
the following: (Objective #11)
A. Reduced number of antigen sites
B. Suppression of the RHD gene through a position
effect
C. Missing epitope
D. Inheritance of Rh null gene
15. Which of the following Rh antisera requires the use of a
negative control?(Objective #15)
A. Low-protein anti-D
B. High-protein anti-D
C. Monoclonal anti-D
D. Polyclonal anti-D
16. Match the sources of error in the Rh typing test: (Objective
#13)
________ Rouleaux A. False-positive reaction
________ RBC suspension too B. False-negative
heavy reaction
________ Failure to add antiserum
________ Autoagglutinins

17. Persons with the weak D phenotype are considered to be:


(Objective #10)
A. Rh positive
B. Rh negative
C. Rh null
D. Neither Rh positive or Rh negative
18. Select the characteristics of Rh blood group system antibodies.
(Objective #16)
A. Naturally occurring
B. Cross the placenta
C. Strongly immunogenic
D. Deteriorate rapidly
19. __________ is a substance that prevents the allogeneic
production of anti-D in pregnant women. (Objective #17)
A. Rh-immunoglobulin (RhIg)
B. Antihuman globulin (AHG)
C. Monoclonalglobulin (MHG)
D. 22% albumin
20. The high-incidence Rh antigen that is missing in an individual
who has a CE deletion is: (Objective #6)
A. e
B. Rh6
C. Rh18
D. F

Review Questions6
1. Select the antibodies that react optimally at room temperature:
(Objective #7)
A . A nti-M
B. A nti-Lea
C. A nti-I
D . A nti-S
2. Which of the following blood group system antibodies is
known to bind complement? (Objective #7)
A . D uffy
B. Kidd
C. R h
D . L utheran
3. Choose the Kell blood group system phenotype that is
associated with a homozygous expression of the Kell antigen:
(Objective #3)
A . (K+k-)
B. (K+k+)
C. (K-k+)
D . (K-k-)
4. Which of the following blood group antigens has the highest
frequency in the population? (Objective #4)
A . Fya
B. M
C. K
D.E
5. The Fy(a–b–) phenotype is most often found in which of
the following blood donor populations? (Objective #4)
A . E uropean
B. A sian
C. A frican
D . H ispanic
6. A technologist performed ficin treatment on a suspension
of red blood cells. Select the expected change in antigen
expression: (Objective #14)
_____ C
_____ Fyb
_____ Jka
_____ M
_____ K
A . D iminished
B. E nhanced
C. U naffected
7. Treatment of red blood cells with _____ can destroy Kell
system antigens. (Objective #15)
A . 0.2M DTT
B. P apain
C. A cid pH 6.5
D . L ISS
8. Select the substance that is NOT able to inhibit or neutralize
antibody reactivity: (Objective #15)
A . H uman saliva
B. P ooled human plasma
C. PE G
D . Guinea pig urine

9. Characteristics of clinically significant antibodies include:


(Objective #9)
A. Ability to cross the placenta
B. In vivo hemolysis of red blood cells
C. Resulting anemia in newborns
D. React optimally at room temperature
10. Chronic granulomatous disease (CGD) is associated with
which of the following phenotypes? (Objective #9)
A. En(a-)
B. McLeod
C. Rh null
D. Kell null
11. Three days after transfusion of packed red blood cells, a
patient experienced a decrease in hemoglobin and hematocrit
to pretransfusion levels. An antibody screen and DAT
were performed on a post-transfusion specimen and the
results were positive. Repeat testing of the pretransfusion
specimen confirmed that the antibody screen and DAT
were both negative prior to the transfusion. What is the
most likely explanation for these results? (Objective #10)
A. The antibody screening reagents were expired and
inert.
B. The post-transfusion specimen was mislabeled.
C. A low-titer antibody was present in the pretransfusion
specimen.
D. The technologist made a clerical error.

12. When an individual inherits both the Lewis (Le) gene and
the secretor gene (Se), which of the following red blood
cell phenotypes will be observed? (Objective #13)
A. Le(a+b-)
B. Le(a+b+)
C. Le(a-b+)
D. Le(a-b-)
13. Select the situations when the Le(a–b–) red blood cell phenotype
can be identified. (Objective #13)
A. Newborns
B. Elderly
C. Pregnancy
D. Inheritance of Le gene
14. The characteristics of the I antigen include: (Objective #16)
A. Straight chain structure
B. Present at birth
C. Reacts best at room temperature
D. Varied expression among adults
15. Which of the following characteristics describe the Sid
blood group antigen? (Objective #18)
A. Mainly room temperature reactive
B. Exhibit a mixed-field, shiny, and refractile appearance
C. Variable expression in the population
D. Highest quantity in urine

Review Questions7
1. Which of the following tests are mandatory to perform on
red blood cell units that have just been received from a
blood supplier? (Objective #5)
A . ABO typing
B . R h(D) typing
C. Weak D test
D . E xtended antigen phenotype
2. The process of obtaining informed consent for a blood
transfusion: (Objective #1)
A . P rovides the patient with a description of risks, benefits,
and viable treatment alternatives
B . I s specific to the patient’s condition and type of blood
product that will be transfused
C. R equires the opportunity for questions to be asked
D . R equires the right to refuse the transfusion

3. What information is needed when a blood component is


requested for transfusion to a patient? (Objective #2)
A. N ame of the ordering physician
B. Clinical information and patient history
C. N umber and type of component to be transfused
D. Donor name and contact information
4. A hemolytic transfusion reaction can be caused by:
(Objective #3)
A. Patient identification error at the time of transfusion
B. Product viability
C. Specimen labeling error at the time of collection
D. Reagent QC failure
5. Select the appropriate patient identifiers for specimen
labeling that should be checked for accuracy against the
request for transfusion. (Objective #4)
A. F ull name of patient
B. Date of birth
C. Medical/hospital record number
D . N umber sequence/blood bank numbers
6. Why should specimens be used no more than 3 days after
collection in patients who have been recently transfused
or who are pregnant? (Objective #3)
A. Delayed hemolytic transfusion reactions may not be
detected.
B. Recent pregnancy or transfusion may stimulate antibodies
that are not detectable at the time of collection.
C. Clinically significant antibodies may fall below detectable
levels after the transfusion.
D. Patient identification errors may not be identified.
7. Steps to reduce ABO blood typing errors in the laboratory
include: (Objective #5)
A. Compare current ABO typing results to a historical
record.
B. Perform two separate ABO typing tests on the same
specimen.
C. Perform ABO typing tests on two specimens drawn
independently.
D. Perform ABO typing tests on two specimens drawn at
the same time.
8. What type of red blood cells should be issued when an
ABO blood type cannot be determined prior to the
request for transfusion? (Objective #6)
A. G roup O
B . G roup O or Group A
C. G roup A or Group AB
D . G roup O, Group A, Group B, or Group AB
9. Select the requirements for the manufacture of screening
cells that are used in the antibody screening test for pretransfusion
testing. (Objective #5)
A. Must be blood group O
B. Must be from a single donor (not pooled)
C. Should contain at least one expression of all significant
antigens
D. Must contain a homozygous form of all antigens
10. Why is patient cell antigen typing, as part of the reflex
testing process, performed only on pretransfusion specimen?
(Objective #7)
A. Circulating patient cells may cause false-negative results
B. Circulating patient cells may stimulate antibody
production
C. Circulating donor cells may cause false-positive results
D. Circulating donor cells may be below detectable levels
11. ABO incompatibility of red blood cells for transfusion is
detected at the _____ phase of the major crossmatch test.
(Objective #12)
A. Immediate spin (I.S.)
B . 98.6° F (37° C)
C. IAT
D. Coombs control cells
12. Which of the following blood products does not require
any pretransfusion testing? (Objective #9)
A. Red blood cells
B . F resh frozen plasma
C. Single-donor platelets
D. Plasma protein fraction
13. Which pretransfusion test method uses antigen coated
wells on a microplate for the identification of clinically significant
antibodies? (Objective #10)
A. G el
B. Solid phase
C. Tube
D . PEG
14. A patient is scheduled for hip replacement surgery and a
pretransfusion work-up, including crossmatch for 2 units
of red blood cells, is requested. Although the patient has
a history of anti-Fya, the antibody screen in the current
sample is negative by gel method. What type of crossmatch
test should be performed? (Objective #12)
A. Immediate spin major crossmatch
B. Antiglobulin major crossmatch
C. Electronic crossmatch
D. Minor crossmatch

Review Questions8
1. The autocontrol tube in an antibody identification study
consists of: (Objective #5)
A. Panel cells and patient plasma
B. Screening cells and patient plasma
C. Patient cells and patient plasma
D. Check cells and patient plasma
2. Anti-K is identified in an antibody identification rule-out
procedure. The autocontrol tube on the panel study
shows negative results. What is the patient’s expected red
blood cell phenotype? (Objective #3)
A. (K+k-)
B. (K+k+)
C. (K-k+)
D. Both A and C
3. Enzyme treatment of red blood cells assists with the identification
of which of the following sets of antibodies?
(Objective #5)
A. Anti-M and anti-Fyb
B. Anti-C and anti-Jka
C. Anti-K and anti-S
D. Anti-Fya and anti-c
4. A blood bank technologist receives a request for 3 units of
packed red blood cells for a patient who has both Anti-Jkb
and Anti-K in the plasma. The antigen frequency of Jkb is
73% and that of K is 10%. How many units of blood will
need to be tested to find 3 compatible units? (Objective #6)
A. 3
B. 7
C. 10
D. 12
5. Select the antibody whose corresponding antigen is
destroyed by enzyme treatment: (Objective #8)
A. Anti-Fya
B. Anti-D
C. Anti-Jka
D. Anti-K
6. Statistical validity for the identification of anti-C on a panel
study is achieved through: (Objective #9)
A. One C+ cell with positive results and one C- cell with
negative results
B. One C+ cell with negative results and one C- cell
with positive results
C. Three C+ cells with positive results and three C- cells
with negative results
D. Three C+ cells with negative results and three Ccells
with positive results
7. Choose the antigen that is an exception to homozygous
exclusion of an antibody: (Objective #2)
A. Fyb
B. D
C. C
D. S
8. Mixed-field agglutination is commonly observed when:
(Objective #10)
A. Group O packed red blood cells are transfused to an
Group A patient
B. Anti-Sda is identified in the plasma
C. Patient has a delayed hemolytic transfusion reaction
D. All of the above

9. Match the neutralization substances with the source of


origin: (Objective #11)
1. _____1. I substance
2. _____2. Lewis substance
3. _____3. P1 substance
4. _____4. Sda substance
A. Saliva
B. Hydatid cyst fluid
C. Urine
D. Breast milk
10. Polyagglutinable red blood cells will react with which of
the following sera? (Objective #17)
A. Cord blood sera
B. Autologous sera
C. Human source reagent antisera
D. All of the above
11. Which elution method is recommended for removal of
antibodies from the red blood cell membrane in suspected
cases of hemolytic disease of the fetus and newborn
(HDFN) due to ABO incompatibility? (Objective #12)
A. Glycine pH 3.0
B. Freeze–thaw
C. Ether
D. Methylene chloride

12. A warm autoantibody is identified in a specimen from a


32-year-old male patient. The patient was transfused with
2 units of packed red blood cells 2 months ago. Which
type of procedure should be performed to rule out the
presence of alloantibodies? (Objective # 14)
A. Autoadsorption
B. Elution
C. Allogeneic adsorption
D. Plasma dilution
13. Select the substance that can be used to destroy IgM
class antibodies in plasma: (Objective #14)
A. Dithiothreitol
B. Chloroquine
C. Ficin
D. Sda + urine
14. A plasma specimen demonstrates 2+ agglutination
strength with 10 out of 10 panel cells in an antibody identification
study. The autocontrol tube is negative. The antibody
specificity is most likely: (Objective #2)
A. Anti-C
B. Anti-D
C. Anti-k (cellano)
D. Anti-Fyb
15. Warm autoantibodies are characterized by all of the following
except: (Objective #13)
A. Strong positive DAT
B. Positive plasma reactions with all panel cells
C. Optimal reactivity at 98.6° F (37° C)
D. Compatible crossmatch with all donor cells tested

Review Questions9
1. The blood donation process includes: (Objective #2)
A. Donor registration
B. Health history
C. Physical examination
D. Blood collection
2. According to regulations, the Donor History Questionnaire
(DHQ) form: (Objective #3)
A. May not be modified with respect to formatting
B. Is not modified when new diseases or viruses emerge
C. May not be modified or edited with respect to content
or wording
D. May not be modified to make screening criteria more
restrictive
3. Select the medications that are on the permanent deferral
list: (Objective #7)
A. Etretinate (tegison)
B. Proscar
C. Accutane
D. G rowth hormone of human origin
4. Match the donor history with the appropriate deferral
period: (Objective #7)
1. _____ Babesiosis 2 years ago A. Permanent deferral
2. _____ Treatment for malaria B. Temporary deferral
1 year ago
3. _____ Blood transfusion 5
months ago
4. _____ Sibling has Creutzfeldt-
Jakob disease
5. _____ L ived in United Kingdom
from 1991 until 1993

5. According to AABB standards, the physical examination


of an acceptable blood donor must include: (Objective #8)
A. Pulse between 50 and 100 bpm
B. Hemoglobin at least 10.5 g/dl
C. Systolic pressure less than or equal to 180 mmHg
D. Temperature less than or equal to 99.8° F (37.5° C)

7. Which of the following criteria describe a low-volume unit


collection? (Objective #10)
A. Amount of blood collected is 450 +/- 45 ml
B . N o clot must be present in the collection bag
C. Plasma must be removed and labeled for use within 6
hours of collection
D. Blood unit label states “quantity not sufficient”
8. Most donor reactions occur within a few _____ after the
donation process. (Objective #11)
A. Minutes
B. Hours
C. Days
D. Weeks

9. What is the deferral period after the donation of a single


plateletpheresis unit? (Objective #12)
A. 24 hours
B. 48 hours
C. 3 days
D. 7 days
10. Mr. C developed a hematoma immediately after donation
of a unit of whole blood. Which of the following steps in
the collection process may have caused this to occur?
(Objective #11)
A. The needle penetrated the distal part of the vein.
B. The needle was removed from the donor’s arm after
release of the tourniquet.
C. Adequate pressure was placed on the venipuncture
site after the needle was removed.
D. The needle insertion caused fluid to penetrate surrounding
tissue.
11. Which donor requirement distinguishes an autologous
donation from an allogeneic donation? (Objective #14)
A. Photo identification is not required
B. Time interval between donations may be shorter
C. All transmissible disease test results must be negative
D. Weight and age requirements may be waived
12. Choose the circumstances when a directed donation can
present a greater risk to the recipient: (Objective #15)
A. A man donates for his mother
B. Husband donates for his wife who is pregnant
C. Directed unit tests positive for a transmissible disease
D. Directed unit is crossed over to general inventory

13. Select the transmissible disease test that is not required to


be performed on allogeneic donor blood: (Objective #16)
A. WNV
B . HTLV
C. CMV
D. Syphilis
14. What procedure should be performed when a repeat
donor tests positive for an infectious disease marker
that was not present in the previous donation?
(Objective #19)
A. Confidential unit exclusion
B. Donor recheck
C. Review of Donor History Questionnaire (DHQ)
D. Donor lookback
15. Which unit modification reduces the exposure to CMV?
(Objective #18)
A. Irradiation
B . L eukoreduction
C. Fractionation
D. Centrifugation

Review Questions10
1. Which of the following components is not prepared from
whole blood? (Objective #3)
A. Red blood cells
B. Fresh frozen plasma
C. Cryoprecipitate
D. Platelets apheresis
2. One unit of cryoprecipitated AHF prepared from whole
blood contains a minimum of how many international units
(IU) of Factor VIII? (Objective #6)
A. 40
B. 80
C. 120
D. 160
3. ISBT 128 blood component labeling guidelines require all
of the following except: (Objective #10)
A. Donor identification number
B. Blood type
C. Product expiration date
D. Recipient name
4. Which of the following statements regarding blood storage
conditions is incorrect? (Objective #15)
A. CPDA-1 red blood cells are stored for 35 days at
33.8–42.8° F (1–6° C).
B. Platelets are stored for 5 days at 33.8–42.8° F (1–6° C)
with gentle agitation.
C. Cryoprecipitated AHF is stored for one year at -0.4° F
(-18° C).
D . G ranulocytes are stored for 24 hours at 68–75.2° F
(20–24° C) with no agitation.

5. The primary populations that benefit from aliquoting


blood products are: (Objective #18)
A. Fetuses and neonates
B. N eonatal and pediatric patients
C. Pediatric and adolescent patients
D. Adolescents and adults
6. The acceptable number of residual WBCs in a leukoreduced
blood product is no greater than: (Objective #18)
A. 5.0 * 106/L
B. 5.0 * 1010/L
C. 1.0 * 106/L
D. 1.0 * 1010/L
7. To prevent engraftment of donor lymphocytes, cellular
blood products should be: (Objective #18)
A. Washed
B. Volume-reduced
C. Irradiated
D. Pooled
8. The first protein to be isolated and fractionated from
human plasma was: (Objective #22)
A. Albumin
B. Immune globulin
C. Coagulation protein
D. Prothrombin complex
9. Components that may be collected by apheresis include:
(Objective #9)
A. Red blood cells
B. Platelets
C. G ranulocytes
D. All of the above
10. The most important consideration for the transport of
blood components between sites is: (Objective #16)
A. Use of disposable devices
B. Shipping temperature
C. Shipping container
D. Insulation of the transport vehicle
11. Which of the following preservative constituents serves to
maintain ATP levels during blood storage? (Objective #2)
A. Citric acid
B. Dextrose
C. Adenine
D. Sodium phosphate

12. Which of the following production methods for preparing


platelet components from whole blood is recommended
for pediatric transfusion? (Objective #7)
A. Buffy coat (B/C)
B. Platelet-rich plasma (PRP)
C. Soft spin
D. Hard spin
13. Biochemical changes that occur during blood storage
include: (Objective #13)
A. Increase in plasma hemoglobin level
B. Increase in pH level
C. Increase in ATP level
D . I ncrease in 2,3-DPG level
14. How much residual plasma remains in a component of
cryoprecipitated AHF that is prepared from whole blood?
(Objective #5)
A. 100 mL
B. 70 mL
C. 35 mL
D. 15 mL
15. Separation of blood components during apheresis is
achieved through: (Objective #8)
A. Washing
B. Aliquoting
C. Centrifugation
D. Dilution
16. Decreased levels of _______ during blood storage results
in impaired oxygen release by red blood cells after the
blood is transfused. (Objective #13)
A. Potassium
B. Sodium
C. 2,3-DPG
D. Hemoglobin F
17. The blood component expiration date must always be
changed to 24 hours when using which of the following
blood component modification procedures? (Objective #20)
A. Aliquoting red blood cells using a sterile connecting
device
B. Washing red blood cells in an open system
C. Pooling platelets
D. Leukoreduction of whole blood

Review Questions11
1. Select the appropriate blood product for each clinical situation:
(Objective #3)
____Patient with mucosal bleeding,
platelet count 5 * 103/ul.
____History of anemia, Hgb 6.0 g/dL
____Bleeding, fibrinogen level 50mg/dL
____Neutropenic patient with sepsis, unresponsive to
antibiotics
____Trauma patient, elevated PT and aPTT, INR = 2.5
A. Fresh frozen plasma
B. Platelets
C. Red blood cells
D. Granulocytes
E. Cryoprecipitate
2. In order for a blood product to be designated as leukoreduced,
the product must have less than ______________
white blood cells per unit. (Objective #8)
A. 1.0 * 1010
B. 5.5 * 1010
C. 5.0 * 106
D. 3.0 * 1011
3. The use of a blood warming device for transfusion of
blood components is beneficial in which of the following
situations? (Objective #4)
A. Trauma patient, massive transfusion
B. Patient with cold agglutinin disease
C. Blood product infusion through a central venous
catheter
D. Pediatric patient, packed red blood cell transfusion
4. Select the substances that will not have an adverse
effect on blood components when they are infused during
a blood transfusion. (Objective #11)
A. ____Penicillin
B. ____0.9% Sodium chloride
C. ____5% Albumin
D. ____Dextrose
E. ____Ringer’s lactate
F. ____ABO-compatible plasma
5. Contraindications to transfusion include: (Objective #5)
A. RBC transfusion for a mild iron deficiency
B. Cryoprecipitate for fibrinogen deficiency
C. Granulocyte transfusion for an infection responsive to
antibiotics
D. FFP for volume expansion
6. A patient donates two units of autologous blood 3 weeks
prior to a scheduled surgical procedure. This type of
donation is called: (Objective #12)
A. Double apheresis
B. Hemodilution
C. Preoperative donation
D. Postoperative salvage
7. Select the statements that are associated with the
process of acute normovolemic hemodilution (ANH):
(Objective #13)
A. Blood units should be reinfused in the same order
they were collected.
B. Transfusions are typically administered by operating
room staff.
C. The first donor unit collected will likely contain the
largest amount of fresh coagulation factors.
D. The last unit reinfused is least likely to stop bleeding
problems as a result of low platelet count.

8. Choose the types of directed donor blood units that


require irradiation prior to transfusion: (Objective #7)
A. Nick C. donates a unit of packed red blood cells to his
friend who is a cardiac patient.
B. Mary L. donates a unit of packed red blood cells to
her mother who is scheduled for hip surgery in 2
weeks.
C. Joan Y. donates a unit of packed red blood cells to
her husband John who is scheduled for a routine
procedure.
D. Anthony M. donates a unit of packed red blood cells
to his mother-in-law who is having knee-replacement
surgery.
9. Specific requirements for transfusion of a unit of granulocytes
include: (Objective #8)
A. HLA compatibility
B. Leukofiltration
C. Irradiation
D. Transfusion within 24 hours of collection
10. Which of the following statements is true regarding the
benefits and risks of peripheral blood stem cell (PBSC) collection
and bone marrow collection? (Objective # 17)
A. PBSC requires insertion of a catheter, which reduces
risk of infection.
B. Bone marrow collection has a shorter engraftment
period than PBSC.
C. Bone marrow collection carries a lower risk for the
donor.
D. PBSC carries a higher incidence of graft versus host
disease (GVHD ).
11. Select the patients who are at risk for transfusion-transmitted
CMV. (Objective #9)
A. Adult patient with agammaglobulinemia
B. Premature newborn
C. Fetus with intrauterine transfusion
D. Bone marrow transplant recipient

12. Choose the expiration date for each modified blood


product:
(Objective #8)
_____ Washed red blood cells (open system)
_____ Frozen, deglycerolized red blood cells (open system)
_____ Directed donor red blood cells, irradiated
_____ CPD red blood cells, leukoreduced
A. 24 hours
B. 21 days
C. 28 days
13. The appropriate blood product for a patient with mild
hemophilia is: (Objective #3)
A. Red blood cells
B. Fresh frozen plasma
C. Cryoprecipitate
D. Platelets
14. Prevention of Rh immunization is achieved through transfusion
of which of the following products? (Objective #15)
A. RhIg
B. Plateletpheresis
C. 22% Albumin
D . IVI G
15. Complications of massive transfusion often include:
(Objective #10)
A. Depletion of coagulation factors
B. High incidence of transfusion reaction
C. Anticoagulant toxicity
D. Hyperglobulinemia

Review Questions12
1. A serious acute hemolytic transfusion reaction can occur as
a result of which of the following situations? (Objective #2)
A. Packed red blood cell product labeled as blood type B
transfused to a group O patient
B. Error in patient identification at the time of specimen
draw for pretransfusion testing
C. Misidentification of the patient at the bedside at the
time of transfusion
D. Computer entry error during reporting of pretransfusion
test result
2. From the following list, select the signs and symptoms of
an acute hemolytic transfusion reaction. (Objective #2)
A. Hemoglobinuria
B. DIC
C. Increased renal blood flow
D. Hypotension
E. Negative DAT
F. Pain at site of infusion
3. A full written report of a transfusion-associated fatality
must be reported to the Centre for Biologics Evaluation
and Research (CBER) within _____ days of the incident.
(Objective #3)
A. 3
B. 7
C. 10
D. 14
4. Patient John Smith is receiving a transfusion of 1 unit of
red blood cells for an anemic condition. Upon taking vital
signs, his nurse suspects he might be having an adverse
reaction to the transfusion. What is her first course of
action? (Objective #7)
A. Elevate the patient to a sitting position
B. Inform his next of kin
C. Administer IV furosemide STAT
D. Discontinue the transfusion
5. Select the mode of prevention for each of the adverse
transfusion reactions: (Objective #5)
_____ Febrile
_____ Allergic
_____ Transfusion-associated GVHD
_____ TACO
_____ Anaphylactic
_____ TRALI
(a) Washed red blood cells
(b) Transfusion of small aliquots
(c) Leukoreduced red blood cells
(d) Pretransfusion administration of antihistimine
(e) Transfusion of plasma from male donors
(f) Irradiation of blood products
6. Choose two specific mechanisms of a FNHTR. (Objective #5)
A. HLA antibodies
B. IgA antibodies
C. Release of cytokine substances
D. Bradykinin production
7. How can anaphylactic and anaphylactoid transfusion reactions
be distinguished from other reactions with a similar
clinical presentation? (Objective #8)
A. Absence of IgA
B. Absence of fever
C. Presence of respiratory distress
D. Shock
8. Which of the following clinical findings best differentiates
TRALI from TACO? (Objective #8)
A. X-ray evidence of pulmonary edema
B. Brain natriuretic peptide (BNP) level
C. Central venous pressure (CVP) level
D. Presence of hypoxia
9. Select the transfusion recipients who are at increased risk
for TACO. (Objective #5)
A. Immunocomprimised patient
B. Elderly patient
C. Cardiac patient
D. Anemic patient
E. Leukemic patient
F. Newborn infant
G. Septic patient
H. Trauma patient
10. Which blood product is most commonly implicated in
hypotensive transfusion reactions? (Objective #5)
A. Platelets
B. Whole blood
C. Packed red blood cells
D. Fresh frozen plasma

11. How does gamma irradiation of blood products prevent


TA-GVHD? (Objective #5)
A. It reduces the number of donor lymphocytes in the
transfused product.
B. It destroys non-nucleated cells such as red blood cells
and platelets.
C. It inhibits the proliferative capability of donor
lymphocytes.
D. It increases HLA compatibility between donor and
recipient.
12. Hemosiderosis is caused by excessive exposure to _____.
(Objective #5)
A. Citrate
B. Calcium
C. Potassium
D. Iron

13. Select the laboratory findings that aid in the differential


diagnosis of PTP. (Objective #5)
A. Presence of platelet-specific antibody
B. Negative D-Dimer
C. Abnormal bone marrow biopsy
D. Positive blood culture
14. What is the main consequence of elevated citrate levels in
transfusion recipients? (Objective #5)
A. Hypocalcemia
B. Hypoproteinemia
C. Hypokalemia
D. Hypomagnesium
15. What is the first step in the initial blood bank laboratory
investigation of a suspected transfusion reaction? (Objective
#7)
A. Repeat ABO testing
B. Visual inspection for hemolysis
C. DAT
D. Clerical check

Review Questions13
1. Which of the following transmissible disease tests is required
to be performed on all donated blood? (Objective #1)
A. CMV antibody
B. HCV antibody
C. Sickle cell screen
D. Toxoplasma antibody
2. How are the hepatitis B and hepatitis C viruses similar?
(Objective #2)
A. Both are DNA viruses.
B. Neither virus contains an envelope.
C. Neither virus has a chronic carrier state.
D. Both viruses are transmitted parenterally.
3. The first marker to appear in a hepatitis B infection is:
(Objective #4)
A. Anti-HBc
B. Anti-HBe
C. HBsAg
D. HevAg
4. Which transfusion-transmitted parasitic infection is caused
by a tick bite? (Objective #14)
A. Chagas disease
B. Malaria
C. Babesiosis
D. Leishmania
5. Which two hepatitis viruses are transmitted through contaminated
food products? (Objective #2)
A. Hepatitis A and E
B. Hepatitis B and D
C. Hepatitis C and E
D. Hepatitis B and C
6. Which of the following hepatitis viruses has the highest
incidence of transfusion transmission in the United States?
(Objective #3)
A. Hepatitis A
B. Hepatitis B
C. Hepatitis C
D. Hepatitis D
7. All of the following are retroviruses except: (Objective #5)
A. HIV-1
B. HTLV-1
C. HIV-2
D. HBV
8. The etiologic agent for syphilis is: (Objective #13)
A. Borrelia burgdorferi
B. Treponema pallidum
C. Plasmodium vivax
D. Babesia microti
9. Approximately 80% of humans that become infected with
_____ remain asymptomatic. (Objective #8)
A. HTLV
B. Parvovirus
C. Epstein-Barr virus
D. West Nile virus
10. How are CJD and vCJD similar? (Objective #11)
A. Both are transfusion transmitted.
B. Their latent periods are identical.
C. They present with identical symptoms.
D. Their median age at onset of symptoms is equivalent.

11. Transfusion-transmitted sepsis occurs most frequently


after the transfusion of which type of blood product?
(Objective #12)
A. Leukoreduced red blood cells
B. F resh frozen plasma
C. Cryoprecipitate
D. Platelet pheresis
12. Select the hepatitis virus that is associated with the highest
chronic carrier rate: (Objective #2)
A. Hepatitis A
B. Hepatitis B
C. Hepatitis C
D. Hepatitis D
13. Infectivity of leukocytes is highest in which three infectious
agents? (Objective #15)
A. EBV
B. CJD
C. CMV
D. HAV

14. Select the correct etiologic agent for each disease:


(
Objective #2, 11, 13, 14)
_____Malaria
_____variant CJD
_____Lymes disease
_____Hepatitis C
A. Parasite
B. Virus
C. Bacteria
D. Prion
15. Which organism causes Chagas disease? (Objective #14)
A. Treponema pallidum
B. Borrelia burgdorferi
C. Plasmodium vivax
D. Trypanosoma cruzi

Review Questions14
1. Select the conditions that are necessary for production of
maternal antibody in HDFN. (Objective #1)
A . A ntibody must be IgG in class.
B. A ntibody must not cross the placenta.
C. Mother must possess the corresponding antigen that
is present on fetal red blood cells.
D . F etal antigen must be developed before birth.
2. Choose the class of immunoglobulin that is implicated in
HDFN. (Objective #2)
A . I gA
B. I gM
C. I gG
D . I Ge

3. Which of the following is not characteristic of HDFN due


to ABO incompatibility? (Objective #3)
A. Occurs exclusively in group O mothers
B. Causes severe hemolysis
C. Antibody specificity is anti-A or anti-B
D. Can occur in the first pregnancy
4. Excessive levels of unconjugated bilirubin in the newborn
cause a condition known as: (Objective #3)
A. Hydrops fetalis
B. Cerebral palsy
C. Seizure
D. Kernicterus
5. Choose the antigenic determinants that affect the severity
of HDFN: (Objective #3)
A. Late development of fetal antigen
B. Density of fetal antigen
C. Structure of fetal antigen
D. Presence of tissue antigens
6. Which of the following could be a harmful consequence of
the elimination of the weak D test in serological studies on
maternal blood specimen? (Objective #6)
A. Production of anti@Rh0(D) in a mother with a rare partial
D phenotype
B. Development of multiple Rh alloantibodies
C. Administration of RhIg to an Rh0(D)@positive mother
D. Reduced potency of anti@Rh0(D) reagent
7. A low-risk method of predicting the risk of HDFN is:
(Objective #7)
A. Fetal blood phenotyping
B. Amniocentesis
C. Paternal antigen typing
D. Cordocentesis
8. Which type of measurement is considered least invasive in
monitoring the presence of anemia in HDFN? (Objective
#8)
A. Doppler
B. Amniocentesis
C. Percutaneous umbilical blood sampling (PUBS)
D. Cordocentesis

9. Select two methods that may be used in the antepartum


treatment of HDFN: (Objective #9)
A. Amniocentesis
B. Intrauterine transfusion
C. Whole blood exchange transfusion
D. Plasma exchange transfusion
10. The results of a prenatal work-up on a 32-year-old female
who is 26 weeks pregnant reveal:
Blood type A, Rh+
Antibody screen = positive, with anti-K identified
An intrauterine transfusion is requested for confirmed
fetal anemia. What type of blood should be chosen for
the transfusion? (Objective #18)
A. Blood type A, Rh0(D) positive; Kell antigen negative
B. Blood type A, Rh0(D) negative; Kell antigen positive
C. Blood type O, Rh0(D); Kell antigen positive
D. Blood type O, Rh0(D) negative; Kell antigen negative

11. Match the indications for Rh-immune globulin (RhIg) administration:


(Objective #10)
12. When an Rh0(D)@negative mother gives birth to an Rh0(D)
positive infant, RhIg should be administered within _____
of delivery. (Objective #10)
A. 24 hours
B. 36 hours
C. 48 hours
D. 72 hours
13. Results of a Kleihauer-Betke test reveal 3.2% fetal cells
in the maternal circulation. How many doses of RhIg
should be administered if the maternal blood volume is
5,000 mL? (Objective #11)
A. 4
B. 5
C. 6
D. 7

14. A mother is blood type O, Rh0(D) negative. Her antibody


screen is positive and anti-C is identified in the plasma.
Cord blood results on her newborn baby boy reveal that
the blood type is A, Rh0(D) positive, DAT is negative.
Select the true statements. (Objective #10)
A. The mother is a candidate for RhIg.
B. The mother is not a candidate for RhIg.
C. The baby is suffering from HDFN due to anti-C.
D. The baby does not have HDFN

15. Which of the following is a treatment that is typically used


for a mild case of HDFN? (Objective #3)
A. Exchange transfusion
B. Cordocentesis
C. Phototherapy
D. ECMO

Review Questions15
1. What is the basic principle of the adsorption test?
(Objective #1)
A. To remove antigens from red blood cells
B. To separate plasma from red blood cells
C. To remove antibodies from red blood cells
D. To separate antibodies from plasma
2. A specimen from a male patient with suspected autoimmune
hemolytic anemia (AIHA) is sent to the blood bank
for testing. The patient medical history reveals that he was
transfused 3 weeks ago at a different facility. The blood
bank work-up reveals a positive DAT, positive antibody
screen, and a panagglutinin antibody in both the plasma
and eluate. Which test should the laboratorian perform
next? (Objective #1)
A. Autoadsorption
B. Alloadsorption
C. Antigen adsorption
D. Reticular adsorption
3. What techniques may be used to perform antigen phenotyping
on a specimen from a recently transfused patient?
(Objective #1)
A. Isolation of endogenous reticulocytes
B. Molecular techniques
C. Adsorption of autoantibody from patient red blood
cells
D. All of the above
4. Which laboratory procedure is used to confirm paroxysmal
cold hemoglobinuria (PCH)? (Objective #2)
A. Allogeneic adsorption
B. Acid elution
C. Donath-Lansteiner test
D. Endogenous reticulocytes
5. A specimen from a 50-year-old female with a history of
anemia reveals a positive DAT. The patient reports she has
not been recently transfused or pregnant. The most likely
explanation for the positive test is: (Objective #3)
A. Autoimmune hemolytic anemia
B. Hemolytic transfusion reaction
C. Delayed serologic reaction
D. Hemolytic disease of the newborn
6. True or false: All patients with a positive DAT are anemic.
(Objective #3)
7. Which of the following circumstances can result in a falsepositive
DAT? (Objective #4)
A. L ow-titer IgG antibody is present
B. Prolonged storage of specimen prior to testing
C. Complete destruction of antibody has occurred
D. Loss of complement-regulating protein
8. Choose all of the laboratory findings that are indicative of
red blood cell hemolysis. (Objective #5)
A. _____ Elevated reticulocyte count
B. _____ Elevated haptoglobin
C. _____ Elevated LDH
D. _____ Elevated hemoglobin
E. _____ Icteric plasma
F. _____ Schistocytes
9. Which type of autoimmune hemolytic anemia is associated
with Mycoplasma pneumoniae? (Objective #6)
A. Drug-induced immune-mediated hemolytic anemia
B. Paroxysmal cold hemoglobinuria
C. Cold agglutinin syndrome
D. Warm autoimmune hemolytic anemia
10. Antibody reactivity in warm autoimmune hemolytic
anemia (WAIHA) is best described as: (Objective #6)
A. Alloagglutinin
B. Panagglutinin
C. Low frequency
D . N egative
11. Which type of hemolytic anemia would most likely have a
nonreactive (negative) eluate? (Objective #6)
A. Warm autoimmune hemolytic anemia
B. Mixed autoimmune hemolytic anemia
C. Cold agglutinin syndrome
D. Delayed hemolytic transfusion reaction
12. Match each type of hemolytic anemia with the expectedDAT results. (Objective #6)

13. A specimen from a patient with cold agglutinin syndrome


is received in the hematology lab for CBC analysis. In
order to obtain an accurate set of results on an automated
analyzer, what step should be taken prior to testing?
(Objective # 6)
A. Dilution of blood specimen with isotonic saline
B. Determination of plasma hemoglobin value
C. Removal of buffy coat
D. Prewarm the specimen to 98.6° F (37° C)
14. What is the first course of treatment for a patient with a
confirmed case of drug-induced immune hemolytic anemia?
(Objective #6)
A. Corticosteroids
B. Transfusion with drug-negative red blood cells
C. Alternative medication
D. Phototherapy

15. What is the most important concern when a panreactive


autoantibody is identified in the blood bank laboratory?
(Objective #10)
A. ABO incompatibility
B. Masking of alloantibodies
C. In vitro hemolysis
D. Antigen phenotyping

Review Questions16
1. A patient is considered to be responsive to platelet therapy
if the immediate post-transfusion corrected count
increment (CCI) is at least: (Objective #2)
A . 5,000
B. 7,500
C. 10,000
D. 12,000
2. GC was transfused with 1 unit of apheresis platelets. The
immediate post-transfusion corrected count increment
(CCI) was within the normal range; however, decreased
platelet survival was noted 24 hours post-transfusion.
What could be a possible explanation for this finding?
(Objective #2)
A . ITP
B. HLA antibody
C. PTP
D. DIC
3. Use the following information to calculate the CCI for a
50-year-old female patient: (Objective #3)
BsA = 1.7m2
# of platelets transfused = 4.0 * 1011
Pretransfusion platelet count = 5.0 * 103/_l (5.0 * 109/l)
Post-transfusion platelet count at
60 minutes = 45.0 * 103/_l (45.0 * 109/l)
A . 2,125
B. 12,500
C. 17,000
D. 19,125
4. The corrected count increment (CCI) calculation is based
on which of the following assumptions? Select all that
apply. (Objective #2)
A . A ccurate report of circulating blood volume
B. S plenic sequestration is in proportion to spleen size
C. A ccurate report of the number of platelets transfused
D. I nitial calculation is performed within 5 minutes of
transfusion
5. Select two medications that are known to cause thrombocytopenia
after administration to the patient: (Objective #5)
A . A cetaminophen
B. H eparin
C. Keflex
D. Digoxin

6. Select the condition that causes platelet refractoriness


through the mechanism of platelet consumption: (Objective
#6)
A. NAIT
B. PTP
C. HPA
D. DIC
7. Treatments for neonatal alloimmune thrombocytopenia
(NAIT) typically include all of the following except: (Objective
#10)
A. Intrauterine platelet transfusion
B. Whole blood exchange transfusion
C. Postnatal platelet transfusion
D. IVIG
8. Under normal circumstances, what percentage of total
body platelets is sequestered in the spleen? (Objective
#13)
A. 10%
B. 15%
C. 30%
D. 45%
9. Which of the following is not a clinical finding in heparininduced
thrombocytopenia type II? (Objective #7)
A. Decreased platelet count
B. IgG antibody production
C. Abnormal bleeding
D. Complement activation
10. Alloantibody specificity in immune-mediated platelet
refractoriness may include all of the following except:
(Objective #6)
A. GPB
B. ABO
C. HPA
D. HLA

11. The severity of ABO antibody-related platelet refractoriness


is highly dependent on which two factors? (Objective #8)
A. Antibody titer
B. GP receptor
C. Antigen dose
D. Avidity
12. Which condition is known to cause nonimmune platelet
refractoriness? (Objective #6)
A. ITP
B. DIC
C. NAIT
D. PTP
13. Which procedure can be used as an alternative to finding
HLA-matched platelets for a refractory patient? (Objective
#11)
A. Platelet crossmatch
B. Antibody identification
C. Molecular phenotyping
D. Cytotoxicity
14. Idiopathic thrombocytopenia purpura (ITP) can occur as a
secondary condition to which of the following diseases?
(Objective #10)
A. Bernard-Soulier
B. Hemophilia A
C. Systemic lupus erythematosus
D. bacterial infection
15. Post-transfusion purpura (PTP) occurs most often in:
(Objective #10)
A. Newborns
B. Multiparous patients
C. Males less than age 50
D. Elderly patients

Review Questions17
1. How are hemophilia A and hemophilia B similar? Select all
that apply. (Objective #1)
A. Genetic disorder
B. Pattern of inheritance is autosomal
C. Occur mainly in males
D . D eficient protein is Factor VIII
2. A mild case of hemophilia A correlates with a _____ Factor
VIII activity level. (Objective #2)
A. 61%
B. 1–5%
C. 5–20%
D. 30–50%
3. What is the half-life of Factor VIII? (Objective #3)
A. 2 hours
B. 12 hours
C. 24 hours
D. 48 hours
4. Disease manifestations in patients with sickle cell disease
include: (Objective #6)
A. Organ damage
B. Bone necrosis
C. Stroke
D. Pulmonary hypertension
5. A physician orders an exchange transfusion for a patient
with sickle cell disease. What are the anticipated benefits
of this type of transfusion? (Objective #7)
A. Decreased hemoglobin level
B. Decreased percentage of sickle cells
C. Increased blood volume
D. Increased blood viscosity
6. Blood transfusion requirements for a patient with sickle
cell disease typically include: (Objective #5)
A. HbS negative
B. Leukoreduced
C. CMV positive
D. Irradiated
7. Which of the following techniques reduces the risk of hyperhemolysis
in sickle cell anemia patients? (Objective #7)
A. Irradiation
B. Leukoreduction
C. Phenotype-matched RBCs
D. Iron chelation
8. Allogeneic hematopoietic progenitor cell (HPC) transplants
require donor and recipient to be: (Objective #8)
A. ABO identical
B. ABO compatible
C. HLA compatible
D . N one of the above
9. How is transfusion-transmitted CMV prevented? (Objective
#14)
A. Serologically tested CMV-negative product
B. Leukoreduction of RBC product
C. Irradiation of RBC product
D. Deferral of CMV-positive donors
10. Transfusion-associated graft versus host disease
(TA-GVHD) is preventable with which of the following
methods? (Objective #14)
A. Leukoreduction
B. Irradiation
C. Washing
D. High-speed centrifugation
11. Red blood cell hemolysis that occurs approximately 7–14
days after transplant of an organ with minor ABO incompatibility
is termed: (Objective #12)
A. Lymphocytotoxicity
B. Hyperacute rejection
C. Passenger lymphocyte syndrome
D. HLA sensitization
12. Complications of liver disease include: (Objective #13)
A. Abnormal bleeding
B. Depletion of vitamin K–dependent factors
C. Decreased fibrinogen
D. Low platelet count

13. Which coagulation factor has the shortest half life?


(Objective #13)
A. F actor II
B. F actor V
C. F actor VII
D . F actor IX
14. The process that is used to remove circulating antibody
that is causing disease is called: (Objective #9)
A. Dialysis
B. Plasmapheresis
C. Hemodilution
D. Irradiation

15. Select the diseases in which plasma exchange therapy is


often indicated as a course of treatment. (Objective #9)
A. Glanzmann’s thrombasthemia
B. Goodpasture’s syndrome
C. von Willebrand’s disease
D. Myasthenia gravis

Review Questions18
1. Which term accurately describes the rapid increase in antibody
production that occurs when a previously encountered
antigen is reintroduced? (Objective #1)
A. Transfusion
B . Anamnestic
C. I nnate
D. Foreign
2. Select the antigen-presenting cells that are involved in
adaptive immunity: (Objective #1)
A. M acrophages
B . Dendritic cells
C. T lymphocytes
D. N eutrophils
3. What types of molecules make up the major histocompatibility
complex (MHC)? (Objective #2)
A. HLA class I
B . HLA class II
C. HLA class III
D. HLA class IV
4. The HLA domain with the highest degree of polymorphism
is: (Objective #3)
A. HLA class I
B . HLA class II _1
C. HLA class II _1
D. HLA class III
5. Choose the appropriate designation for class II region
molecules. (Objective #5)
A. HLA-A, HLA-B, HLA-C
B . HLA-DP, HLA-DQ, HLA-DR
C. HLA-F, HLA-G
D. MI C-C, MI C-D, MI C-E
6. How many HLA genes are typically expressed in humans?
(Objective #5)
A. 3
B.4
C. 5
D. 6
7. HLA antibodies that are specific for an epitope found on
two or more antigens are designated as: (Objective #7)
A. Polymorphic
B . Polygenic
C. Cross-reactive
D. M utagenic
8. Assign the correct order of the number sets in accordance
with the 2010 HLA gene-naming convention criteria.
(Objective #7)
_____ Silent mutation
_____ Allele family
_____ Noncoding, intron areas
_____ Order of allele discovery

9. When only one HLA allele is identified at a particular locus


in a commercial HLA-typing procedure, the other allele is
designated as: (Objective #9)
A. Amorph
B. Silent
C. Blank
D. Mutagene
10. Which of the following is not a DNA-typing method for
HLA antigen identification? (Objective #9)
A. Sequence-specific oligonucleotide
B. Microlymphocytotoxicity
C. Sequence-specific primer
D. Sequence-based typing
11. Select the HLA typing method that determines the nucleotide
sequence of an entire exon, including the regions
that are usually constant: (Objective #9)
A. Sequence-based typing
B. Sequence-specific primer
C. Microlymphotoxicity
D. Sequence-specific oligonucleotide
12. What is the interpretation of a microlymphocytotoxicity
crossmatching test if lysis of donor lymphocyte occurs at
the end of the procedure? (Objective #9)
A. HLA antibody is present.
B. HLA antibody is not present.
C. The test is invalid.
D. None of the above
13. Which substance, when produced post transplant, is a
predictor of acute or chronic rejection? (Objective #10)
A. HLA antigen
B. RBC antibody
C. Cytokine
D. Albumin
14. Which of the following consequences can occur as a result
of an HLA mismatch in a human progenitor cell (HPC)
transplant? (Objective #10)
A. GVHD
B. Wiskott-Aldrich syndrome
C. CMV
D. HTLV
15. A positive HLA antibody screen test result is typically
reported as: (Objective #9)
A. Panagglutinin
B. Incompatible
C. Titration level
D. Percentage of cells reactive

Review Questions19
1. The practice of using blood groups for parentage testing
in the 1950s was based on which principle? (Objective #2)
A . I nclusion
B. E xclusion
C. Confirmation
D . P robability
2. Select the advantages to using DNA typing instead of
HLA typing for parentage testing. (Objective #7)
A . DNA is more stable.
B. O ld blood specimen can be used.
C. Many specimen types are acceptable.
D . T esting can be performed at any temperature

3. How many short tandem repeat (STR) loci exist in the


human genome? (Objective #8)
A. Less than 100
B. Between 100 and 500
C. Between 500 and 1,000
D. More than 1,000
4. What technology was the first to be used for DNA profiling?
(Objective #6)
A. RFLP
B. PCR
C. STR
D. VNTR
5. How are alleles separated by size after STR loci have been
amplified? (Objective #8)
A. Centrifugation
B. Capillary action
C. Electrophoresis
D. Fluorescence
6. After STR alleles have been separated by size, which process
is needed for specific identification? (Objective #6)
A. Radioactivity
B. Fluorescence
C. Electrophoresis
D. Centrifugation

7. In parentage testing, which hypothesis estimates the likelihood


or probability that the alleged parent is the true
parent of the child? (Objective #10)
A. H0
B. H1
C. H2
D. H3
8. How is the probability of exclusion used in parentage testing?
(Objective #11)
A. It estimates the probability that an alleged parent is
the true parent of the child.
B. It estimates the population that possesses an obligate
allele.
C. It estimates the population that lacks an obligate
allele.
D. It estimates the population frequency of the obligate
allele.
9. The statistical logic that allows a level of certainty for a
hypothesis to be modified by incorporating new information
is referred to as: (Objective #14)
A. Likelihood ratio
B. Parentage
C. Discriminatory power
D. Bayes’ theorem
10. True or False: In parentage testing, it is possible to achieve
a 100% probability that an alleged parent is the true parent
of a child. (Objective #11)

Review Questions20
1. The Federal Food, Drug and Cosmetic Act of 1938 defined
governmental authority over which aspects of the pharmaceutical
industry? (Objective #1)
A . D rug safety
B. D rug labeling
C. L imits for poisonous substances
D . F actory inspections
2. The Kefauver-Harris drug amendments of 1962 required
_____ registration of blood banks. (Objective #1)
A . Monthly
B. Quarterly
C. A nnual
D . Biannual
3. What year did the National Institutes of Health (NIH)
issue the first license for a product produced from human
blood? (Objective #1)
A. 1930
B. 1934
C. 1949
D . 1972
4. Which infectious disease was required to be tested for in
all donated blood as of 1970? (Objective #1)
A . H epatitis B
B. H epatitis C
C. HI V
D . WNV

5. The guidelines that are used for manufacturing and processing


quality products in the blood bank are called:
(Objective #2)
A. CFR
B. FDA
C. cGMPs
D. CLIA
6. Select the FDA requirement for each type of blood bank
facility. (Objective #3)
_____ Small community hospital with a blood bank that
receives donor blood from an outside supplier
_____ Blood donor center that collects and processes
donor blood
_____ Trauma center hospital with blood bank donor room
services for autologous and directed donors
A. Licensure and registration
B. Registration only
C. Exempt from registration and licensure
7. The inspection of an FDA-licensed blood bank facility
includes the evaluation of which of the following aspects
of service? (Objective #4)
A. Standard operating procedures (SOPs)
B. Blood safety practices
C. Equipment validation
D. Shift scheduling
8. Choose the appropriate FDA category for enforcement
based on the action taken. (Objective #5)
_____ A license to practice is suspended
_____ A warning letter is issued
_____ A facility is prosecuted for engaging in llegal practices
_____ Voluntary recall of product by the manufacturer
A. Advisory
B. Recall
C. Administrative
D. Judicial
9. An example of a reportable incident during the manufacture
and processing of blood components is: (Objective #5)
A. Donor blood is discarded due to self-exclusion of the
donor.
B. Blood unit is quarantined pending transmissible disease
test results.
C. Donor reports a diagnosis of hepatitis B 2 weeks after
his last donation.
D. Blood typing of donor unit is repeated due to weak
isoagglutinins.

10. The most frequent biological product deviation category


that is reported by blood bank facilities is: (Objective #5)
A. Donor suitability
B. Component preparation
C. Laboratory testing
D. Labeling
11. According to CLIA regulations, how frequently must a certified
blood bank laboratory be inspected? (Objective #6)
A. Semi-annually
B. Annually
C. Every 2 years
D. Every 5 years
12. How does the responsibility for minimum safety and efficiency
of medical devices differ between the European
and the American regulatory system? (Objective #7)
A. In the United States, responsibility lies with the
government.
B. In the European system, responsibility lies with independent
companies.
C. In the European system, responsibility lies with the
manufacturer.
D. In both systems, responsibility lies with the consumer.
13. Which of the following criteria are required for certification
of medical laboratory professionals? (Objective #8)
A. Completion of an accredited training program
B. Mandated by law
C. Continuing education
D. Passing an examination
14. Select the benefits of seeking voluntary accreditation for
blood banks. (Objective #9)
A. Interaction with experts in the field
B. Error reduction
C. Cost savings
D. Patient-care improvements
15. Which of the following is not a primary responsibility of
the International Organization for Standardization (ISO)?
(Objective #10)
A. Develop technical standards
B. Perform compliance checks of technical standards
C. Outline assessment protocols for compliance with
technical standards
D. Periodic review of published technical standards

Review Questions20
1. A daily check of reagents to ensure that they are functioning
as expected is an example of: (Objective #2)
A. Quality management
B. Quality system
C. Quality control
D. Quality assurance
2. Monitoring performed to ensure that the quality system is
effective is the definition of which of the following terms?
(Objective #1)
A. Quality assurance (QA)
B. Quality control (QC)
C. Quality management (QM)
D. Quality program
3. The quality assurance unit in a blood bank is responsible
for: (Objective #5)
A. All human resource activities
B. Oversight of all activities related to product quality
C. Oversight of all activities related to donor recruitment
D. All strategic planning activities
4. The purpose of a document control system includes which
of the following? Check all that apply. (Objective #9)
A. Ensures only current documents are in use
B. Ensures only executive members change procedures
C. Protects documents from unauthorized changes
D. Protects documents from fire or flood
5. When should calibration of equipment be performed?
(Objective #6)
A. At the same time as specimen testing
B. Prior to initial equipment use
C. After equipment is taken out of service
D. Each day of equipment use
6. An effective employee training program includes all of the
following elements except: (Objective #4)
A . N ew employee orientation
B. Development of threshold limits for retraining
C. Established competency
D. Storage of documents with all other personnel records
7. Employee competency can be established through which
of the following types of activities? Select all that apply.
(Objective #4)
A. Written exam
B. Practical exam
C. Error reporting
D. Direct observation checklist
8. A transfusion service has just purchased a new computer
system. What process must be conducted prior to implementation
of the new system at the facility? (Objective #7)
A. Cost–benefit analysis
B. Error analysis
C. Validation
D. QA audit
9. Which of the following is not required for the development
of a standard operating procedure (SOP)? (Objective
#9)
A. Written in language that can be understood by all
employees
B. Use of a consistent format
C. Must be a text document
D. Must include all critical steps for the procedure
10. Match each validation process with the circumstances for
which it should be performed: (Objective #6)
______ Concurrent validation
______ Prospective validation
______ Retrospective validation
(a) A new process
(b) Inadequate prior validation
(c) During a live run
11. The two purposes of the lookback process are:
(Objective #11)
A. Quarantine or recall in-date blood products
B. Quarantine or recall expired blood products
C. N otify blood donors of disease risk
D. N otify blood recipients of disease risk

12. The internal audit process should be: (Objective #12)


A. Conducted by an accrediting agency such as AABB
B. Spontaneous and unplanned
C. Punitive in nature
D. Performed according to an established plan
13. Which statement regarding the use of quality indicators
as part of a quality monitoring process is false?
(Objective #3)
A. Each quality indicator is evaluated only once.
B. There must be an established threshold for each quality
indicator.
C. A quality indicator must be measurable.
D. A quality indicator may be changed or updated.

14. The governmental agency that sets the standards for


safety in the workplace is: (Objective #14)
A. JCAHO
B. OSHA
C. CAP
D. AABB
15. Which of the following strategies must be included in
a problem-solving plan in order for it to be successful?
(Objective #13)
A. Use of multiple tools
B. Start at the beginning point
C. Put together a team
D. Determine the root cause

ANSWERS

Chapter 1
1. Matching
A–4
B–5
C–1
D–3
E–2
2. A, B, D
3. A
4. B
5. A
6. B
7. B, D
8. B, C, D
9. A, B, C
10. A, B, C, D
11. A, C
12. A, B, C, D
13. C
14. A, B, C, D
15. A, B, D
16. A, C, D
17. A, D
Chapter 2
1. a, b, b, a
2. B
3. D
4. A, C
5. B
6. C
7. C
8. B
9. A
10. A, B, C
Chapter 3
1. A, B, C
2. B
3. A, D
4. B
5. D
6. D
7. A, B, D
8. A
9. B, C
10. A
11. 1. b, 2. a, 3. b, 4. b, 5. a
12. D
13. B
14. B, C
15. B
Chapter 4
1. C
2. A
3. C
4. D
5. B

6. A
7. D
8. A
9. B
10. C
11. D
12. B
13. A
14. C
15. D
Chapter 5
1. A
2. B
3. A, B, C
4. D
5. A, B
6. A, B, C
7. A, B, C, D
8. B
9. A
10. B, C, A, D
11. B, D
12. B
13. D
14. A, B, C
15. B
16. A, B, B, A
17. A
18. B, C
19. A
20. C
Chapter 6
1. A, B, C
2. B
3. A

4. D
5. C
6. B, A, B, A, C
7. A
8. C
9. A, B, C
10. B
11. C
12. C
13. A, C
14. C, D
15. A, B, C, D
Chapter 7
1. A, B
2. A, B, C, D
3. A, B, C
4. A, C
5. A, B, C, D
6. B
7. A, B, C, D
8. A
9. A, B, C
10. C
11. A
12. D
13. B
14. B
15. A
16. A, B, C, D
17. B
Chapter 8
1. C
2. C
3. D

4. D
Step 1: Determine antigen negative frequencies
Jkb: 100 - 73 = 27% _ K: 100 - 10 = 90%
Step 2: Convert % values obtained in step 1 to decimals
Jkb: 27% = .27 _ K: 10% = .90
Step 3: Multiply the values obtained in step 2
.27 * .90 = 0.243
Step 4: Fill-in appropriate values into the formula
3 units desired/0.243 = 12.3 or 12
5. A
6. C
7. B
8. D
9. 1. D; 2. A; 3. B; 4. C
10. C
11. B
12. C
13. A
14. C
15. D
Chapter 9
1. A, B, C, D
2. C
3. A, D
4. 1. A, 2. B, 3. B, 4. A, 5. A
5. A, C
6. 1. A, 2. C, 3. A, 4. B
7. B
8. A
9. B
10. A
11. B, D
12. A, B, C
13. C
14. D
15. B

Chapter 10
1. D
2. B
3. D
4. B
5. B
6. A
7. C
8. A
9. D
10. B
11. C
12. B
13. A
14. D
15. C
16. C
17. B
18. A
19. D
20. C
Chapter 11
1. b, c, e, d, a
2. C
3. A, B, C
4. B, C, F
5. A, C, D
6. C
7. B, C
8. B
9. A, C, D
10. D
11. A, B, C, D
12. a, a, c, b
13. C

14. A
15. A, B, C
Chapter 12
1. A, B, C, D
2. A, B, D, F
3. B
4. D
5. c, d, f, b, a, e
6. A, C
7. B
8. B
9. B, C, D, F
10. A
11. C
12. D
13. A, B
14. A
15. D
Chapter 13
1. B
2. D
3. C
4. C
5. A
6. B
7. D
8. B
9. D
10. C
11. D
12. C
13. A, B, C
14. A, D, C, B
15. D

Chapter 14
1. A, D
2. C
3. B
4. D
5. B, C
6. A
7. C
8. A
9. B, D
10. D
11. b, a, a, b
12. D
13. C; using the formula (% fetal cells in decimal format) *
5,000 mL blood volume/30 mL (amount of fetal whole
blood covered by one vial of RhIG):
Step 1: convert % fetal cells to decimal : 3.2% = 0.032
Step 2: multiple value obtained in step 1 by 5,000 mL
blood volume: (0.032) * (5,000) = 160
Step 3: divide value obtained in step 2 by 30 mL:
160/30 = 5.33 = 5
Step 4: add one to the value obtained in step 3: 5 + 1 = 6
doses
14. A, D
15. C
Chapter 15
1. D
2. B
3. D
4. C
5. A
6. False
7. B
8. A, C, E, F
9. C
10. B
11. C
12. C, A, A, C, B

13. D
14. C
15. B
Chapter 16
1. B
2. D
3. C
CCI =
BSA (m2) * Platelet increment (per _L) * 1011
Number of platelets transfused * 1011
Note: This formula applies when measurement units for
the BSA is m2 and the platelet increment is per _L
Step 1: Determine the platelet increment by subtracting
the pretransfusion platelet count per _L from the
post-transfusion platelet count per _L
Pretransfusion count = 5 * 103
= 5 * 1000 = 5,000 _L
Post-transfusion count = 45 * 103 = 45 * 1000
= 45,000 _L
45,000 - 5,000 = 40,000 _L
Step 2: Insert the platelet increment determined in step 1
into the formula as follows:
1.7 (BsA) * 40,000 (platelet increment) * 1011/4.0
* 1011 (number of platelets transfused)
the 1011 portions cancel out leaving:
1.7 * 40,000/4.0 = 68,000/4.0 = 17,000
4. A, B, C
5. A, B
6. D
7. B
8. C
9. D
10. A
11. A, C
12. B
13. A
14. C
15. B
Chapter 17
1. A, C
2. C
3. B

4. A, B, C, D
5. B
6. A, B
7. C
8. C
9. A, B
10. B
11. C
12. A, B, C, D
13. C
14. B
15. B, D
Chapter 18
1. B
2. A, B, C
3. A, B
4. A
5. B
6. D
7. C
8. 3rd, 1st, 4th, 2nd
9. C
10. B
11. A
12. A
13. C
14. A
15. D
Chapter 19
1. B
2. A, B, C
3. D
4. A
5. C
6. B
7. A

8. C
9. D
10. False
Chapter 20
1. A, B, C, D
2. C
3. B
4. A
5. C
6. C, A, B
7. A, B, C
8. C, A, D, B
9. C
10. A
11. C
12. A
13. A, C, D
14. A, B, C, D
15. B

Chapter 21
1. C
2. A
3. B
4. A, C
5. B
6. D
7. A, B, D
8. C
9. C
10. c, a, b
11. A, D
12. D
13. A
14. B
15. D

Immunohematology and Transfusion Medicine ( PDFDrive )CASE STUDY ALL THE PDF
Transfusion Medicine - Case Studies and Clinical Management - Springer London (2012) ALL
PDF

Transfusion Medicine - Medical Laboratory Science Review 4e 2013 PRIT PDF 49PAGES

Transfusion Medicine - Success in Clinical Laboratory Medicine 4e 2010 ALL PDF

Transfusion Medicine, Apheresis, and Hemostasis - Qs and Case Studies 2018***********

Transfusion Management Of The Obstetracial Patient - A Clinical Casebook 2018 PDF


BOC FROM PDF

You might also like